The Residue Theorem · 2010-10-04 · 5.We will prove the requisite theorem (the Residue Theorem)...

256
logo1 Singularities Residues Residue Theorem Residue at Infinity Types of Isolated Singularities Residues at Poles The Residue Theorem Bernd Schr ¨ oder Bernd Schr¨ oder Louisiana Tech University, College of Engineering and Science The Residue Theorem

Transcript of The Residue Theorem · 2010-10-04 · 5.We will prove the requisite theorem (the Residue Theorem)...

Page 1: The Residue Theorem · 2010-10-04 · 5.We will prove the requisite theorem (the Residue Theorem) in this presentation and we will also lay the abstract groundwork. 6.We will then

logo1

Singularities Residues Residue Theorem Residue at Infinity Types of Isolated Singularities Residues at Poles

The Residue Theorem

Bernd Schroder

Bernd Schroder Louisiana Tech University, College of Engineering and Science

The Residue Theorem

Page 2: The Residue Theorem · 2010-10-04 · 5.We will prove the requisite theorem (the Residue Theorem) in this presentation and we will also lay the abstract groundwork. 6.We will then

logo1

Singularities Residues Residue Theorem Residue at Infinity Types of Isolated Singularities Residues at Poles

Introduction

1. The Cauchy-Goursat Theorem says that if a function is analyticon and in a closed contour C, then the integral over the closedcontour is zero.

2. But what if the function is not analytic?3. We will avoid situations where the function “blows up” (goes to

infinity) on the contour. So we will not need to generalizecontour integrals to “improper contour integrals”.

4. But the situation in which the function is not analytic inside thecontour turns out to be quite interesting.

5. We will prove the requisite theorem (the Residue Theorem) inthis presentation and we will also lay the abstract groundwork.

6. We will then spend an extensive amount of time with examplesthat show how widely applicable the Residue Theorem is.

Bernd Schroder Louisiana Tech University, College of Engineering and Science

The Residue Theorem

Page 3: The Residue Theorem · 2010-10-04 · 5.We will prove the requisite theorem (the Residue Theorem) in this presentation and we will also lay the abstract groundwork. 6.We will then

logo1

Singularities Residues Residue Theorem Residue at Infinity Types of Isolated Singularities Residues at Poles

Introduction1. The Cauchy-Goursat Theorem says that if a function is analytic

on and in a closed contour C, then the integral over the closedcontour is zero.

2. But what if the function is not analytic?3. We will avoid situations where the function “blows up” (goes to

infinity) on the contour. So we will not need to generalizecontour integrals to “improper contour integrals”.

4. But the situation in which the function is not analytic inside thecontour turns out to be quite interesting.

5. We will prove the requisite theorem (the Residue Theorem) inthis presentation and we will also lay the abstract groundwork.

6. We will then spend an extensive amount of time with examplesthat show how widely applicable the Residue Theorem is.

Bernd Schroder Louisiana Tech University, College of Engineering and Science

The Residue Theorem

Page 4: The Residue Theorem · 2010-10-04 · 5.We will prove the requisite theorem (the Residue Theorem) in this presentation and we will also lay the abstract groundwork. 6.We will then

logo1

Singularities Residues Residue Theorem Residue at Infinity Types of Isolated Singularities Residues at Poles

Introduction1. The Cauchy-Goursat Theorem says that if a function is analytic

on and in a closed contour C, then the integral over the closedcontour is zero.

2. But what if the function is not analytic?

3. We will avoid situations where the function “blows up” (goes toinfinity) on the contour. So we will not need to generalizecontour integrals to “improper contour integrals”.

4. But the situation in which the function is not analytic inside thecontour turns out to be quite interesting.

5. We will prove the requisite theorem (the Residue Theorem) inthis presentation and we will also lay the abstract groundwork.

6. We will then spend an extensive amount of time with examplesthat show how widely applicable the Residue Theorem is.

Bernd Schroder Louisiana Tech University, College of Engineering and Science

The Residue Theorem

Page 5: The Residue Theorem · 2010-10-04 · 5.We will prove the requisite theorem (the Residue Theorem) in this presentation and we will also lay the abstract groundwork. 6.We will then

logo1

Singularities Residues Residue Theorem Residue at Infinity Types of Isolated Singularities Residues at Poles

Introduction1. The Cauchy-Goursat Theorem says that if a function is analytic

on and in a closed contour C, then the integral over the closedcontour is zero.

2. But what if the function is not analytic?3. We will avoid situations where the function “blows up” (goes to

infinity) on the contour.

So we will not need to generalizecontour integrals to “improper contour integrals”.

4. But the situation in which the function is not analytic inside thecontour turns out to be quite interesting.

5. We will prove the requisite theorem (the Residue Theorem) inthis presentation and we will also lay the abstract groundwork.

6. We will then spend an extensive amount of time with examplesthat show how widely applicable the Residue Theorem is.

Bernd Schroder Louisiana Tech University, College of Engineering and Science

The Residue Theorem

Page 6: The Residue Theorem · 2010-10-04 · 5.We will prove the requisite theorem (the Residue Theorem) in this presentation and we will also lay the abstract groundwork. 6.We will then

logo1

Singularities Residues Residue Theorem Residue at Infinity Types of Isolated Singularities Residues at Poles

Introduction1. The Cauchy-Goursat Theorem says that if a function is analytic

on and in a closed contour C, then the integral over the closedcontour is zero.

2. But what if the function is not analytic?3. We will avoid situations where the function “blows up” (goes to

infinity) on the contour. So we will not need to generalizecontour integrals to “improper contour integrals”.

4. But the situation in which the function is not analytic inside thecontour turns out to be quite interesting.

5. We will prove the requisite theorem (the Residue Theorem) inthis presentation and we will also lay the abstract groundwork.

6. We will then spend an extensive amount of time with examplesthat show how widely applicable the Residue Theorem is.

Bernd Schroder Louisiana Tech University, College of Engineering and Science

The Residue Theorem

Page 7: The Residue Theorem · 2010-10-04 · 5.We will prove the requisite theorem (the Residue Theorem) in this presentation and we will also lay the abstract groundwork. 6.We will then

logo1

Singularities Residues Residue Theorem Residue at Infinity Types of Isolated Singularities Residues at Poles

Introduction1. The Cauchy-Goursat Theorem says that if a function is analytic

on and in a closed contour C, then the integral over the closedcontour is zero.

2. But what if the function is not analytic?3. We will avoid situations where the function “blows up” (goes to

infinity) on the contour. So we will not need to generalizecontour integrals to “improper contour integrals”.

4. But the situation in which the function is not analytic inside thecontour turns out to be quite interesting.

5. We will prove the requisite theorem (the Residue Theorem) inthis presentation and we will also lay the abstract groundwork.

6. We will then spend an extensive amount of time with examplesthat show how widely applicable the Residue Theorem is.

Bernd Schroder Louisiana Tech University, College of Engineering and Science

The Residue Theorem

Page 8: The Residue Theorem · 2010-10-04 · 5.We will prove the requisite theorem (the Residue Theorem) in this presentation and we will also lay the abstract groundwork. 6.We will then

logo1

Singularities Residues Residue Theorem Residue at Infinity Types of Isolated Singularities Residues at Poles

Introduction1. The Cauchy-Goursat Theorem says that if a function is analytic

on and in a closed contour C, then the integral over the closedcontour is zero.

2. But what if the function is not analytic?3. We will avoid situations where the function “blows up” (goes to

infinity) on the contour. So we will not need to generalizecontour integrals to “improper contour integrals”.

4. But the situation in which the function is not analytic inside thecontour turns out to be quite interesting.

5. We will prove the requisite theorem

(the Residue Theorem) inthis presentation and we will also lay the abstract groundwork.

6. We will then spend an extensive amount of time with examplesthat show how widely applicable the Residue Theorem is.

Bernd Schroder Louisiana Tech University, College of Engineering and Science

The Residue Theorem

Page 9: The Residue Theorem · 2010-10-04 · 5.We will prove the requisite theorem (the Residue Theorem) in this presentation and we will also lay the abstract groundwork. 6.We will then

logo1

Singularities Residues Residue Theorem Residue at Infinity Types of Isolated Singularities Residues at Poles

Introduction1. The Cauchy-Goursat Theorem says that if a function is analytic

on and in a closed contour C, then the integral over the closedcontour is zero.

2. But what if the function is not analytic?3. We will avoid situations where the function “blows up” (goes to

infinity) on the contour. So we will not need to generalizecontour integrals to “improper contour integrals”.

4. But the situation in which the function is not analytic inside thecontour turns out to be quite interesting.

5. We will prove the requisite theorem (the Residue Theorem)

inthis presentation and we will also lay the abstract groundwork.

6. We will then spend an extensive amount of time with examplesthat show how widely applicable the Residue Theorem is.

Bernd Schroder Louisiana Tech University, College of Engineering and Science

The Residue Theorem

Page 10: The Residue Theorem · 2010-10-04 · 5.We will prove the requisite theorem (the Residue Theorem) in this presentation and we will also lay the abstract groundwork. 6.We will then

logo1

Singularities Residues Residue Theorem Residue at Infinity Types of Isolated Singularities Residues at Poles

Introduction1. The Cauchy-Goursat Theorem says that if a function is analytic

on and in a closed contour C, then the integral over the closedcontour is zero.

2. But what if the function is not analytic?3. We will avoid situations where the function “blows up” (goes to

infinity) on the contour. So we will not need to generalizecontour integrals to “improper contour integrals”.

4. But the situation in which the function is not analytic inside thecontour turns out to be quite interesting.

5. We will prove the requisite theorem (the Residue Theorem) inthis presentation and we will also lay the abstract groundwork.

6. We will then spend an extensive amount of time with examplesthat show how widely applicable the Residue Theorem is.

Bernd Schroder Louisiana Tech University, College of Engineering and Science

The Residue Theorem

Page 11: The Residue Theorem · 2010-10-04 · 5.We will prove the requisite theorem (the Residue Theorem) in this presentation and we will also lay the abstract groundwork. 6.We will then

logo1

Singularities Residues Residue Theorem Residue at Infinity Types of Isolated Singularities Residues at Poles

Introduction1. The Cauchy-Goursat Theorem says that if a function is analytic

on and in a closed contour C, then the integral over the closedcontour is zero.

2. But what if the function is not analytic?3. We will avoid situations where the function “blows up” (goes to

infinity) on the contour. So we will not need to generalizecontour integrals to “improper contour integrals”.

4. But the situation in which the function is not analytic inside thecontour turns out to be quite interesting.

5. We will prove the requisite theorem (the Residue Theorem) inthis presentation and we will also lay the abstract groundwork.

6. We will then spend an extensive amount of time with examplesthat show how widely applicable the Residue Theorem is.

Bernd Schroder Louisiana Tech University, College of Engineering and Science

The Residue Theorem

Page 12: The Residue Theorem · 2010-10-04 · 5.We will prove the requisite theorem (the Residue Theorem) in this presentation and we will also lay the abstract groundwork. 6.We will then

logo1

Singularities Residues Residue Theorem Residue at Infinity Types of Isolated Singularities Residues at Poles

Example.

The function f (z) =1

(z−1)(1+ z2)is not analytic at

z = 1, i,−i.

-

6ℑ(z)

ℜ(z)1−1

i

−i

rr

r

Bernd Schroder Louisiana Tech University, College of Engineering and Science

The Residue Theorem

Page 13: The Residue Theorem · 2010-10-04 · 5.We will prove the requisite theorem (the Residue Theorem) in this presentation and we will also lay the abstract groundwork. 6.We will then

logo1

Singularities Residues Residue Theorem Residue at Infinity Types of Isolated Singularities Residues at Poles

Example. The function f (z) =1

(z−1)(1+ z2)is not analytic at

z = 1, i,−i.

-

6ℑ(z)

ℜ(z)1−1

i

−i

rr

r

Bernd Schroder Louisiana Tech University, College of Engineering and Science

The Residue Theorem

Page 14: The Residue Theorem · 2010-10-04 · 5.We will prove the requisite theorem (the Residue Theorem) in this presentation and we will also lay the abstract groundwork. 6.We will then

logo1

Singularities Residues Residue Theorem Residue at Infinity Types of Isolated Singularities Residues at Poles

Example. The function f (z) =1

(z−1)(1+ z2)is not analytic at

z = 1, i,−i.

-

6ℑ(z)

ℜ(z)

1−1

i

−i

rr

r

Bernd Schroder Louisiana Tech University, College of Engineering and Science

The Residue Theorem

Page 15: The Residue Theorem · 2010-10-04 · 5.We will prove the requisite theorem (the Residue Theorem) in this presentation and we will also lay the abstract groundwork. 6.We will then

logo1

Singularities Residues Residue Theorem Residue at Infinity Types of Isolated Singularities Residues at Poles

Example. The function f (z) =1

(z−1)(1+ z2)is not analytic at

z = 1, i,−i.

-

6ℑ(z)

ℜ(z)

1−1

i

−i

rr

r

Bernd Schroder Louisiana Tech University, College of Engineering and Science

The Residue Theorem

Page 16: The Residue Theorem · 2010-10-04 · 5.We will prove the requisite theorem (the Residue Theorem) in this presentation and we will also lay the abstract groundwork. 6.We will then

logo1

Singularities Residues Residue Theorem Residue at Infinity Types of Isolated Singularities Residues at Poles

Example. The function f (z) =1

(z−1)(1+ z2)is not analytic at

z = 1, i,−i.

-

6ℑ(z)

ℜ(z)

1−1

i

−i

rr

r

Bernd Schroder Louisiana Tech University, College of Engineering and Science

The Residue Theorem

Page 17: The Residue Theorem · 2010-10-04 · 5.We will prove the requisite theorem (the Residue Theorem) in this presentation and we will also lay the abstract groundwork. 6.We will then

logo1

Singularities Residues Residue Theorem Residue at Infinity Types of Isolated Singularities Residues at Poles

Example. The function f (z) =1

(z−1)(1+ z2)is not analytic at

z = 1, i,−i.

-

6ℑ(z)

ℜ(z)

1−1

i

−i

rr

r

Bernd Schroder Louisiana Tech University, College of Engineering and Science

The Residue Theorem

Page 18: The Residue Theorem · 2010-10-04 · 5.We will prove the requisite theorem (the Residue Theorem) in this presentation and we will also lay the abstract groundwork. 6.We will then

logo1

Singularities Residues Residue Theorem Residue at Infinity Types of Isolated Singularities Residues at Poles

Example. The function f (z) =1

(z−1)(1+ z2)is not analytic at

z = 1, i,−i.

-

6ℑ(z)

ℜ(z)

1−1

i

−i

rr

r

Bernd Schroder Louisiana Tech University, College of Engineering and Science

The Residue Theorem

Page 19: The Residue Theorem · 2010-10-04 · 5.We will prove the requisite theorem (the Residue Theorem) in this presentation and we will also lay the abstract groundwork. 6.We will then

logo1

Singularities Residues Residue Theorem Residue at Infinity Types of Isolated Singularities Residues at Poles

Example. The function f (z) =1

(z−1)(1+ z2)is not analytic at

z = 1, i,−i.

-

6ℑ(z)

ℜ(z)1

−1

i

−i

rr

r

Bernd Schroder Louisiana Tech University, College of Engineering and Science

The Residue Theorem

Page 20: The Residue Theorem · 2010-10-04 · 5.We will prove the requisite theorem (the Residue Theorem) in this presentation and we will also lay the abstract groundwork. 6.We will then

logo1

Singularities Residues Residue Theorem Residue at Infinity Types of Isolated Singularities Residues at Poles

Example. The function f (z) =1

(z−1)(1+ z2)is not analytic at

z = 1, i,−i.

-

6ℑ(z)

ℜ(z)1−1

i

−i

rr

r

Bernd Schroder Louisiana Tech University, College of Engineering and Science

The Residue Theorem

Page 21: The Residue Theorem · 2010-10-04 · 5.We will prove the requisite theorem (the Residue Theorem) in this presentation and we will also lay the abstract groundwork. 6.We will then

logo1

Singularities Residues Residue Theorem Residue at Infinity Types of Isolated Singularities Residues at Poles

Example. The function f (z) =1

(z−1)(1+ z2)is not analytic at

z = 1, i,−i.

-

6ℑ(z)

ℜ(z)1−1

i

−i

rr

r

Bernd Schroder Louisiana Tech University, College of Engineering and Science

The Residue Theorem

Page 22: The Residue Theorem · 2010-10-04 · 5.We will prove the requisite theorem (the Residue Theorem) in this presentation and we will also lay the abstract groundwork. 6.We will then

logo1

Singularities Residues Residue Theorem Residue at Infinity Types of Isolated Singularities Residues at Poles

Example. The function f (z) =1

(z−1)(1+ z2)is not analytic at

z = 1, i,−i.

-

6ℑ(z)

ℜ(z)1−1

i

−i

rr

r

Bernd Schroder Louisiana Tech University, College of Engineering and Science

The Residue Theorem

Page 23: The Residue Theorem · 2010-10-04 · 5.We will prove the requisite theorem (the Residue Theorem) in this presentation and we will also lay the abstract groundwork. 6.We will then

logo1

Singularities Residues Residue Theorem Residue at Infinity Types of Isolated Singularities Residues at Poles

Example. The function f (z) =1

(z−1)(1+ z2)is not analytic at

z = 1, i,−i.

-

6ℑ(z)

ℜ(z)1−1

i

−i

r

r

r

Bernd Schroder Louisiana Tech University, College of Engineering and Science

The Residue Theorem

Page 24: The Residue Theorem · 2010-10-04 · 5.We will prove the requisite theorem (the Residue Theorem) in this presentation and we will also lay the abstract groundwork. 6.We will then

logo1

Singularities Residues Residue Theorem Residue at Infinity Types of Isolated Singularities Residues at Poles

Example. The function f (z) =1

(z−1)(1+ z2)is not analytic at

z = 1, i,−i.

-

6ℑ(z)

ℜ(z)1−1

i

−i

rr

r

Bernd Schroder Louisiana Tech University, College of Engineering and Science

The Residue Theorem

Page 25: The Residue Theorem · 2010-10-04 · 5.We will prove the requisite theorem (the Residue Theorem) in this presentation and we will also lay the abstract groundwork. 6.We will then

logo1

Singularities Residues Residue Theorem Residue at Infinity Types of Isolated Singularities Residues at Poles

Example. The function f (z) =1

(z−1)(1+ z2)is not analytic at

z = 1, i,−i.

-

6ℑ(z)

ℜ(z)1−1

i

−i

rr

r

Bernd Schroder Louisiana Tech University, College of Engineering and Science

The Residue Theorem

Page 26: The Residue Theorem · 2010-10-04 · 5.We will prove the requisite theorem (the Residue Theorem) in this presentation and we will also lay the abstract groundwork. 6.We will then

logo1

Singularities Residues Residue Theorem Residue at Infinity Types of Isolated Singularities Residues at Poles

Example.

The function f (z) =1

sin(

π

z

) is not analytic at

z = 1,12,13,14, . . . and at 0 and at z =−1,−1

2,−1

3,−1

4, . . ..

-

6ℑ(z)

ℜ(z)1−1

i

−i

rrr· · ·rr r r· · ·

Bernd Schroder Louisiana Tech University, College of Engineering and Science

The Residue Theorem

Page 27: The Residue Theorem · 2010-10-04 · 5.We will prove the requisite theorem (the Residue Theorem) in this presentation and we will also lay the abstract groundwork. 6.We will then

logo1

Singularities Residues Residue Theorem Residue at Infinity Types of Isolated Singularities Residues at Poles

Example. The function f (z) =1

sin(

π

z

) is not analytic at

z = 1,12,13,14, . . . and at 0

and at z =−1,−12,−1

3,−1

4, . . ..

-

6ℑ(z)

ℜ(z)1−1

i

−i

rrr· · ·rr r r· · ·

Bernd Schroder Louisiana Tech University, College of Engineering and Science

The Residue Theorem

Page 28: The Residue Theorem · 2010-10-04 · 5.We will prove the requisite theorem (the Residue Theorem) in this presentation and we will also lay the abstract groundwork. 6.We will then

logo1

Singularities Residues Residue Theorem Residue at Infinity Types of Isolated Singularities Residues at Poles

Example. The function f (z) =1

sin(

π

z

) is not analytic at

z = 1,12,13,14, . . . and at 0 and at z =−1,−1

2,−1

3,−1

4, . . ..

-

6ℑ(z)

ℜ(z)1−1

i

−i

rrr· · ·rr r r· · ·

Bernd Schroder Louisiana Tech University, College of Engineering and Science

The Residue Theorem

Page 29: The Residue Theorem · 2010-10-04 · 5.We will prove the requisite theorem (the Residue Theorem) in this presentation and we will also lay the abstract groundwork. 6.We will then

logo1

Singularities Residues Residue Theorem Residue at Infinity Types of Isolated Singularities Residues at Poles

Example. The function f (z) =1

sin(

π

z

) is not analytic at

z = 1,12,13,14, . . . and at 0 and at z =−1,−1

2,−1

3,−1

4, . . ..

-

6ℑ(z)

ℜ(z)1−1

i

−i

rrr· · ·rr r r· · ·

Bernd Schroder Louisiana Tech University, College of Engineering and Science

The Residue Theorem

Page 30: The Residue Theorem · 2010-10-04 · 5.We will prove the requisite theorem (the Residue Theorem) in this presentation and we will also lay the abstract groundwork. 6.We will then

logo1

Singularities Residues Residue Theorem Residue at Infinity Types of Isolated Singularities Residues at Poles

Example. The function f (z) =1

sin(

π

z

) is not analytic at

z = 1,12,13,14, . . . and at 0 and at z =−1,−1

2,−1

3,−1

4, . . ..

-

6ℑ(z)

ℜ(z)1−1

i

−i

r

rr· · ·rr r r· · ·

Bernd Schroder Louisiana Tech University, College of Engineering and Science

The Residue Theorem

Page 31: The Residue Theorem · 2010-10-04 · 5.We will prove the requisite theorem (the Residue Theorem) in this presentation and we will also lay the abstract groundwork. 6.We will then

logo1

Singularities Residues Residue Theorem Residue at Infinity Types of Isolated Singularities Residues at Poles

Example. The function f (z) =1

sin(

π

z

) is not analytic at

z = 1,12,13,14, . . . and at 0 and at z =−1,−1

2,−1

3,−1

4, . . ..

-

6ℑ(z)

ℜ(z)1−1

i

−i

rr

r· · ·rr r r· · ·

Bernd Schroder Louisiana Tech University, College of Engineering and Science

The Residue Theorem

Page 32: The Residue Theorem · 2010-10-04 · 5.We will prove the requisite theorem (the Residue Theorem) in this presentation and we will also lay the abstract groundwork. 6.We will then

logo1

Singularities Residues Residue Theorem Residue at Infinity Types of Isolated Singularities Residues at Poles

Example. The function f (z) =1

sin(

π

z

) is not analytic at

z = 1,12,13,14, . . . and at 0 and at z =−1,−1

2,−1

3,−1

4, . . ..

-

6ℑ(z)

ℜ(z)1−1

i

−i

rrr

· · ·rr r r· · ·

Bernd Schroder Louisiana Tech University, College of Engineering and Science

The Residue Theorem

Page 33: The Residue Theorem · 2010-10-04 · 5.We will prove the requisite theorem (the Residue Theorem) in this presentation and we will also lay the abstract groundwork. 6.We will then

logo1

Singularities Residues Residue Theorem Residue at Infinity Types of Isolated Singularities Residues at Poles

Example. The function f (z) =1

sin(

π

z

) is not analytic at

z = 1,12,13,14, . . . and at 0 and at z =−1,−1

2,−1

3,−1

4, . . ..

-

6ℑ(z)

ℜ(z)1−1

i

−i

rrr· · ·

rr r r· · ·

Bernd Schroder Louisiana Tech University, College of Engineering and Science

The Residue Theorem

Page 34: The Residue Theorem · 2010-10-04 · 5.We will prove the requisite theorem (the Residue Theorem) in this presentation and we will also lay the abstract groundwork. 6.We will then

logo1

Singularities Residues Residue Theorem Residue at Infinity Types of Isolated Singularities Residues at Poles

Example. The function f (z) =1

sin(

π

z

) is not analytic at

z = 1,12,13,14, . . . and at 0 and at z =−1,−1

2,−1

3,−1

4, . . ..

-

6ℑ(z)

ℜ(z)1−1

i

−i

rrr· · ·r

r r r· · ·

Bernd Schroder Louisiana Tech University, College of Engineering and Science

The Residue Theorem

Page 35: The Residue Theorem · 2010-10-04 · 5.We will prove the requisite theorem (the Residue Theorem) in this presentation and we will also lay the abstract groundwork. 6.We will then

logo1

Singularities Residues Residue Theorem Residue at Infinity Types of Isolated Singularities Residues at Poles

Example. The function f (z) =1

sin(

π

z

) is not analytic at

z = 1,12,13,14, . . . and at 0 and at z =−1,−1

2,−1

3,−1

4, . . ..

-

6ℑ(z)

ℜ(z)1−1

i

−i

rrr· · ·rr

r r· · ·

Bernd Schroder Louisiana Tech University, College of Engineering and Science

The Residue Theorem

Page 36: The Residue Theorem · 2010-10-04 · 5.We will prove the requisite theorem (the Residue Theorem) in this presentation and we will also lay the abstract groundwork. 6.We will then

logo1

Singularities Residues Residue Theorem Residue at Infinity Types of Isolated Singularities Residues at Poles

Example. The function f (z) =1

sin(

π

z

) is not analytic at

z = 1,12,13,14, . . . and at 0 and at z =−1,−1

2,−1

3,−1

4, . . ..

-

6ℑ(z)

ℜ(z)1−1

i

−i

rrr· · ·rr r

r· · ·

Bernd Schroder Louisiana Tech University, College of Engineering and Science

The Residue Theorem

Page 37: The Residue Theorem · 2010-10-04 · 5.We will prove the requisite theorem (the Residue Theorem) in this presentation and we will also lay the abstract groundwork. 6.We will then

logo1

Singularities Residues Residue Theorem Residue at Infinity Types of Isolated Singularities Residues at Poles

Example. The function f (z) =1

sin(

π

z

) is not analytic at

z = 1,12,13,14, . . . and at 0 and at z =−1,−1

2,−1

3,−1

4, . . ..

-

6ℑ(z)

ℜ(z)1−1

i

−i

rrr· · ·rr r r

· · ·

Bernd Schroder Louisiana Tech University, College of Engineering and Science

The Residue Theorem

Page 38: The Residue Theorem · 2010-10-04 · 5.We will prove the requisite theorem (the Residue Theorem) in this presentation and we will also lay the abstract groundwork. 6.We will then

logo1

Singularities Residues Residue Theorem Residue at Infinity Types of Isolated Singularities Residues at Poles

Example. The function f (z) =1

sin(

π

z

) is not analytic at

z = 1,12,13,14, . . . and at 0 and at z =−1,−1

2,−1

3,−1

4, . . ..

-

6ℑ(z)

ℜ(z)1−1

i

−i

rrr· · ·rr r r· · ·

Bernd Schroder Louisiana Tech University, College of Engineering and Science

The Residue Theorem

Page 39: The Residue Theorem · 2010-10-04 · 5.We will prove the requisite theorem (the Residue Theorem) in this presentation and we will also lay the abstract groundwork. 6.We will then

logo1

Singularities Residues Residue Theorem Residue at Infinity Types of Isolated Singularities Residues at Poles

Definition.

If the function f is analytic for 0 < |z− z0|< R, then z0 iscalled an isolated singularity of f .

Notes.1. z = 1, i,−i are isolated singularities of f (z) =

1(z−1)(1+ z2)

.

2. z = 1,12,13,14, . . . are isolated singularities of f (z) =

1sin(

π

z

) .

But 0 is not an isolated singularity of f (z) =1

sin(

π

z

) .

3. 0 is not an isolated singularity of f (z) = Log(z) or of any rootfunction. (Remember that every branch cut must contain zero, sothese functions will not be analytic on a set 0 < |z|< R.)

Bernd Schroder Louisiana Tech University, College of Engineering and Science

The Residue Theorem

Page 40: The Residue Theorem · 2010-10-04 · 5.We will prove the requisite theorem (the Residue Theorem) in this presentation and we will also lay the abstract groundwork. 6.We will then

logo1

Singularities Residues Residue Theorem Residue at Infinity Types of Isolated Singularities Residues at Poles

Definition. If the function f is analytic for 0 < |z− z0|< R

, then z0 iscalled an isolated singularity of f .

Notes.1. z = 1, i,−i are isolated singularities of f (z) =

1(z−1)(1+ z2)

.

2. z = 1,12,13,14, . . . are isolated singularities of f (z) =

1sin(

π

z

) .

But 0 is not an isolated singularity of f (z) =1

sin(

π

z

) .

3. 0 is not an isolated singularity of f (z) = Log(z) or of any rootfunction. (Remember that every branch cut must contain zero, sothese functions will not be analytic on a set 0 < |z|< R.)

Bernd Schroder Louisiana Tech University, College of Engineering and Science

The Residue Theorem

Page 41: The Residue Theorem · 2010-10-04 · 5.We will prove the requisite theorem (the Residue Theorem) in this presentation and we will also lay the abstract groundwork. 6.We will then

logo1

Singularities Residues Residue Theorem Residue at Infinity Types of Isolated Singularities Residues at Poles

Definition. If the function f is analytic for 0 < |z− z0|< R, then z0 iscalled an isolated singularity of f .

Notes.1. z = 1, i,−i are isolated singularities of f (z) =

1(z−1)(1+ z2)

.

2. z = 1,12,13,14, . . . are isolated singularities of f (z) =

1sin(

π

z

) .

But 0 is not an isolated singularity of f (z) =1

sin(

π

z

) .

3. 0 is not an isolated singularity of f (z) = Log(z) or of any rootfunction. (Remember that every branch cut must contain zero, sothese functions will not be analytic on a set 0 < |z|< R.)

Bernd Schroder Louisiana Tech University, College of Engineering and Science

The Residue Theorem

Page 42: The Residue Theorem · 2010-10-04 · 5.We will prove the requisite theorem (the Residue Theorem) in this presentation and we will also lay the abstract groundwork. 6.We will then

logo1

Singularities Residues Residue Theorem Residue at Infinity Types of Isolated Singularities Residues at Poles

Definition. If the function f is analytic for 0 < |z− z0|< R, then z0 iscalled an isolated singularity of f .

Notes.

1. z = 1, i,−i are isolated singularities of f (z) =1

(z−1)(1+ z2).

2. z = 1,12,13,14, . . . are isolated singularities of f (z) =

1sin(

π

z

) .

But 0 is not an isolated singularity of f (z) =1

sin(

π

z

) .

3. 0 is not an isolated singularity of f (z) = Log(z) or of any rootfunction. (Remember that every branch cut must contain zero, sothese functions will not be analytic on a set 0 < |z|< R.)

Bernd Schroder Louisiana Tech University, College of Engineering and Science

The Residue Theorem

Page 43: The Residue Theorem · 2010-10-04 · 5.We will prove the requisite theorem (the Residue Theorem) in this presentation and we will also lay the abstract groundwork. 6.We will then

logo1

Singularities Residues Residue Theorem Residue at Infinity Types of Isolated Singularities Residues at Poles

Definition. If the function f is analytic for 0 < |z− z0|< R, then z0 iscalled an isolated singularity of f .

Notes.1. z = 1, i,−i are isolated singularities of f (z) =

1(z−1)(1+ z2)

.

2. z = 1,12,13,14, . . . are isolated singularities of f (z) =

1sin(

π

z

) .

But 0 is not an isolated singularity of f (z) =1

sin(

π

z

) .

3. 0 is not an isolated singularity of f (z) = Log(z) or of any rootfunction. (Remember that every branch cut must contain zero, sothese functions will not be analytic on a set 0 < |z|< R.)

Bernd Schroder Louisiana Tech University, College of Engineering and Science

The Residue Theorem

Page 44: The Residue Theorem · 2010-10-04 · 5.We will prove the requisite theorem (the Residue Theorem) in this presentation and we will also lay the abstract groundwork. 6.We will then

logo1

Singularities Residues Residue Theorem Residue at Infinity Types of Isolated Singularities Residues at Poles

Definition. If the function f is analytic for 0 < |z− z0|< R, then z0 iscalled an isolated singularity of f .

Notes.1. z = 1, i,−i are isolated singularities of f (z) =

1(z−1)(1+ z2)

.

2. z = 1,12,13,14, . . . are isolated singularities of f (z) =

1sin(

π

z

) .

But 0 is not an isolated singularity of f (z) =1

sin(

π

z

) .

3. 0 is not an isolated singularity of f (z) = Log(z) or of any rootfunction. (Remember that every branch cut must contain zero, sothese functions will not be analytic on a set 0 < |z|< R.)

Bernd Schroder Louisiana Tech University, College of Engineering and Science

The Residue Theorem

Page 45: The Residue Theorem · 2010-10-04 · 5.We will prove the requisite theorem (the Residue Theorem) in this presentation and we will also lay the abstract groundwork. 6.We will then

logo1

Singularities Residues Residue Theorem Residue at Infinity Types of Isolated Singularities Residues at Poles

Definition. If the function f is analytic for 0 < |z− z0|< R, then z0 iscalled an isolated singularity of f .

Notes.1. z = 1, i,−i are isolated singularities of f (z) =

1(z−1)(1+ z2)

.

2. z = 1,12,13,14, . . . are isolated singularities of f (z) =

1sin(

π

z

) .

But 0 is not an isolated singularity of f (z) =1

sin(

π

z

) .

3. 0 is not an isolated singularity of f (z) = Log(z) or of any rootfunction. (Remember that every branch cut must contain zero, sothese functions will not be analytic on a set 0 < |z|< R.)

Bernd Schroder Louisiana Tech University, College of Engineering and Science

The Residue Theorem

Page 46: The Residue Theorem · 2010-10-04 · 5.We will prove the requisite theorem (the Residue Theorem) in this presentation and we will also lay the abstract groundwork. 6.We will then

logo1

Singularities Residues Residue Theorem Residue at Infinity Types of Isolated Singularities Residues at Poles

Definition. If the function f is analytic for 0 < |z− z0|< R, then z0 iscalled an isolated singularity of f .

Notes.1. z = 1, i,−i are isolated singularities of f (z) =

1(z−1)(1+ z2)

.

2. z = 1,12,13,14, . . . are isolated singularities of f (z) =

1sin(

π

z

) .

But 0 is not an isolated singularity of f (z) =1

sin(

π

z

) .

3. 0 is not an isolated singularity of f (z) = Log(z) or of any rootfunction.

(Remember that every branch cut must contain zero, sothese functions will not be analytic on a set 0 < |z|< R.)

Bernd Schroder Louisiana Tech University, College of Engineering and Science

The Residue Theorem

Page 47: The Residue Theorem · 2010-10-04 · 5.We will prove the requisite theorem (the Residue Theorem) in this presentation and we will also lay the abstract groundwork. 6.We will then

logo1

Singularities Residues Residue Theorem Residue at Infinity Types of Isolated Singularities Residues at Poles

Definition. If the function f is analytic for 0 < |z− z0|< R, then z0 iscalled an isolated singularity of f .

Notes.1. z = 1, i,−i are isolated singularities of f (z) =

1(z−1)(1+ z2)

.

2. z = 1,12,13,14, . . . are isolated singularities of f (z) =

1sin(

π

z

) .

But 0 is not an isolated singularity of f (z) =1

sin(

π

z

) .

3. 0 is not an isolated singularity of f (z) = Log(z) or of any rootfunction. (Remember that every branch cut must contain zero

, sothese functions will not be analytic on a set 0 < |z|< R.)

Bernd Schroder Louisiana Tech University, College of Engineering and Science

The Residue Theorem

Page 48: The Residue Theorem · 2010-10-04 · 5.We will prove the requisite theorem (the Residue Theorem) in this presentation and we will also lay the abstract groundwork. 6.We will then

logo1

Singularities Residues Residue Theorem Residue at Infinity Types of Isolated Singularities Residues at Poles

Definition. If the function f is analytic for 0 < |z− z0|< R, then z0 iscalled an isolated singularity of f .

Notes.1. z = 1, i,−i are isolated singularities of f (z) =

1(z−1)(1+ z2)

.

2. z = 1,12,13,14, . . . are isolated singularities of f (z) =

1sin(

π

z

) .

But 0 is not an isolated singularity of f (z) =1

sin(

π

z

) .

3. 0 is not an isolated singularity of f (z) = Log(z) or of any rootfunction. (Remember that every branch cut must contain zero, sothese functions will not be analytic on a set 0 < |z|< R.)

Bernd Schroder Louisiana Tech University, College of Engineering and Science

The Residue Theorem

Page 49: The Residue Theorem · 2010-10-04 · 5.We will prove the requisite theorem (the Residue Theorem) in this presentation and we will also lay the abstract groundwork. 6.We will then

logo1

Singularities Residues Residue Theorem Residue at Infinity Types of Isolated Singularities Residues at Poles

Definition.

If the function f is analytic for 0 < |z− z0|< R, then it has

a Laurent expansion∞

∑n=−∞

cn(z− z0)n about z0. The coefficient c−1 is

called the residue of f at z0. It is also denoted Resz=z0(f ) := c−1.

The next result will show the relevance of residues.

Bernd Schroder Louisiana Tech University, College of Engineering and Science

The Residue Theorem

Page 50: The Residue Theorem · 2010-10-04 · 5.We will prove the requisite theorem (the Residue Theorem) in this presentation and we will also lay the abstract groundwork. 6.We will then

logo1

Singularities Residues Residue Theorem Residue at Infinity Types of Isolated Singularities Residues at Poles

Definition. If the function f is analytic for 0 < |z− z0|< R

, then it has

a Laurent expansion∞

∑n=−∞

cn(z− z0)n about z0. The coefficient c−1 is

called the residue of f at z0. It is also denoted Resz=z0(f ) := c−1.

The next result will show the relevance of residues.

Bernd Schroder Louisiana Tech University, College of Engineering and Science

The Residue Theorem

Page 51: The Residue Theorem · 2010-10-04 · 5.We will prove the requisite theorem (the Residue Theorem) in this presentation and we will also lay the abstract groundwork. 6.We will then

logo1

Singularities Residues Residue Theorem Residue at Infinity Types of Isolated Singularities Residues at Poles

Definition. If the function f is analytic for 0 < |z− z0|< R, then it has

a Laurent expansion∞

∑n=−∞

cn(z− z0)n about z0.

The coefficient c−1 is

called the residue of f at z0. It is also denoted Resz=z0(f ) := c−1.

The next result will show the relevance of residues.

Bernd Schroder Louisiana Tech University, College of Engineering and Science

The Residue Theorem

Page 52: The Residue Theorem · 2010-10-04 · 5.We will prove the requisite theorem (the Residue Theorem) in this presentation and we will also lay the abstract groundwork. 6.We will then

logo1

Singularities Residues Residue Theorem Residue at Infinity Types of Isolated Singularities Residues at Poles

Definition. If the function f is analytic for 0 < |z− z0|< R, then it has

a Laurent expansion∞

∑n=−∞

cn(z− z0)n about z0. The coefficient c−1 is

called the residue of f at z0.

It is also denoted Resz=z0(f ) := c−1.

The next result will show the relevance of residues.

Bernd Schroder Louisiana Tech University, College of Engineering and Science

The Residue Theorem

Page 53: The Residue Theorem · 2010-10-04 · 5.We will prove the requisite theorem (the Residue Theorem) in this presentation and we will also lay the abstract groundwork. 6.We will then

logo1

Singularities Residues Residue Theorem Residue at Infinity Types of Isolated Singularities Residues at Poles

Definition. If the function f is analytic for 0 < |z− z0|< R, then it has

a Laurent expansion∞

∑n=−∞

cn(z− z0)n about z0. The coefficient c−1 is

called the residue of f at z0. It is also denoted Resz=z0(f ) := c−1.

The next result will show the relevance of residues.

Bernd Schroder Louisiana Tech University, College of Engineering and Science

The Residue Theorem

Page 54: The Residue Theorem · 2010-10-04 · 5.We will prove the requisite theorem (the Residue Theorem) in this presentation and we will also lay the abstract groundwork. 6.We will then

logo1

Singularities Residues Residue Theorem Residue at Infinity Types of Isolated Singularities Residues at Poles

Definition. If the function f is analytic for 0 < |z− z0|< R, then it has

a Laurent expansion∞

∑n=−∞

cn(z− z0)n about z0. The coefficient c−1 is

called the residue of f at z0. It is also denoted Resz=z0(f ) := c−1.

The next result will show the relevance of residues.

Bernd Schroder Louisiana Tech University, College of Engineering and Science

The Residue Theorem

Page 55: The Residue Theorem · 2010-10-04 · 5.We will prove the requisite theorem (the Residue Theorem) in this presentation and we will also lay the abstract groundwork. 6.We will then

logo1

Singularities Residues Residue Theorem Residue at Infinity Types of Isolated Singularities Residues at Poles

Theorem.

If the function f is analytic for 0 < |z− z0|< R and C is apositively oriented simple closed contour around z0 that is contained

in 0 < |z− z0|< R, then∫

Cf (z) dz = 2πiResz=z0(f ).

Proof. From the theorem on Laurent expansions, we have that

Resz=z0(f ) = a−1 =1

2πi

∮C

f (ξ )(ξ − z0)(−1)+1 dξ =

12πi

∮C

f (ξ ) dξ ,

where C is any circle around z0 with radius < R. Replacement of thecircle with any contour around the origin requires an argument similarto the one that shows that we can use circles of any radius.

Bernd Schroder Louisiana Tech University, College of Engineering and Science

The Residue Theorem

Page 56: The Residue Theorem · 2010-10-04 · 5.We will prove the requisite theorem (the Residue Theorem) in this presentation and we will also lay the abstract groundwork. 6.We will then

logo1

Singularities Residues Residue Theorem Residue at Infinity Types of Isolated Singularities Residues at Poles

Theorem. If the function f is analytic for 0 < |z− z0|< R

and C is apositively oriented simple closed contour around z0 that is contained

in 0 < |z− z0|< R, then∫

Cf (z) dz = 2πiResz=z0(f ).

Proof. From the theorem on Laurent expansions, we have that

Resz=z0(f ) = a−1 =1

2πi

∮C

f (ξ )(ξ − z0)(−1)+1 dξ =

12πi

∮C

f (ξ ) dξ ,

where C is any circle around z0 with radius < R. Replacement of thecircle with any contour around the origin requires an argument similarto the one that shows that we can use circles of any radius.

Bernd Schroder Louisiana Tech University, College of Engineering and Science

The Residue Theorem

Page 57: The Residue Theorem · 2010-10-04 · 5.We will prove the requisite theorem (the Residue Theorem) in this presentation and we will also lay the abstract groundwork. 6.We will then

logo1

Singularities Residues Residue Theorem Residue at Infinity Types of Isolated Singularities Residues at Poles

Theorem. If the function f is analytic for 0 < |z− z0|< R and C is apositively oriented simple closed contour around z0 that is contained

in 0 < |z− z0|< R

, then∫

Cf (z) dz = 2πiResz=z0(f ).

Proof. From the theorem on Laurent expansions, we have that

Resz=z0(f ) = a−1 =1

2πi

∮C

f (ξ )(ξ − z0)(−1)+1 dξ =

12πi

∮C

f (ξ ) dξ ,

where C is any circle around z0 with radius < R. Replacement of thecircle with any contour around the origin requires an argument similarto the one that shows that we can use circles of any radius.

Bernd Schroder Louisiana Tech University, College of Engineering and Science

The Residue Theorem

Page 58: The Residue Theorem · 2010-10-04 · 5.We will prove the requisite theorem (the Residue Theorem) in this presentation and we will also lay the abstract groundwork. 6.We will then

logo1

Singularities Residues Residue Theorem Residue at Infinity Types of Isolated Singularities Residues at Poles

Theorem. If the function f is analytic for 0 < |z− z0|< R and C is apositively oriented simple closed contour around z0 that is contained

in 0 < |z− z0|< R, then∫

Cf (z) dz = 2πiResz=z0(f ).

Proof. From the theorem on Laurent expansions, we have that

Resz=z0(f ) = a−1 =1

2πi

∮C

f (ξ )(ξ − z0)(−1)+1 dξ =

12πi

∮C

f (ξ ) dξ ,

where C is any circle around z0 with radius < R. Replacement of thecircle with any contour around the origin requires an argument similarto the one that shows that we can use circles of any radius.

Bernd Schroder Louisiana Tech University, College of Engineering and Science

The Residue Theorem

Page 59: The Residue Theorem · 2010-10-04 · 5.We will prove the requisite theorem (the Residue Theorem) in this presentation and we will also lay the abstract groundwork. 6.We will then

logo1

Singularities Residues Residue Theorem Residue at Infinity Types of Isolated Singularities Residues at Poles

Theorem. If the function f is analytic for 0 < |z− z0|< R and C is apositively oriented simple closed contour around z0 that is contained

in 0 < |z− z0|< R, then∫

Cf (z) dz = 2πiResz=z0(f ).

Proof.

From the theorem on Laurent expansions, we have that

Resz=z0(f ) = a−1 =1

2πi

∮C

f (ξ )(ξ − z0)(−1)+1 dξ =

12πi

∮C

f (ξ ) dξ ,

where C is any circle around z0 with radius < R. Replacement of thecircle with any contour around the origin requires an argument similarto the one that shows that we can use circles of any radius.

Bernd Schroder Louisiana Tech University, College of Engineering and Science

The Residue Theorem

Page 60: The Residue Theorem · 2010-10-04 · 5.We will prove the requisite theorem (the Residue Theorem) in this presentation and we will also lay the abstract groundwork. 6.We will then

logo1

Singularities Residues Residue Theorem Residue at Infinity Types of Isolated Singularities Residues at Poles

Theorem. If the function f is analytic for 0 < |z− z0|< R and C is apositively oriented simple closed contour around z0 that is contained

in 0 < |z− z0|< R, then∫

Cf (z) dz = 2πiResz=z0(f ).

Proof. From the theorem on Laurent expansions, we have that

Resz=z0(f )

= a−1 =1

2πi

∮C

f (ξ )(ξ − z0)(−1)+1 dξ =

12πi

∮C

f (ξ ) dξ ,

where C is any circle around z0 with radius < R. Replacement of thecircle with any contour around the origin requires an argument similarto the one that shows that we can use circles of any radius.

Bernd Schroder Louisiana Tech University, College of Engineering and Science

The Residue Theorem

Page 61: The Residue Theorem · 2010-10-04 · 5.We will prove the requisite theorem (the Residue Theorem) in this presentation and we will also lay the abstract groundwork. 6.We will then

logo1

Singularities Residues Residue Theorem Residue at Infinity Types of Isolated Singularities Residues at Poles

Theorem. If the function f is analytic for 0 < |z− z0|< R and C is apositively oriented simple closed contour around z0 that is contained

in 0 < |z− z0|< R, then∫

Cf (z) dz = 2πiResz=z0(f ).

Proof. From the theorem on Laurent expansions, we have that

Resz=z0(f ) = a−1

=1

2πi

∮C

f (ξ )(ξ − z0)(−1)+1 dξ =

12πi

∮C

f (ξ ) dξ ,

where C is any circle around z0 with radius < R. Replacement of thecircle with any contour around the origin requires an argument similarto the one that shows that we can use circles of any radius.

Bernd Schroder Louisiana Tech University, College of Engineering and Science

The Residue Theorem

Page 62: The Residue Theorem · 2010-10-04 · 5.We will prove the requisite theorem (the Residue Theorem) in this presentation and we will also lay the abstract groundwork. 6.We will then

logo1

Singularities Residues Residue Theorem Residue at Infinity Types of Isolated Singularities Residues at Poles

Theorem. If the function f is analytic for 0 < |z− z0|< R and C is apositively oriented simple closed contour around z0 that is contained

in 0 < |z− z0|< R, then∫

Cf (z) dz = 2πiResz=z0(f ).

Proof. From the theorem on Laurent expansions, we have that

Resz=z0(f ) = a−1 =1

2πi

∮C

f (ξ )(ξ − z0)(−1)+1 dξ

=1

2πi

∮C

f (ξ ) dξ ,

where C is any circle around z0 with radius < R. Replacement of thecircle with any contour around the origin requires an argument similarto the one that shows that we can use circles of any radius.

Bernd Schroder Louisiana Tech University, College of Engineering and Science

The Residue Theorem

Page 63: The Residue Theorem · 2010-10-04 · 5.We will prove the requisite theorem (the Residue Theorem) in this presentation and we will also lay the abstract groundwork. 6.We will then

logo1

Singularities Residues Residue Theorem Residue at Infinity Types of Isolated Singularities Residues at Poles

Theorem. If the function f is analytic for 0 < |z− z0|< R and C is apositively oriented simple closed contour around z0 that is contained

in 0 < |z− z0|< R, then∫

Cf (z) dz = 2πiResz=z0(f ).

Proof. From the theorem on Laurent expansions, we have that

Resz=z0(f ) = a−1 =1

2πi

∮C

f (ξ )(ξ − z0)(−1)+1 dξ =

12πi

∮C

f (ξ ) dξ

,

where C is any circle around z0 with radius < R. Replacement of thecircle with any contour around the origin requires an argument similarto the one that shows that we can use circles of any radius.

Bernd Schroder Louisiana Tech University, College of Engineering and Science

The Residue Theorem

Page 64: The Residue Theorem · 2010-10-04 · 5.We will prove the requisite theorem (the Residue Theorem) in this presentation and we will also lay the abstract groundwork. 6.We will then

logo1

Singularities Residues Residue Theorem Residue at Infinity Types of Isolated Singularities Residues at Poles

Theorem. If the function f is analytic for 0 < |z− z0|< R and C is apositively oriented simple closed contour around z0 that is contained

in 0 < |z− z0|< R, then∫

Cf (z) dz = 2πiResz=z0(f ).

Proof. From the theorem on Laurent expansions, we have that

Resz=z0(f ) = a−1 =1

2πi

∮C

f (ξ )(ξ − z0)(−1)+1 dξ =

12πi

∮C

f (ξ ) dξ ,

where C is any circle around z0 with radius < R.

Replacement of thecircle with any contour around the origin requires an argument similarto the one that shows that we can use circles of any radius.

Bernd Schroder Louisiana Tech University, College of Engineering and Science

The Residue Theorem

Page 65: The Residue Theorem · 2010-10-04 · 5.We will prove the requisite theorem (the Residue Theorem) in this presentation and we will also lay the abstract groundwork. 6.We will then

logo1

Singularities Residues Residue Theorem Residue at Infinity Types of Isolated Singularities Residues at Poles

Theorem. If the function f is analytic for 0 < |z− z0|< R and C is apositively oriented simple closed contour around z0 that is contained

in 0 < |z− z0|< R, then∫

Cf (z) dz = 2πiResz=z0(f ).

Proof. From the theorem on Laurent expansions, we have that

Resz=z0(f ) = a−1 =1

2πi

∮C

f (ξ )(ξ − z0)(−1)+1 dξ =

12πi

∮C

f (ξ ) dξ ,

where C is any circle around z0 with radius < R. Replacement of thecircle with any contour around the origin requires an argument similarto the one that shows that we can use circles of any radius.

Bernd Schroder Louisiana Tech University, College of Engineering and Science

The Residue Theorem

Page 66: The Residue Theorem · 2010-10-04 · 5.We will prove the requisite theorem (the Residue Theorem) in this presentation and we will also lay the abstract groundwork. 6.We will then

logo1

Singularities Residues Residue Theorem Residue at Infinity Types of Isolated Singularities Residues at Poles

Proof (concl.)

r-�

?O

-

-

1

]

C

0

rrrrIR

Bernd Schroder Louisiana Tech University, College of Engineering and Science

The Residue Theorem

Page 67: The Residue Theorem · 2010-10-04 · 5.We will prove the requisite theorem (the Residue Theorem) in this presentation and we will also lay the abstract groundwork. 6.We will then

logo1

Singularities Residues Residue Theorem Residue at Infinity Types of Isolated Singularities Residues at Poles

Proof (concl.)

r-�

?O

-

-

1

]

C

0

rrrrIR

Bernd Schroder Louisiana Tech University, College of Engineering and Science

The Residue Theorem

Page 68: The Residue Theorem · 2010-10-04 · 5.We will prove the requisite theorem (the Residue Theorem) in this presentation and we will also lay the abstract groundwork. 6.We will then

logo1

Singularities Residues Residue Theorem Residue at Infinity Types of Isolated Singularities Residues at Poles

Proof (concl.)

r-�

?O

-

-

1

]

C

0

r

rrrIR

Bernd Schroder Louisiana Tech University, College of Engineering and Science

The Residue Theorem

Page 69: The Residue Theorem · 2010-10-04 · 5.We will prove the requisite theorem (the Residue Theorem) in this presentation and we will also lay the abstract groundwork. 6.We will then

logo1

Singularities Residues Residue Theorem Residue at Infinity Types of Isolated Singularities Residues at Poles

Proof (concl.)

r-�

?O

-

-

1

]

C

0

rr

rrIR

Bernd Schroder Louisiana Tech University, College of Engineering and Science

The Residue Theorem

Page 70: The Residue Theorem · 2010-10-04 · 5.We will prove the requisite theorem (the Residue Theorem) in this presentation and we will also lay the abstract groundwork. 6.We will then

logo1

Singularities Residues Residue Theorem Residue at Infinity Types of Isolated Singularities Residues at Poles

Proof (concl.)

r-�

?O

-

-

1

]

C

0

rrr

rIR

Bernd Schroder Louisiana Tech University, College of Engineering and Science

The Residue Theorem

Page 71: The Residue Theorem · 2010-10-04 · 5.We will prove the requisite theorem (the Residue Theorem) in this presentation and we will also lay the abstract groundwork. 6.We will then

logo1

Singularities Residues Residue Theorem Residue at Infinity Types of Isolated Singularities Residues at Poles

Proof (concl.)

r-�

?O

-

-

1

]

C

0

rrrr

I

R

Bernd Schroder Louisiana Tech University, College of Engineering and Science

The Residue Theorem

Page 72: The Residue Theorem · 2010-10-04 · 5.We will prove the requisite theorem (the Residue Theorem) in this presentation and we will also lay the abstract groundwork. 6.We will then

logo1

Singularities Residues Residue Theorem Residue at Infinity Types of Isolated Singularities Residues at Poles

Proof (concl.)

r-�

?O

-

-

1

]

C

0

rrrrI

R

Bernd Schroder Louisiana Tech University, College of Engineering and Science

The Residue Theorem

Page 73: The Residue Theorem · 2010-10-04 · 5.We will prove the requisite theorem (the Residue Theorem) in this presentation and we will also lay the abstract groundwork. 6.We will then

logo1

Singularities Residues Residue Theorem Residue at Infinity Types of Isolated Singularities Residues at Poles

Proof (concl.)

r-�

?O

-

-

1

]

C

0

rrrrIR

Bernd Schroder Louisiana Tech University, College of Engineering and Science

The Residue Theorem

Page 74: The Residue Theorem · 2010-10-04 · 5.We will prove the requisite theorem (the Residue Theorem) in this presentation and we will also lay the abstract groundwork. 6.We will then

logo1

Singularities Residues Residue Theorem Residue at Infinity Types of Isolated Singularities Residues at Poles

Proof (concl.)

r-�

?O

-

-

1

]

C

0

rrrrIR

Bernd Schroder Louisiana Tech University, College of Engineering and Science

The Residue Theorem

Page 75: The Residue Theorem · 2010-10-04 · 5.We will prove the requisite theorem (the Residue Theorem) in this presentation and we will also lay the abstract groundwork. 6.We will then

logo1

Singularities Residues Residue Theorem Residue at Infinity Types of Isolated Singularities Residues at Poles

Example.

Let C be the unit circle, traversed in the positive

orientation. Then∫

C

ez

zdz = 2πi

The functionez

zhas only the singularity at 0 inside the contour. The

residue ofez

z=

1z

+∞

∑n=0

zn

(n+1)!at z = 0 is 1. Now apply the

preceding theorem.Note that direct computation gives the same result, because the

integral of∞

∑n=0

zn

(n+1)!over any closed contour is 0 and

∫C

1z

dz =∫ 2π

0

1eiθ ieiθ dθ = 2πi.

Bernd Schroder Louisiana Tech University, College of Engineering and Science

The Residue Theorem

Page 76: The Residue Theorem · 2010-10-04 · 5.We will prove the requisite theorem (the Residue Theorem) in this presentation and we will also lay the abstract groundwork. 6.We will then

logo1

Singularities Residues Residue Theorem Residue at Infinity Types of Isolated Singularities Residues at Poles

Example. Let C be the unit circle, traversed in the positive

orientation.

Then∫

C

ez

zdz = 2πi

The functionez

zhas only the singularity at 0 inside the contour. The

residue ofez

z=

1z

+∞

∑n=0

zn

(n+1)!at z = 0 is 1. Now apply the

preceding theorem.Note that direct computation gives the same result, because the

integral of∞

∑n=0

zn

(n+1)!over any closed contour is 0 and

∫C

1z

dz =∫ 2π

0

1eiθ ieiθ dθ = 2πi.

Bernd Schroder Louisiana Tech University, College of Engineering and Science

The Residue Theorem

Page 77: The Residue Theorem · 2010-10-04 · 5.We will prove the requisite theorem (the Residue Theorem) in this presentation and we will also lay the abstract groundwork. 6.We will then

logo1

Singularities Residues Residue Theorem Residue at Infinity Types of Isolated Singularities Residues at Poles

Example. Let C be the unit circle, traversed in the positive

orientation. Then∫

C

ez

zdz = 2πi

The functionez

zhas only the singularity at 0 inside the contour. The

residue ofez

z=

1z

+∞

∑n=0

zn

(n+1)!at z = 0 is 1. Now apply the

preceding theorem.Note that direct computation gives the same result, because the

integral of∞

∑n=0

zn

(n+1)!over any closed contour is 0 and

∫C

1z

dz =∫ 2π

0

1eiθ ieiθ dθ = 2πi.

Bernd Schroder Louisiana Tech University, College of Engineering and Science

The Residue Theorem

Page 78: The Residue Theorem · 2010-10-04 · 5.We will prove the requisite theorem (the Residue Theorem) in this presentation and we will also lay the abstract groundwork. 6.We will then

logo1

Singularities Residues Residue Theorem Residue at Infinity Types of Isolated Singularities Residues at Poles

Example. Let C be the unit circle, traversed in the positive

orientation. Then∫

C

ez

zdz = 2πi

The functionez

zhas only the singularity at 0 inside the contour.

The

residue ofez

z=

1z

+∞

∑n=0

zn

(n+1)!at z = 0 is 1. Now apply the

preceding theorem.Note that direct computation gives the same result, because the

integral of∞

∑n=0

zn

(n+1)!over any closed contour is 0 and

∫C

1z

dz =∫ 2π

0

1eiθ ieiθ dθ = 2πi.

Bernd Schroder Louisiana Tech University, College of Engineering and Science

The Residue Theorem

Page 79: The Residue Theorem · 2010-10-04 · 5.We will prove the requisite theorem (the Residue Theorem) in this presentation and we will also lay the abstract groundwork. 6.We will then

logo1

Singularities Residues Residue Theorem Residue at Infinity Types of Isolated Singularities Residues at Poles

Example. Let C be the unit circle, traversed in the positive

orientation. Then∫

C

ez

zdz = 2πi

The functionez

zhas only the singularity at 0 inside the contour. The

residue ofez

z

=1z

+∞

∑n=0

zn

(n+1)!at z = 0 is 1. Now apply the

preceding theorem.Note that direct computation gives the same result, because the

integral of∞

∑n=0

zn

(n+1)!over any closed contour is 0 and

∫C

1z

dz =∫ 2π

0

1eiθ ieiθ dθ = 2πi.

Bernd Schroder Louisiana Tech University, College of Engineering and Science

The Residue Theorem

Page 80: The Residue Theorem · 2010-10-04 · 5.We will prove the requisite theorem (the Residue Theorem) in this presentation and we will also lay the abstract groundwork. 6.We will then

logo1

Singularities Residues Residue Theorem Residue at Infinity Types of Isolated Singularities Residues at Poles

Example. Let C be the unit circle, traversed in the positive

orientation. Then∫

C

ez

zdz = 2πi

The functionez

zhas only the singularity at 0 inside the contour. The

residue ofez

z=

1z

+∞

∑n=0

zn

(n+1)!

at z = 0 is 1. Now apply the

preceding theorem.Note that direct computation gives the same result, because the

integral of∞

∑n=0

zn

(n+1)!over any closed contour is 0 and

∫C

1z

dz =∫ 2π

0

1eiθ ieiθ dθ = 2πi.

Bernd Schroder Louisiana Tech University, College of Engineering and Science

The Residue Theorem

Page 81: The Residue Theorem · 2010-10-04 · 5.We will prove the requisite theorem (the Residue Theorem) in this presentation and we will also lay the abstract groundwork. 6.We will then

logo1

Singularities Residues Residue Theorem Residue at Infinity Types of Isolated Singularities Residues at Poles

Example. Let C be the unit circle, traversed in the positive

orientation. Then∫

C

ez

zdz = 2πi

The functionez

zhas only the singularity at 0 inside the contour. The

residue ofez

z=

1z

+∞

∑n=0

zn

(n+1)!at z = 0 is 1.

Now apply the

preceding theorem.Note that direct computation gives the same result, because the

integral of∞

∑n=0

zn

(n+1)!over any closed contour is 0 and

∫C

1z

dz =∫ 2π

0

1eiθ ieiθ dθ = 2πi.

Bernd Schroder Louisiana Tech University, College of Engineering and Science

The Residue Theorem

Page 82: The Residue Theorem · 2010-10-04 · 5.We will prove the requisite theorem (the Residue Theorem) in this presentation and we will also lay the abstract groundwork. 6.We will then

logo1

Singularities Residues Residue Theorem Residue at Infinity Types of Isolated Singularities Residues at Poles

Example. Let C be the unit circle, traversed in the positive

orientation. Then∫

C

ez

zdz = 2πi

The functionez

zhas only the singularity at 0 inside the contour. The

residue ofez

z=

1z

+∞

∑n=0

zn

(n+1)!at z = 0 is 1. Now apply the

preceding theorem.

Note that direct computation gives the same result, because the

integral of∞

∑n=0

zn

(n+1)!over any closed contour is 0 and

∫C

1z

dz =∫ 2π

0

1eiθ ieiθ dθ = 2πi.

Bernd Schroder Louisiana Tech University, College of Engineering and Science

The Residue Theorem

Page 83: The Residue Theorem · 2010-10-04 · 5.We will prove the requisite theorem (the Residue Theorem) in this presentation and we will also lay the abstract groundwork. 6.We will then

logo1

Singularities Residues Residue Theorem Residue at Infinity Types of Isolated Singularities Residues at Poles

Example. Let C be the unit circle, traversed in the positive

orientation. Then∫

C

ez

zdz = 2πi

The functionez

zhas only the singularity at 0 inside the contour. The

residue ofez

z=

1z

+∞

∑n=0

zn

(n+1)!at z = 0 is 1. Now apply the

preceding theorem.Note that direct computation gives the same result

, because the

integral of∞

∑n=0

zn

(n+1)!over any closed contour is 0 and

∫C

1z

dz =∫ 2π

0

1eiθ ieiθ dθ = 2πi.

Bernd Schroder Louisiana Tech University, College of Engineering and Science

The Residue Theorem

Page 84: The Residue Theorem · 2010-10-04 · 5.We will prove the requisite theorem (the Residue Theorem) in this presentation and we will also lay the abstract groundwork. 6.We will then

logo1

Singularities Residues Residue Theorem Residue at Infinity Types of Isolated Singularities Residues at Poles

Example. Let C be the unit circle, traversed in the positive

orientation. Then∫

C

ez

zdz = 2πi

The functionez

zhas only the singularity at 0 inside the contour. The

residue ofez

z=

1z

+∞

∑n=0

zn

(n+1)!at z = 0 is 1. Now apply the

preceding theorem.Note that direct computation gives the same result, because the

integral of∞

∑n=0

zn

(n+1)!over any closed contour is 0

and

∫C

1z

dz =∫ 2π

0

1eiθ ieiθ dθ = 2πi.

Bernd Schroder Louisiana Tech University, College of Engineering and Science

The Residue Theorem

Page 85: The Residue Theorem · 2010-10-04 · 5.We will prove the requisite theorem (the Residue Theorem) in this presentation and we will also lay the abstract groundwork. 6.We will then

logo1

Singularities Residues Residue Theorem Residue at Infinity Types of Isolated Singularities Residues at Poles

Example. Let C be the unit circle, traversed in the positive

orientation. Then∫

C

ez

zdz = 2πi

The functionez

zhas only the singularity at 0 inside the contour. The

residue ofez

z=

1z

+∞

∑n=0

zn

(n+1)!at z = 0 is 1. Now apply the

preceding theorem.Note that direct computation gives the same result, because the

integral of∞

∑n=0

zn

(n+1)!over any closed contour is 0 and

∫C

1z

dz

=∫ 2π

0

1eiθ ieiθ dθ = 2πi.

Bernd Schroder Louisiana Tech University, College of Engineering and Science

The Residue Theorem

Page 86: The Residue Theorem · 2010-10-04 · 5.We will prove the requisite theorem (the Residue Theorem) in this presentation and we will also lay the abstract groundwork. 6.We will then

logo1

Singularities Residues Residue Theorem Residue at Infinity Types of Isolated Singularities Residues at Poles

Example. Let C be the unit circle, traversed in the positive

orientation. Then∫

C

ez

zdz = 2πi

The functionez

zhas only the singularity at 0 inside the contour. The

residue ofez

z=

1z

+∞

∑n=0

zn

(n+1)!at z = 0 is 1. Now apply the

preceding theorem.Note that direct computation gives the same result, because the

integral of∞

∑n=0

zn

(n+1)!over any closed contour is 0 and

∫C

1z

dz =∫ 2π

0

1eiθ ieiθ dθ

= 2πi.

Bernd Schroder Louisiana Tech University, College of Engineering and Science

The Residue Theorem

Page 87: The Residue Theorem · 2010-10-04 · 5.We will prove the requisite theorem (the Residue Theorem) in this presentation and we will also lay the abstract groundwork. 6.We will then

logo1

Singularities Residues Residue Theorem Residue at Infinity Types of Isolated Singularities Residues at Poles

Example. Let C be the unit circle, traversed in the positive

orientation. Then∫

C

ez

zdz = 2πi

The functionez

zhas only the singularity at 0 inside the contour. The

residue ofez

z=

1z

+∞

∑n=0

zn

(n+1)!at z = 0 is 1. Now apply the

preceding theorem.Note that direct computation gives the same result, because the

integral of∞

∑n=0

zn

(n+1)!over any closed contour is 0 and

∫C

1z

dz =∫ 2π

0

1eiθ ieiθ dθ = 2πi.

Bernd Schroder Louisiana Tech University, College of Engineering and Science

The Residue Theorem

Page 88: The Residue Theorem · 2010-10-04 · 5.We will prove the requisite theorem (the Residue Theorem) in this presentation and we will also lay the abstract groundwork. 6.We will then

logo1

Singularities Residues Residue Theorem Residue at Infinity Types of Isolated Singularities Residues at Poles

Example.

Let C be the unit circle, traversed in the positive

orientation. Then∫

C

ez2

z2 dz = 0

The functionez2

z2 has only the singularity at 0 inside the contour. The

residue ofez2

z2 =1z2 +

∑n=0

z2n

(n+1)!at z = 0 is 0. Now apply the

preceding theorem.

Bernd Schroder Louisiana Tech University, College of Engineering and Science

The Residue Theorem

Page 89: The Residue Theorem · 2010-10-04 · 5.We will prove the requisite theorem (the Residue Theorem) in this presentation and we will also lay the abstract groundwork. 6.We will then

logo1

Singularities Residues Residue Theorem Residue at Infinity Types of Isolated Singularities Residues at Poles

Example. Let C be the unit circle, traversed in the positive

orientation.

Then∫

C

ez2

z2 dz = 0

The functionez2

z2 has only the singularity at 0 inside the contour. The

residue ofez2

z2 =1z2 +

∑n=0

z2n

(n+1)!at z = 0 is 0. Now apply the

preceding theorem.

Bernd Schroder Louisiana Tech University, College of Engineering and Science

The Residue Theorem

Page 90: The Residue Theorem · 2010-10-04 · 5.We will prove the requisite theorem (the Residue Theorem) in this presentation and we will also lay the abstract groundwork. 6.We will then

logo1

Singularities Residues Residue Theorem Residue at Infinity Types of Isolated Singularities Residues at Poles

Example. Let C be the unit circle, traversed in the positive

orientation. Then∫

C

ez2

z2 dz = 0

The functionez2

z2 has only the singularity at 0 inside the contour. The

residue ofez2

z2 =1z2 +

∑n=0

z2n

(n+1)!at z = 0 is 0. Now apply the

preceding theorem.

Bernd Schroder Louisiana Tech University, College of Engineering and Science

The Residue Theorem

Page 91: The Residue Theorem · 2010-10-04 · 5.We will prove the requisite theorem (the Residue Theorem) in this presentation and we will also lay the abstract groundwork. 6.We will then

logo1

Singularities Residues Residue Theorem Residue at Infinity Types of Isolated Singularities Residues at Poles

Example. Let C be the unit circle, traversed in the positive

orientation. Then∫

C

ez2

z2 dz = 0

The functionez2

z2 has only the singularity at 0 inside the contour.

The

residue ofez2

z2 =1z2 +

∑n=0

z2n

(n+1)!at z = 0 is 0. Now apply the

preceding theorem.

Bernd Schroder Louisiana Tech University, College of Engineering and Science

The Residue Theorem

Page 92: The Residue Theorem · 2010-10-04 · 5.We will prove the requisite theorem (the Residue Theorem) in this presentation and we will also lay the abstract groundwork. 6.We will then

logo1

Singularities Residues Residue Theorem Residue at Infinity Types of Isolated Singularities Residues at Poles

Example. Let C be the unit circle, traversed in the positive

orientation. Then∫

C

ez2

z2 dz = 0

The functionez2

z2 has only the singularity at 0 inside the contour. The

residue ofez2

z2

=1z2 +

∑n=0

z2n

(n+1)!at z = 0 is 0. Now apply the

preceding theorem.

Bernd Schroder Louisiana Tech University, College of Engineering and Science

The Residue Theorem

Page 93: The Residue Theorem · 2010-10-04 · 5.We will prove the requisite theorem (the Residue Theorem) in this presentation and we will also lay the abstract groundwork. 6.We will then

logo1

Singularities Residues Residue Theorem Residue at Infinity Types of Isolated Singularities Residues at Poles

Example. Let C be the unit circle, traversed in the positive

orientation. Then∫

C

ez2

z2 dz = 0

The functionez2

z2 has only the singularity at 0 inside the contour. The

residue ofez2

z2 =1z2 +

∑n=0

z2n

(n+1)!

at z = 0 is 0. Now apply the

preceding theorem.

Bernd Schroder Louisiana Tech University, College of Engineering and Science

The Residue Theorem

Page 94: The Residue Theorem · 2010-10-04 · 5.We will prove the requisite theorem (the Residue Theorem) in this presentation and we will also lay the abstract groundwork. 6.We will then

logo1

Singularities Residues Residue Theorem Residue at Infinity Types of Isolated Singularities Residues at Poles

Example. Let C be the unit circle, traversed in the positive

orientation. Then∫

C

ez2

z2 dz = 0

The functionez2

z2 has only the singularity at 0 inside the contour. The

residue ofez2

z2 =1z2 +

∑n=0

z2n

(n+1)!at z = 0 is 0.

Now apply the

preceding theorem.

Bernd Schroder Louisiana Tech University, College of Engineering and Science

The Residue Theorem

Page 95: The Residue Theorem · 2010-10-04 · 5.We will prove the requisite theorem (the Residue Theorem) in this presentation and we will also lay the abstract groundwork. 6.We will then

logo1

Singularities Residues Residue Theorem Residue at Infinity Types of Isolated Singularities Residues at Poles

Example. Let C be the unit circle, traversed in the positive

orientation. Then∫

C

ez2

z2 dz = 0

The functionez2

z2 has only the singularity at 0 inside the contour. The

residue ofez2

z2 =1z2 +

∑n=0

z2n

(n+1)!at z = 0 is 0. Now apply the

preceding theorem.

Bernd Schroder Louisiana Tech University, College of Engineering and Science

The Residue Theorem

Page 96: The Residue Theorem · 2010-10-04 · 5.We will prove the requisite theorem (the Residue Theorem) in this presentation and we will also lay the abstract groundwork. 6.We will then

logo1

Singularities Residues Residue Theorem Residue at Infinity Types of Isolated Singularities Residues at Poles

Theorem.

Residue Theorem. Let C be a simple closed positivelyoriented contour, let z1, . . . ,zn be points in the interior of C, and letthe function f be analytic on C and in its interior, except possibly atthe zj. Then

12πi

∫C

f (ξ ) dξ =n

∑j=1

Resz=zj(f ).

Proof. Let Cj be positively oriented circle around zj so that no two ofC1, . . . ,Cn intersect and so that all are contained in the interior of C.Then by extension of Cauchy-Goursat theorem∫

Cf (z) dz =

n

∑j=1

∫Cj

f (z) dz = 2πin

∑j=1

Resz=zj(f ).

Bernd Schroder Louisiana Tech University, College of Engineering and Science

The Residue Theorem

Page 97: The Residue Theorem · 2010-10-04 · 5.We will prove the requisite theorem (the Residue Theorem) in this presentation and we will also lay the abstract groundwork. 6.We will then

logo1

Singularities Residues Residue Theorem Residue at Infinity Types of Isolated Singularities Residues at Poles

Theorem. Residue Theorem. Let C be a simple closed positivelyoriented contour, let z1, . . . ,zn be points in the interior of C, and letthe function f be analytic on C and in its interior, except possibly atthe zj. Then

12πi

∫C

f (ξ ) dξ =n

∑j=1

Resz=zj(f ).

Proof. Let Cj be positively oriented circle around zj so that no two ofC1, . . . ,Cn intersect and so that all are contained in the interior of C.Then by extension of Cauchy-Goursat theorem∫

Cf (z) dz =

n

∑j=1

∫Cj

f (z) dz = 2πin

∑j=1

Resz=zj(f ).

Bernd Schroder Louisiana Tech University, College of Engineering and Science

The Residue Theorem

Page 98: The Residue Theorem · 2010-10-04 · 5.We will prove the requisite theorem (the Residue Theorem) in this presentation and we will also lay the abstract groundwork. 6.We will then

logo1

Singularities Residues Residue Theorem Residue at Infinity Types of Isolated Singularities Residues at Poles

Theorem. Residue Theorem. Let C be a simple closed positivelyoriented contour, let z1, . . . ,zn be points in the interior of C, and letthe function f be analytic on C and in its interior, except possibly atthe zj. Then

12πi

∫C

f (ξ ) dξ =n

∑j=1

Resz=zj(f ).

Proof.

Let Cj be positively oriented circle around zj so that no two ofC1, . . . ,Cn intersect and so that all are contained in the interior of C.Then by extension of Cauchy-Goursat theorem∫

Cf (z) dz =

n

∑j=1

∫Cj

f (z) dz = 2πin

∑j=1

Resz=zj(f ).

Bernd Schroder Louisiana Tech University, College of Engineering and Science

The Residue Theorem

Page 99: The Residue Theorem · 2010-10-04 · 5.We will prove the requisite theorem (the Residue Theorem) in this presentation and we will also lay the abstract groundwork. 6.We will then

logo1

Singularities Residues Residue Theorem Residue at Infinity Types of Isolated Singularities Residues at Poles

Theorem. Residue Theorem. Let C be a simple closed positivelyoriented contour, let z1, . . . ,zn be points in the interior of C, and letthe function f be analytic on C and in its interior, except possibly atthe zj. Then

12πi

∫C

f (ξ ) dξ =n

∑j=1

Resz=zj(f ).

Proof. Let Cj be positively oriented circle around zj

so that no two ofC1, . . . ,Cn intersect and so that all are contained in the interior of C.Then by extension of Cauchy-Goursat theorem∫

Cf (z) dz =

n

∑j=1

∫Cj

f (z) dz = 2πin

∑j=1

Resz=zj(f ).

Bernd Schroder Louisiana Tech University, College of Engineering and Science

The Residue Theorem

Page 100: The Residue Theorem · 2010-10-04 · 5.We will prove the requisite theorem (the Residue Theorem) in this presentation and we will also lay the abstract groundwork. 6.We will then

logo1

Singularities Residues Residue Theorem Residue at Infinity Types of Isolated Singularities Residues at Poles

Theorem. Residue Theorem. Let C be a simple closed positivelyoriented contour, let z1, . . . ,zn be points in the interior of C, and letthe function f be analytic on C and in its interior, except possibly atthe zj. Then

12πi

∫C

f (ξ ) dξ =n

∑j=1

Resz=zj(f ).

Proof. Let Cj be positively oriented circle around zj so that no two ofC1, . . . ,Cn intersect and so that all are contained in the interior of C.

Then by extension of Cauchy-Goursat theorem∫C

f (z) dz =n

∑j=1

∫Cj

f (z) dz = 2πin

∑j=1

Resz=zj(f ).

Bernd Schroder Louisiana Tech University, College of Engineering and Science

The Residue Theorem

Page 101: The Residue Theorem · 2010-10-04 · 5.We will prove the requisite theorem (the Residue Theorem) in this presentation and we will also lay the abstract groundwork. 6.We will then

logo1

Singularities Residues Residue Theorem Residue at Infinity Types of Isolated Singularities Residues at Poles

Theorem. Residue Theorem. Let C be a simple closed positivelyoriented contour, let z1, . . . ,zn be points in the interior of C, and letthe function f be analytic on C and in its interior, except possibly atthe zj. Then

12πi

∫C

f (ξ ) dξ =n

∑j=1

Resz=zj(f ).

Proof. Let Cj be positively oriented circle around zj so that no two ofC1, . . . ,Cn intersect and so that all are contained in the interior of C.Then by extension of Cauchy-Goursat theorem∫

Cf (z) dz =

n

∑j=1

∫Cj

f (z) dz

= 2πin

∑j=1

Resz=zj(f ).

Bernd Schroder Louisiana Tech University, College of Engineering and Science

The Residue Theorem

Page 102: The Residue Theorem · 2010-10-04 · 5.We will prove the requisite theorem (the Residue Theorem) in this presentation and we will also lay the abstract groundwork. 6.We will then

logo1

Singularities Residues Residue Theorem Residue at Infinity Types of Isolated Singularities Residues at Poles

Theorem. Residue Theorem. Let C be a simple closed positivelyoriented contour, let z1, . . . ,zn be points in the interior of C, and letthe function f be analytic on C and in its interior, except possibly atthe zj. Then

12πi

∫C

f (ξ ) dξ =n

∑j=1

Resz=zj(f ).

Proof. Let Cj be positively oriented circle around zj so that no two ofC1, . . . ,Cn intersect and so that all are contained in the interior of C.Then by extension of Cauchy-Goursat theorem∫

Cf (z) dz =

n

∑j=1

∫Cj

f (z) dz = 2πin

∑j=1

Resz=zj(f ).

Bernd Schroder Louisiana Tech University, College of Engineering and Science

The Residue Theorem

Page 103: The Residue Theorem · 2010-10-04 · 5.We will prove the requisite theorem (the Residue Theorem) in this presentation and we will also lay the abstract groundwork. 6.We will then

logo1

Singularities Residues Residue Theorem Residue at Infinity Types of Isolated Singularities Residues at Poles

Theorem. Residue Theorem. Let C be a simple closed positivelyoriented contour, let z1, . . . ,zn be points in the interior of C, and letthe function f be analytic on C and in its interior, except possibly atthe zj. Then

12πi

∫C

f (ξ ) dξ =n

∑j=1

Resz=zj(f ).

Proof. Let Cj be positively oriented circle around zj so that no two ofC1, . . . ,Cn intersect and so that all are contained in the interior of C.Then by extension of Cauchy-Goursat theorem∫

Cf (z) dz =

n

∑j=1

∫Cj

f (z) dz = 2πin

∑j=1

Resz=zj(f ).

Bernd Schroder Louisiana Tech University, College of Engineering and Science

The Residue Theorem

Page 104: The Residue Theorem · 2010-10-04 · 5.We will prove the requisite theorem (the Residue Theorem) in this presentation and we will also lay the abstract groundwork. 6.We will then

logo1

Singularities Residues Residue Theorem Residue at Infinity Types of Isolated Singularities Residues at Poles

Example.

Integrate the function f (z) =z2

z2 +1over the positively

oriented boundary of the square of numbers z = x+ iy with (x,y) in[−3,3]× [−3,3].

-

6ℑ(z)

ℜ(z)6

?

-

3

ri

r−i

Bernd Schroder Louisiana Tech University, College of Engineering and Science

The Residue Theorem

Page 105: The Residue Theorem · 2010-10-04 · 5.We will prove the requisite theorem (the Residue Theorem) in this presentation and we will also lay the abstract groundwork. 6.We will then

logo1

Singularities Residues Residue Theorem Residue at Infinity Types of Isolated Singularities Residues at Poles

Example. Integrate the function f (z) =z2

z2 +1over the positively

oriented boundary of the square of numbers z = x+ iy with (x,y) in[−3,3]× [−3,3].

-

6ℑ(z)

ℜ(z)6

?

-

3

ri

r−i

Bernd Schroder Louisiana Tech University, College of Engineering and Science

The Residue Theorem

Page 106: The Residue Theorem · 2010-10-04 · 5.We will prove the requisite theorem (the Residue Theorem) in this presentation and we will also lay the abstract groundwork. 6.We will then

logo1

Singularities Residues Residue Theorem Residue at Infinity Types of Isolated Singularities Residues at Poles

Example. Integrate the function f (z) =z2

z2 +1over the positively

oriented boundary of the square of numbers z = x+ iy with (x,y) in[−3,3]× [−3,3].

-

6ℑ(z)

ℜ(z)

6

?

-

3

ri

r−i

Bernd Schroder Louisiana Tech University, College of Engineering and Science

The Residue Theorem

Page 107: The Residue Theorem · 2010-10-04 · 5.We will prove the requisite theorem (the Residue Theorem) in this presentation and we will also lay the abstract groundwork. 6.We will then

logo1

Singularities Residues Residue Theorem Residue at Infinity Types of Isolated Singularities Residues at Poles

Example. Integrate the function f (z) =z2

z2 +1over the positively

oriented boundary of the square of numbers z = x+ iy with (x,y) in[−3,3]× [−3,3].

-

6ℑ(z)

ℜ(z)6

?

-

3

ri

r−i

Bernd Schroder Louisiana Tech University, College of Engineering and Science

The Residue Theorem

Page 108: The Residue Theorem · 2010-10-04 · 5.We will prove the requisite theorem (the Residue Theorem) in this presentation and we will also lay the abstract groundwork. 6.We will then

logo1

Singularities Residues Residue Theorem Residue at Infinity Types of Isolated Singularities Residues at Poles

Example. Integrate the function f (z) =z2

z2 +1over the positively

oriented boundary of the square of numbers z = x+ iy with (x,y) in[−3,3]× [−3,3].

-

6ℑ(z)

ℜ(z)6

?

-

3

ri

r−i

Bernd Schroder Louisiana Tech University, College of Engineering and Science

The Residue Theorem

Page 109: The Residue Theorem · 2010-10-04 · 5.We will prove the requisite theorem (the Residue Theorem) in this presentation and we will also lay the abstract groundwork. 6.We will then

logo1

Singularities Residues Residue Theorem Residue at Infinity Types of Isolated Singularities Residues at Poles

Example. Integrate the function f (z) =z2

z2 +1over the positively

oriented boundary of the square of numbers z = x+ iy with (x,y) in[−3,3]× [−3,3].

-

6ℑ(z)

ℜ(z)6

?

-

3

ri

r−i

Bernd Schroder Louisiana Tech University, College of Engineering and Science

The Residue Theorem

Page 110: The Residue Theorem · 2010-10-04 · 5.We will prove the requisite theorem (the Residue Theorem) in this presentation and we will also lay the abstract groundwork. 6.We will then

logo1

Singularities Residues Residue Theorem Residue at Infinity Types of Isolated Singularities Residues at Poles

Example. Integrate the function f (z) =z2

z2 +1over the positively

oriented boundary of the square of numbers z = x+ iy with (x,y) in[−3,3]× [−3,3].

-

6ℑ(z)

ℜ(z)6

?

-

3

ri

r−i

Bernd Schroder Louisiana Tech University, College of Engineering and Science

The Residue Theorem

Page 111: The Residue Theorem · 2010-10-04 · 5.We will prove the requisite theorem (the Residue Theorem) in this presentation and we will also lay the abstract groundwork. 6.We will then

logo1

Singularities Residues Residue Theorem Residue at Infinity Types of Isolated Singularities Residues at Poles

Example. Integrate the function f (z) =z2

z2 +1over the positively

oriented boundary of the square of numbers z = x+ iy with (x,y) in[−3,3]× [−3,3].

-

6ℑ(z)

ℜ(z)6

?

-

3

ri

r−i

Bernd Schroder Louisiana Tech University, College of Engineering and Science

The Residue Theorem

Page 112: The Residue Theorem · 2010-10-04 · 5.We will prove the requisite theorem (the Residue Theorem) in this presentation and we will also lay the abstract groundwork. 6.We will then

logo1

Singularities Residues Residue Theorem Residue at Infinity Types of Isolated Singularities Residues at Poles

Example. Integrate the function f (z) =z2

z2 +1over the positively

oriented boundary of the square of numbers z = x+ iy with (x,y) in[−3,3]× [−3,3].

-

6ℑ(z)

ℜ(z)6

?

-

3

r

i

r−i

Bernd Schroder Louisiana Tech University, College of Engineering and Science

The Residue Theorem

Page 113: The Residue Theorem · 2010-10-04 · 5.We will prove the requisite theorem (the Residue Theorem) in this presentation and we will also lay the abstract groundwork. 6.We will then

logo1

Singularities Residues Residue Theorem Residue at Infinity Types of Isolated Singularities Residues at Poles

Example. Integrate the function f (z) =z2

z2 +1over the positively

oriented boundary of the square of numbers z = x+ iy with (x,y) in[−3,3]× [−3,3].

-

6ℑ(z)

ℜ(z)6

?

-

3

ri

r−i

Bernd Schroder Louisiana Tech University, College of Engineering and Science

The Residue Theorem

Page 114: The Residue Theorem · 2010-10-04 · 5.We will prove the requisite theorem (the Residue Theorem) in this presentation and we will also lay the abstract groundwork. 6.We will then

logo1

Singularities Residues Residue Theorem Residue at Infinity Types of Isolated Singularities Residues at Poles

Example. Integrate the function f (z) =z2

z2 +1over the positively

oriented boundary of the square of numbers z = x+ iy with (x,y) in[−3,3]× [−3,3].

-

6ℑ(z)

ℜ(z)6

?

-

3

ri

r

−i

Bernd Schroder Louisiana Tech University, College of Engineering and Science

The Residue Theorem

Page 115: The Residue Theorem · 2010-10-04 · 5.We will prove the requisite theorem (the Residue Theorem) in this presentation and we will also lay the abstract groundwork. 6.We will then

logo1

Singularities Residues Residue Theorem Residue at Infinity Types of Isolated Singularities Residues at Poles

Example. Integrate the function f (z) =z2

z2 +1over the positively

oriented boundary of the square of numbers z = x+ iy with (x,y) in[−3,3]× [−3,3].

-

6ℑ(z)

ℜ(z)6

?

-

3

ri

r−i

Bernd Schroder Louisiana Tech University, College of Engineering and Science

The Residue Theorem

Page 116: The Residue Theorem · 2010-10-04 · 5.We will prove the requisite theorem (the Residue Theorem) in this presentation and we will also lay the abstract groundwork. 6.We will then

logo1

Singularities Residues Residue Theorem Residue at Infinity Types of Isolated Singularities Residues at Poles

Example. Integrate the function f (z) =z2

z2 +1over the positively

oriented boundary of the square of numbers z = x+ iy with (x,y) in[−3,3]× [−3,3].

Residue at i:z2

z2 +1=

z2 +1−1z2 +1

= 1− 1z2 +1

= 1− 1z+ i

1z− i

Resz=i

(z2

z2 +1

)= − 1

i+ i=

i2

Residue at −i:z2

z2 +1=

z2 +1−1z2 +1

= 1− 1z2 +1

= 1− 1z− i

1z+ i

Resz=−i

(z2

z2 +1

)= − 1

−i− i=− i

2

Bernd Schroder Louisiana Tech University, College of Engineering and Science

The Residue Theorem

Page 117: The Residue Theorem · 2010-10-04 · 5.We will prove the requisite theorem (the Residue Theorem) in this presentation and we will also lay the abstract groundwork. 6.We will then

logo1

Singularities Residues Residue Theorem Residue at Infinity Types of Isolated Singularities Residues at Poles

Example. Integrate the function f (z) =z2

z2 +1over the positively

oriented boundary of the square of numbers z = x+ iy with (x,y) in[−3,3]× [−3,3].

Residue at i:

z2

z2 +1=

z2 +1−1z2 +1

= 1− 1z2 +1

= 1− 1z+ i

1z− i

Resz=i

(z2

z2 +1

)= − 1

i+ i=

i2

Residue at −i:z2

z2 +1=

z2 +1−1z2 +1

= 1− 1z2 +1

= 1− 1z− i

1z+ i

Resz=−i

(z2

z2 +1

)= − 1

−i− i=− i

2

Bernd Schroder Louisiana Tech University, College of Engineering and Science

The Residue Theorem

Page 118: The Residue Theorem · 2010-10-04 · 5.We will prove the requisite theorem (the Residue Theorem) in this presentation and we will also lay the abstract groundwork. 6.We will then

logo1

Singularities Residues Residue Theorem Residue at Infinity Types of Isolated Singularities Residues at Poles

Example. Integrate the function f (z) =z2

z2 +1over the positively

oriented boundary of the square of numbers z = x+ iy with (x,y) in[−3,3]× [−3,3].

Residue at i:z2

z2 +1

=z2 +1−1

z2 +1= 1− 1

z2 +1= 1− 1

z+ i1

z− i

Resz=i

(z2

z2 +1

)= − 1

i+ i=

i2

Residue at −i:z2

z2 +1=

z2 +1−1z2 +1

= 1− 1z2 +1

= 1− 1z− i

1z+ i

Resz=−i

(z2

z2 +1

)= − 1

−i− i=− i

2

Bernd Schroder Louisiana Tech University, College of Engineering and Science

The Residue Theorem

Page 119: The Residue Theorem · 2010-10-04 · 5.We will prove the requisite theorem (the Residue Theorem) in this presentation and we will also lay the abstract groundwork. 6.We will then

logo1

Singularities Residues Residue Theorem Residue at Infinity Types of Isolated Singularities Residues at Poles

Example. Integrate the function f (z) =z2

z2 +1over the positively

oriented boundary of the square of numbers z = x+ iy with (x,y) in[−3,3]× [−3,3].

Residue at i:z2

z2 +1=

z2 +1−1z2 +1

= 1− 1z2 +1

= 1− 1z+ i

1z− i

Resz=i

(z2

z2 +1

)= − 1

i+ i=

i2

Residue at −i:z2

z2 +1=

z2 +1−1z2 +1

= 1− 1z2 +1

= 1− 1z− i

1z+ i

Resz=−i

(z2

z2 +1

)= − 1

−i− i=− i

2

Bernd Schroder Louisiana Tech University, College of Engineering and Science

The Residue Theorem

Page 120: The Residue Theorem · 2010-10-04 · 5.We will prove the requisite theorem (the Residue Theorem) in this presentation and we will also lay the abstract groundwork. 6.We will then

logo1

Singularities Residues Residue Theorem Residue at Infinity Types of Isolated Singularities Residues at Poles

Example. Integrate the function f (z) =z2

z2 +1over the positively

oriented boundary of the square of numbers z = x+ iy with (x,y) in[−3,3]× [−3,3].

Residue at i:z2

z2 +1=

z2 +1−1z2 +1

= 1− 1z2 +1

= 1− 1z+ i

1z− i

Resz=i

(z2

z2 +1

)= − 1

i+ i=

i2

Residue at −i:z2

z2 +1=

z2 +1−1z2 +1

= 1− 1z2 +1

= 1− 1z− i

1z+ i

Resz=−i

(z2

z2 +1

)= − 1

−i− i=− i

2

Bernd Schroder Louisiana Tech University, College of Engineering and Science

The Residue Theorem

Page 121: The Residue Theorem · 2010-10-04 · 5.We will prove the requisite theorem (the Residue Theorem) in this presentation and we will also lay the abstract groundwork. 6.We will then

logo1

Singularities Residues Residue Theorem Residue at Infinity Types of Isolated Singularities Residues at Poles

Example. Integrate the function f (z) =z2

z2 +1over the positively

oriented boundary of the square of numbers z = x+ iy with (x,y) in[−3,3]× [−3,3].

Residue at i:z2

z2 +1=

z2 +1−1z2 +1

= 1− 1z2 +1

= 1− 1z+ i

1z− i

Resz=i

(z2

z2 +1

)= − 1

i+ i=

i2

Residue at −i:z2

z2 +1=

z2 +1−1z2 +1

= 1− 1z2 +1

= 1− 1z− i

1z+ i

Resz=−i

(z2

z2 +1

)= − 1

−i− i=− i

2

Bernd Schroder Louisiana Tech University, College of Engineering and Science

The Residue Theorem

Page 122: The Residue Theorem · 2010-10-04 · 5.We will prove the requisite theorem (the Residue Theorem) in this presentation and we will also lay the abstract groundwork. 6.We will then

logo1

Singularities Residues Residue Theorem Residue at Infinity Types of Isolated Singularities Residues at Poles

Example. Integrate the function f (z) =z2

z2 +1over the positively

oriented boundary of the square of numbers z = x+ iy with (x,y) in[−3,3]× [−3,3].

Residue at i:z2

z2 +1=

z2 +1−1z2 +1

= 1− 1z2 +1

= 1− 1z+ i

1z− i

Resz=i

(z2

z2 +1

)

= − 1i+ i

=i2

Residue at −i:z2

z2 +1=

z2 +1−1z2 +1

= 1− 1z2 +1

= 1− 1z− i

1z+ i

Resz=−i

(z2

z2 +1

)= − 1

−i− i=− i

2

Bernd Schroder Louisiana Tech University, College of Engineering and Science

The Residue Theorem

Page 123: The Residue Theorem · 2010-10-04 · 5.We will prove the requisite theorem (the Residue Theorem) in this presentation and we will also lay the abstract groundwork. 6.We will then

logo1

Singularities Residues Residue Theorem Residue at Infinity Types of Isolated Singularities Residues at Poles

Example. Integrate the function f (z) =z2

z2 +1over the positively

oriented boundary of the square of numbers z = x+ iy with (x,y) in[−3,3]× [−3,3].

Residue at i:z2

z2 +1=

z2 +1−1z2 +1

= 1− 1z2 +1

= 1− 1z+ i

1z− i

Resz=i

(z2

z2 +1

)= − 1

i+ i

=i2

Residue at −i:z2

z2 +1=

z2 +1−1z2 +1

= 1− 1z2 +1

= 1− 1z− i

1z+ i

Resz=−i

(z2

z2 +1

)= − 1

−i− i=− i

2

Bernd Schroder Louisiana Tech University, College of Engineering and Science

The Residue Theorem

Page 124: The Residue Theorem · 2010-10-04 · 5.We will prove the requisite theorem (the Residue Theorem) in this presentation and we will also lay the abstract groundwork. 6.We will then

logo1

Singularities Residues Residue Theorem Residue at Infinity Types of Isolated Singularities Residues at Poles

Example. Integrate the function f (z) =z2

z2 +1over the positively

oriented boundary of the square of numbers z = x+ iy with (x,y) in[−3,3]× [−3,3].

Residue at i:z2

z2 +1=

z2 +1−1z2 +1

= 1− 1z2 +1

= 1− 1z+ i

1z− i

Resz=i

(z2

z2 +1

)= − 1

i+ i=

i2

Residue at −i:z2

z2 +1=

z2 +1−1z2 +1

= 1− 1z2 +1

= 1− 1z− i

1z+ i

Resz=−i

(z2

z2 +1

)= − 1

−i− i=− i

2

Bernd Schroder Louisiana Tech University, College of Engineering and Science

The Residue Theorem

Page 125: The Residue Theorem · 2010-10-04 · 5.We will prove the requisite theorem (the Residue Theorem) in this presentation and we will also lay the abstract groundwork. 6.We will then

logo1

Singularities Residues Residue Theorem Residue at Infinity Types of Isolated Singularities Residues at Poles

Example. Integrate the function f (z) =z2

z2 +1over the positively

oriented boundary of the square of numbers z = x+ iy with (x,y) in[−3,3]× [−3,3].

Residue at i:z2

z2 +1=

z2 +1−1z2 +1

= 1− 1z2 +1

= 1− 1z+ i

1z− i

Resz=i

(z2

z2 +1

)= − 1

i+ i=

i2

Residue at −i:

z2

z2 +1=

z2 +1−1z2 +1

= 1− 1z2 +1

= 1− 1z− i

1z+ i

Resz=−i

(z2

z2 +1

)= − 1

−i− i=− i

2

Bernd Schroder Louisiana Tech University, College of Engineering and Science

The Residue Theorem

Page 126: The Residue Theorem · 2010-10-04 · 5.We will prove the requisite theorem (the Residue Theorem) in this presentation and we will also lay the abstract groundwork. 6.We will then

logo1

Singularities Residues Residue Theorem Residue at Infinity Types of Isolated Singularities Residues at Poles

Example. Integrate the function f (z) =z2

z2 +1over the positively

oriented boundary of the square of numbers z = x+ iy with (x,y) in[−3,3]× [−3,3].

Residue at i:z2

z2 +1=

z2 +1−1z2 +1

= 1− 1z2 +1

= 1− 1z+ i

1z− i

Resz=i

(z2

z2 +1

)= − 1

i+ i=

i2

Residue at −i:z2

z2 +1

=z2 +1−1

z2 +1= 1− 1

z2 +1= 1− 1

z− i1

z+ i

Resz=−i

(z2

z2 +1

)= − 1

−i− i=− i

2

Bernd Schroder Louisiana Tech University, College of Engineering and Science

The Residue Theorem

Page 127: The Residue Theorem · 2010-10-04 · 5.We will prove the requisite theorem (the Residue Theorem) in this presentation and we will also lay the abstract groundwork. 6.We will then

logo1

Singularities Residues Residue Theorem Residue at Infinity Types of Isolated Singularities Residues at Poles

Example. Integrate the function f (z) =z2

z2 +1over the positively

oriented boundary of the square of numbers z = x+ iy with (x,y) in[−3,3]× [−3,3].

Residue at i:z2

z2 +1=

z2 +1−1z2 +1

= 1− 1z2 +1

= 1− 1z+ i

1z− i

Resz=i

(z2

z2 +1

)= − 1

i+ i=

i2

Residue at −i:z2

z2 +1=

z2 +1−1z2 +1

= 1− 1z2 +1

= 1− 1z− i

1z+ i

Resz=−i

(z2

z2 +1

)= − 1

−i− i=− i

2

Bernd Schroder Louisiana Tech University, College of Engineering and Science

The Residue Theorem

Page 128: The Residue Theorem · 2010-10-04 · 5.We will prove the requisite theorem (the Residue Theorem) in this presentation and we will also lay the abstract groundwork. 6.We will then

logo1

Singularities Residues Residue Theorem Residue at Infinity Types of Isolated Singularities Residues at Poles

Example. Integrate the function f (z) =z2

z2 +1over the positively

oriented boundary of the square of numbers z = x+ iy with (x,y) in[−3,3]× [−3,3].

Residue at i:z2

z2 +1=

z2 +1−1z2 +1

= 1− 1z2 +1

= 1− 1z+ i

1z− i

Resz=i

(z2

z2 +1

)= − 1

i+ i=

i2

Residue at −i:z2

z2 +1=

z2 +1−1z2 +1

= 1− 1z2 +1

= 1− 1z− i

1z+ i

Resz=−i

(z2

z2 +1

)= − 1

−i− i=− i

2

Bernd Schroder Louisiana Tech University, College of Engineering and Science

The Residue Theorem

Page 129: The Residue Theorem · 2010-10-04 · 5.We will prove the requisite theorem (the Residue Theorem) in this presentation and we will also lay the abstract groundwork. 6.We will then

logo1

Singularities Residues Residue Theorem Residue at Infinity Types of Isolated Singularities Residues at Poles

Example. Integrate the function f (z) =z2

z2 +1over the positively

oriented boundary of the square of numbers z = x+ iy with (x,y) in[−3,3]× [−3,3].

Residue at i:z2

z2 +1=

z2 +1−1z2 +1

= 1− 1z2 +1

= 1− 1z+ i

1z− i

Resz=i

(z2

z2 +1

)= − 1

i+ i=

i2

Residue at −i:z2

z2 +1=

z2 +1−1z2 +1

= 1− 1z2 +1

= 1− 1z− i

1z+ i

Resz=−i

(z2

z2 +1

)= − 1

−i− i=− i

2

Bernd Schroder Louisiana Tech University, College of Engineering and Science

The Residue Theorem

Page 130: The Residue Theorem · 2010-10-04 · 5.We will prove the requisite theorem (the Residue Theorem) in this presentation and we will also lay the abstract groundwork. 6.We will then

logo1

Singularities Residues Residue Theorem Residue at Infinity Types of Isolated Singularities Residues at Poles

Example. Integrate the function f (z) =z2

z2 +1over the positively

oriented boundary of the square of numbers z = x+ iy with (x,y) in[−3,3]× [−3,3].

Residue at i:z2

z2 +1=

z2 +1−1z2 +1

= 1− 1z2 +1

= 1− 1z+ i

1z− i

Resz=i

(z2

z2 +1

)= − 1

i+ i=

i2

Residue at −i:z2

z2 +1=

z2 +1−1z2 +1

= 1− 1z2 +1

= 1− 1z− i

1z+ i

Resz=−i

(z2

z2 +1

)

= − 1−i− i

=− i2

Bernd Schroder Louisiana Tech University, College of Engineering and Science

The Residue Theorem

Page 131: The Residue Theorem · 2010-10-04 · 5.We will prove the requisite theorem (the Residue Theorem) in this presentation and we will also lay the abstract groundwork. 6.We will then

logo1

Singularities Residues Residue Theorem Residue at Infinity Types of Isolated Singularities Residues at Poles

Example. Integrate the function f (z) =z2

z2 +1over the positively

oriented boundary of the square of numbers z = x+ iy with (x,y) in[−3,3]× [−3,3].

Residue at i:z2

z2 +1=

z2 +1−1z2 +1

= 1− 1z2 +1

= 1− 1z+ i

1z− i

Resz=i

(z2

z2 +1

)= − 1

i+ i=

i2

Residue at −i:z2

z2 +1=

z2 +1−1z2 +1

= 1− 1z2 +1

= 1− 1z− i

1z+ i

Resz=−i

(z2

z2 +1

)= − 1

−i− i

=− i2

Bernd Schroder Louisiana Tech University, College of Engineering and Science

The Residue Theorem

Page 132: The Residue Theorem · 2010-10-04 · 5.We will prove the requisite theorem (the Residue Theorem) in this presentation and we will also lay the abstract groundwork. 6.We will then

logo1

Singularities Residues Residue Theorem Residue at Infinity Types of Isolated Singularities Residues at Poles

Example. Integrate the function f (z) =z2

z2 +1over the positively

oriented boundary of the square of numbers z = x+ iy with (x,y) in[−3,3]× [−3,3].

Residue at i:z2

z2 +1=

z2 +1−1z2 +1

= 1− 1z2 +1

= 1− 1z+ i

1z− i

Resz=i

(z2

z2 +1

)= − 1

i+ i=

i2

Residue at −i:z2

z2 +1=

z2 +1−1z2 +1

= 1− 1z2 +1

= 1− 1z− i

1z+ i

Resz=−i

(z2

z2 +1

)= − 1

−i− i=− i

2

Bernd Schroder Louisiana Tech University, College of Engineering and Science

The Residue Theorem

Page 133: The Residue Theorem · 2010-10-04 · 5.We will prove the requisite theorem (the Residue Theorem) in this presentation and we will also lay the abstract groundwork. 6.We will then

logo1

Singularities Residues Residue Theorem Residue at Infinity Types of Isolated Singularities Residues at Poles

Example. Integrate the function f (z) =z2

z2 +1over the positively

oriented boundary of the square of numbers z = x+ iy with (x,y) in[−3,3]× [−3,3].

∫C

z2

z2 +1dz = 2πi

[Resz=i

(z2

z2 +1

)+Resz=−i

(z2

z2 +1

)]= 2πi

[i2

+(− i

2

)]= 0

Bernd Schroder Louisiana Tech University, College of Engineering and Science

The Residue Theorem

Page 134: The Residue Theorem · 2010-10-04 · 5.We will prove the requisite theorem (the Residue Theorem) in this presentation and we will also lay the abstract groundwork. 6.We will then

logo1

Singularities Residues Residue Theorem Residue at Infinity Types of Isolated Singularities Residues at Poles

Example. Integrate the function f (z) =z2

z2 +1over the positively

oriented boundary of the square of numbers z = x+ iy with (x,y) in[−3,3]× [−3,3].

∫C

z2

z2 +1dz

= 2πi[

Resz=i

(z2

z2 +1

)+Resz=−i

(z2

z2 +1

)]= 2πi

[i2

+(− i

2

)]= 0

Bernd Schroder Louisiana Tech University, College of Engineering and Science

The Residue Theorem

Page 135: The Residue Theorem · 2010-10-04 · 5.We will prove the requisite theorem (the Residue Theorem) in this presentation and we will also lay the abstract groundwork. 6.We will then

logo1

Singularities Residues Residue Theorem Residue at Infinity Types of Isolated Singularities Residues at Poles

Example. Integrate the function f (z) =z2

z2 +1over the positively

oriented boundary of the square of numbers z = x+ iy with (x,y) in[−3,3]× [−3,3].

∫C

z2

z2 +1dz = 2πi

[Resz=i

(z2

z2 +1

)+Resz=−i

(z2

z2 +1

)]

= 2πi[

i2

+(− i

2

)]= 0

Bernd Schroder Louisiana Tech University, College of Engineering and Science

The Residue Theorem

Page 136: The Residue Theorem · 2010-10-04 · 5.We will prove the requisite theorem (the Residue Theorem) in this presentation and we will also lay the abstract groundwork. 6.We will then

logo1

Singularities Residues Residue Theorem Residue at Infinity Types of Isolated Singularities Residues at Poles

Example. Integrate the function f (z) =z2

z2 +1over the positively

oriented boundary of the square of numbers z = x+ iy with (x,y) in[−3,3]× [−3,3].

∫C

z2

z2 +1dz = 2πi

[Resz=i

(z2

z2 +1

)+Resz=−i

(z2

z2 +1

)]= 2πi

[i2

+(− i

2

)]

= 0

Bernd Schroder Louisiana Tech University, College of Engineering and Science

The Residue Theorem

Page 137: The Residue Theorem · 2010-10-04 · 5.We will prove the requisite theorem (the Residue Theorem) in this presentation and we will also lay the abstract groundwork. 6.We will then

logo1

Singularities Residues Residue Theorem Residue at Infinity Types of Isolated Singularities Residues at Poles

Example. Integrate the function f (z) =z2

z2 +1over the positively

oriented boundary of the square of numbers z = x+ iy with (x,y) in[−3,3]× [−3,3].

∫C

z2

z2 +1dz = 2πi

[Resz=i

(z2

z2 +1

)+Resz=−i

(z2

z2 +1

)]= 2πi

[i2

+(− i

2

)]= 0

Bernd Schroder Louisiana Tech University, College of Engineering and Science

The Residue Theorem

Page 138: The Residue Theorem · 2010-10-04 · 5.We will prove the requisite theorem (the Residue Theorem) in this presentation and we will also lay the abstract groundwork. 6.We will then

logo1

Singularities Residues Residue Theorem Residue at Infinity Types of Isolated Singularities Residues at Poles

Definition.

Let f be analytic on C except for a finite number ofsingular points z1, . . . ,zn. Assume that R1 is so that all |zj| ≤ R1. ForR0 > R1 let C−0 be the circle around the origin of radius R0, traversedin the clockwise, that is, the mathematically negative direction. Thenwe define the residue at infinity of f as

Resz=∞(f ) =1

2πi

∫C−0

f (z) dz.

Resz=∞(f ) =1

2πi

∫C−0

f (z) dz =− 12πi

∫C+

0

f (z) dz =− 12πi

∫C+

0

∑n=−∞

cnzn dz

= −c−1 =− 12πi

∫C+

0

∑n=−∞

cn−2

zn dz =− 12πi

∫C+

0

1z2

∑n=−∞

cn−2

zn−2 dz

= − 12πi

∫C+

0

1z2 f(

1z

)dz =−Resz=0

(1z2 f(

1z

))

Bernd Schroder Louisiana Tech University, College of Engineering and Science

The Residue Theorem

Page 139: The Residue Theorem · 2010-10-04 · 5.We will prove the requisite theorem (the Residue Theorem) in this presentation and we will also lay the abstract groundwork. 6.We will then

logo1

Singularities Residues Residue Theorem Residue at Infinity Types of Isolated Singularities Residues at Poles

Definition. Let f be analytic on C except for a finite number ofsingular points z1, . . . ,zn.

Assume that R1 is so that all |zj| ≤ R1. ForR0 > R1 let C−0 be the circle around the origin of radius R0, traversedin the clockwise, that is, the mathematically negative direction. Thenwe define the residue at infinity of f as

Resz=∞(f ) =1

2πi

∫C−0

f (z) dz.

Resz=∞(f ) =1

2πi

∫C−0

f (z) dz =− 12πi

∫C+

0

f (z) dz =− 12πi

∫C+

0

∑n=−∞

cnzn dz

= −c−1 =− 12πi

∫C+

0

∑n=−∞

cn−2

zn dz =− 12πi

∫C+

0

1z2

∑n=−∞

cn−2

zn−2 dz

= − 12πi

∫C+

0

1z2 f(

1z

)dz =−Resz=0

(1z2 f(

1z

))

Bernd Schroder Louisiana Tech University, College of Engineering and Science

The Residue Theorem

Page 140: The Residue Theorem · 2010-10-04 · 5.We will prove the requisite theorem (the Residue Theorem) in this presentation and we will also lay the abstract groundwork. 6.We will then

logo1

Singularities Residues Residue Theorem Residue at Infinity Types of Isolated Singularities Residues at Poles

Definition. Let f be analytic on C except for a finite number ofsingular points z1, . . . ,zn. Assume that R1 is so that all |zj| ≤ R1.

ForR0 > R1 let C−0 be the circle around the origin of radius R0, traversedin the clockwise, that is, the mathematically negative direction. Thenwe define the residue at infinity of f as

Resz=∞(f ) =1

2πi

∫C−0

f (z) dz.

Resz=∞(f ) =1

2πi

∫C−0

f (z) dz =− 12πi

∫C+

0

f (z) dz =− 12πi

∫C+

0

∑n=−∞

cnzn dz

= −c−1 =− 12πi

∫C+

0

∑n=−∞

cn−2

zn dz =− 12πi

∫C+

0

1z2

∑n=−∞

cn−2

zn−2 dz

= − 12πi

∫C+

0

1z2 f(

1z

)dz =−Resz=0

(1z2 f(

1z

))

Bernd Schroder Louisiana Tech University, College of Engineering and Science

The Residue Theorem

Page 141: The Residue Theorem · 2010-10-04 · 5.We will prove the requisite theorem (the Residue Theorem) in this presentation and we will also lay the abstract groundwork. 6.We will then

logo1

Singularities Residues Residue Theorem Residue at Infinity Types of Isolated Singularities Residues at Poles

Definition. Let f be analytic on C except for a finite number ofsingular points z1, . . . ,zn. Assume that R1 is so that all |zj| ≤ R1. ForR0 > R1 let C−0 be the circle around the origin of radius R0, traversedin the clockwise

, that is, the mathematically negative direction. Thenwe define the residue at infinity of f as

Resz=∞(f ) =1

2πi

∫C−0

f (z) dz.

Resz=∞(f ) =1

2πi

∫C−0

f (z) dz =− 12πi

∫C+

0

f (z) dz =− 12πi

∫C+

0

∑n=−∞

cnzn dz

= −c−1 =− 12πi

∫C+

0

∑n=−∞

cn−2

zn dz =− 12πi

∫C+

0

1z2

∑n=−∞

cn−2

zn−2 dz

= − 12πi

∫C+

0

1z2 f(

1z

)dz =−Resz=0

(1z2 f(

1z

))

Bernd Schroder Louisiana Tech University, College of Engineering and Science

The Residue Theorem

Page 142: The Residue Theorem · 2010-10-04 · 5.We will prove the requisite theorem (the Residue Theorem) in this presentation and we will also lay the abstract groundwork. 6.We will then

logo1

Singularities Residues Residue Theorem Residue at Infinity Types of Isolated Singularities Residues at Poles

Definition. Let f be analytic on C except for a finite number ofsingular points z1, . . . ,zn. Assume that R1 is so that all |zj| ≤ R1. ForR0 > R1 let C−0 be the circle around the origin of radius R0, traversedin the clockwise, that is, the mathematically negative

direction. Thenwe define the residue at infinity of f as

Resz=∞(f ) =1

2πi

∫C−0

f (z) dz.

Resz=∞(f ) =1

2πi

∫C−0

f (z) dz =− 12πi

∫C+

0

f (z) dz =− 12πi

∫C+

0

∑n=−∞

cnzn dz

= −c−1 =− 12πi

∫C+

0

∑n=−∞

cn−2

zn dz =− 12πi

∫C+

0

1z2

∑n=−∞

cn−2

zn−2 dz

= − 12πi

∫C+

0

1z2 f(

1z

)dz =−Resz=0

(1z2 f(

1z

))

Bernd Schroder Louisiana Tech University, College of Engineering and Science

The Residue Theorem

Page 143: The Residue Theorem · 2010-10-04 · 5.We will prove the requisite theorem (the Residue Theorem) in this presentation and we will also lay the abstract groundwork. 6.We will then

logo1

Singularities Residues Residue Theorem Residue at Infinity Types of Isolated Singularities Residues at Poles

Definition. Let f be analytic on C except for a finite number ofsingular points z1, . . . ,zn. Assume that R1 is so that all |zj| ≤ R1. ForR0 > R1 let C−0 be the circle around the origin of radius R0, traversedin the clockwise, that is, the mathematically negative direction.

Thenwe define the residue at infinity of f as

Resz=∞(f ) =1

2πi

∫C−0

f (z) dz.

Resz=∞(f ) =1

2πi

∫C−0

f (z) dz =− 12πi

∫C+

0

f (z) dz =− 12πi

∫C+

0

∑n=−∞

cnzn dz

= −c−1 =− 12πi

∫C+

0

∑n=−∞

cn−2

zn dz =− 12πi

∫C+

0

1z2

∑n=−∞

cn−2

zn−2 dz

= − 12πi

∫C+

0

1z2 f(

1z

)dz =−Resz=0

(1z2 f(

1z

))

Bernd Schroder Louisiana Tech University, College of Engineering and Science

The Residue Theorem

Page 144: The Residue Theorem · 2010-10-04 · 5.We will prove the requisite theorem (the Residue Theorem) in this presentation and we will also lay the abstract groundwork. 6.We will then

logo1

Singularities Residues Residue Theorem Residue at Infinity Types of Isolated Singularities Residues at Poles

Definition. Let f be analytic on C except for a finite number ofsingular points z1, . . . ,zn. Assume that R1 is so that all |zj| ≤ R1. ForR0 > R1 let C−0 be the circle around the origin of radius R0, traversedin the clockwise, that is, the mathematically negative direction. Thenwe define the residue at infinity of f as

Resz=∞(f ) =1

2πi

∫C−0

f (z) dz.

Resz=∞(f ) =1

2πi

∫C−0

f (z) dz =− 12πi

∫C+

0

f (z) dz =− 12πi

∫C+

0

∑n=−∞

cnzn dz

= −c−1 =− 12πi

∫C+

0

∑n=−∞

cn−2

zn dz =− 12πi

∫C+

0

1z2

∑n=−∞

cn−2

zn−2 dz

= − 12πi

∫C+

0

1z2 f(

1z

)dz =−Resz=0

(1z2 f(

1z

))

Bernd Schroder Louisiana Tech University, College of Engineering and Science

The Residue Theorem

Page 145: The Residue Theorem · 2010-10-04 · 5.We will prove the requisite theorem (the Residue Theorem) in this presentation and we will also lay the abstract groundwork. 6.We will then

logo1

Singularities Residues Residue Theorem Residue at Infinity Types of Isolated Singularities Residues at Poles

Definition. Let f be analytic on C except for a finite number ofsingular points z1, . . . ,zn. Assume that R1 is so that all |zj| ≤ R1. ForR0 > R1 let C−0 be the circle around the origin of radius R0, traversedin the clockwise, that is, the mathematically negative direction. Thenwe define the residue at infinity of f as

Resz=∞(f ) =1

2πi

∫C−0

f (z) dz.

Resz=∞(f ) =1

2πi

∫C−0

f (z) dz

=− 12πi

∫C+

0

f (z) dz =− 12πi

∫C+

0

∑n=−∞

cnzn dz

= −c−1 =− 12πi

∫C+

0

∑n=−∞

cn−2

zn dz =− 12πi

∫C+

0

1z2

∑n=−∞

cn−2

zn−2 dz

= − 12πi

∫C+

0

1z2 f(

1z

)dz =−Resz=0

(1z2 f(

1z

))

Bernd Schroder Louisiana Tech University, College of Engineering and Science

The Residue Theorem

Page 146: The Residue Theorem · 2010-10-04 · 5.We will prove the requisite theorem (the Residue Theorem) in this presentation and we will also lay the abstract groundwork. 6.We will then

logo1

Singularities Residues Residue Theorem Residue at Infinity Types of Isolated Singularities Residues at Poles

Definition. Let f be analytic on C except for a finite number ofsingular points z1, . . . ,zn. Assume that R1 is so that all |zj| ≤ R1. ForR0 > R1 let C−0 be the circle around the origin of radius R0, traversedin the clockwise, that is, the mathematically negative direction. Thenwe define the residue at infinity of f as

Resz=∞(f ) =1

2πi

∫C−0

f (z) dz.

Resz=∞(f ) =1

2πi

∫C−0

f (z) dz =− 12πi

∫C+

0

f (z) dz

=− 12πi

∫C+

0

∑n=−∞

cnzn dz

= −c−1 =− 12πi

∫C+

0

∑n=−∞

cn−2

zn dz =− 12πi

∫C+

0

1z2

∑n=−∞

cn−2

zn−2 dz

= − 12πi

∫C+

0

1z2 f(

1z

)dz =−Resz=0

(1z2 f(

1z

))

Bernd Schroder Louisiana Tech University, College of Engineering and Science

The Residue Theorem

Page 147: The Residue Theorem · 2010-10-04 · 5.We will prove the requisite theorem (the Residue Theorem) in this presentation and we will also lay the abstract groundwork. 6.We will then

logo1

Singularities Residues Residue Theorem Residue at Infinity Types of Isolated Singularities Residues at Poles

Definition. Let f be analytic on C except for a finite number ofsingular points z1, . . . ,zn. Assume that R1 is so that all |zj| ≤ R1. ForR0 > R1 let C−0 be the circle around the origin of radius R0, traversedin the clockwise, that is, the mathematically negative direction. Thenwe define the residue at infinity of f as

Resz=∞(f ) =1

2πi

∫C−0

f (z) dz.

Resz=∞(f ) =1

2πi

∫C−0

f (z) dz =− 12πi

∫C+

0

f (z) dz =− 12πi

∫C+

0

∑n=−∞

cnzn dz

= −c−1 =− 12πi

∫C+

0

∑n=−∞

cn−2

zn dz =− 12πi

∫C+

0

1z2

∑n=−∞

cn−2

zn−2 dz

= − 12πi

∫C+

0

1z2 f(

1z

)dz =−Resz=0

(1z2 f(

1z

))

Bernd Schroder Louisiana Tech University, College of Engineering and Science

The Residue Theorem

Page 148: The Residue Theorem · 2010-10-04 · 5.We will prove the requisite theorem (the Residue Theorem) in this presentation and we will also lay the abstract groundwork. 6.We will then

logo1

Singularities Residues Residue Theorem Residue at Infinity Types of Isolated Singularities Residues at Poles

Definition. Let f be analytic on C except for a finite number ofsingular points z1, . . . ,zn. Assume that R1 is so that all |zj| ≤ R1. ForR0 > R1 let C−0 be the circle around the origin of radius R0, traversedin the clockwise, that is, the mathematically negative direction. Thenwe define the residue at infinity of f as

Resz=∞(f ) =1

2πi

∫C−0

f (z) dz.

Resz=∞(f ) =1

2πi

∫C−0

f (z) dz =− 12πi

∫C+

0

f (z) dz =− 12πi

∫C+

0

∑n=−∞

cnzn dz

= −c−1

=− 12πi

∫C+

0

∑n=−∞

cn−2

zn dz =− 12πi

∫C+

0

1z2

∑n=−∞

cn−2

zn−2 dz

= − 12πi

∫C+

0

1z2 f(

1z

)dz =−Resz=0

(1z2 f(

1z

))

Bernd Schroder Louisiana Tech University, College of Engineering and Science

The Residue Theorem

Page 149: The Residue Theorem · 2010-10-04 · 5.We will prove the requisite theorem (the Residue Theorem) in this presentation and we will also lay the abstract groundwork. 6.We will then

logo1

Singularities Residues Residue Theorem Residue at Infinity Types of Isolated Singularities Residues at Poles

Definition. Let f be analytic on C except for a finite number ofsingular points z1, . . . ,zn. Assume that R1 is so that all |zj| ≤ R1. ForR0 > R1 let C−0 be the circle around the origin of radius R0, traversedin the clockwise, that is, the mathematically negative direction. Thenwe define the residue at infinity of f as

Resz=∞(f ) =1

2πi

∫C−0

f (z) dz.

Resz=∞(f ) =1

2πi

∫C−0

f (z) dz =− 12πi

∫C+

0

f (z) dz =− 12πi

∫C+

0

∑n=−∞

cnzn dz

= −c−1 =− 12πi

∫C+

0

∑n=−∞

cn−2

zn dz

=− 12πi

∫C+

0

1z2

∑n=−∞

cn−2

zn−2 dz

= − 12πi

∫C+

0

1z2 f(

1z

)dz =−Resz=0

(1z2 f(

1z

))

Bernd Schroder Louisiana Tech University, College of Engineering and Science

The Residue Theorem

Page 150: The Residue Theorem · 2010-10-04 · 5.We will prove the requisite theorem (the Residue Theorem) in this presentation and we will also lay the abstract groundwork. 6.We will then

logo1

Singularities Residues Residue Theorem Residue at Infinity Types of Isolated Singularities Residues at Poles

Definition. Let f be analytic on C except for a finite number ofsingular points z1, . . . ,zn. Assume that R1 is so that all |zj| ≤ R1. ForR0 > R1 let C−0 be the circle around the origin of radius R0, traversedin the clockwise, that is, the mathematically negative direction. Thenwe define the residue at infinity of f as

Resz=∞(f ) =1

2πi

∫C−0

f (z) dz.

Resz=∞(f ) =1

2πi

∫C−0

f (z) dz =− 12πi

∫C+

0

f (z) dz =− 12πi

∫C+

0

∑n=−∞

cnzn dz

= −c−1 =− 12πi

∫C+

0

∑n=−∞

cn−2

zn dz =− 12πi

∫C+

0

1z2

∑n=−∞

cn−2

zn−2 dz

= − 12πi

∫C+

0

1z2 f(

1z

)dz =−Resz=0

(1z2 f(

1z

))

Bernd Schroder Louisiana Tech University, College of Engineering and Science

The Residue Theorem

Page 151: The Residue Theorem · 2010-10-04 · 5.We will prove the requisite theorem (the Residue Theorem) in this presentation and we will also lay the abstract groundwork. 6.We will then

logo1

Singularities Residues Residue Theorem Residue at Infinity Types of Isolated Singularities Residues at Poles

Definition. Let f be analytic on C except for a finite number ofsingular points z1, . . . ,zn. Assume that R1 is so that all |zj| ≤ R1. ForR0 > R1 let C−0 be the circle around the origin of radius R0, traversedin the clockwise, that is, the mathematically negative direction. Thenwe define the residue at infinity of f as

Resz=∞(f ) =1

2πi

∫C−0

f (z) dz.

Resz=∞(f ) =1

2πi

∫C−0

f (z) dz =− 12πi

∫C+

0

f (z) dz =− 12πi

∫C+

0

∑n=−∞

cnzn dz

= −c−1 =− 12πi

∫C+

0

∑n=−∞

cn−2

zn dz =− 12πi

∫C+

0

1z2

∑n=−∞

cn−2

zn−2 dz

= − 12πi

∫C+

0

1z2 f(

1z

)dz

=−Resz=0

(1z2 f(

1z

))

Bernd Schroder Louisiana Tech University, College of Engineering and Science

The Residue Theorem

Page 152: The Residue Theorem · 2010-10-04 · 5.We will prove the requisite theorem (the Residue Theorem) in this presentation and we will also lay the abstract groundwork. 6.We will then

logo1

Singularities Residues Residue Theorem Residue at Infinity Types of Isolated Singularities Residues at Poles

Definition. Let f be analytic on C except for a finite number ofsingular points z1, . . . ,zn. Assume that R1 is so that all |zj| ≤ R1. ForR0 > R1 let C−0 be the circle around the origin of radius R0, traversedin the clockwise, that is, the mathematically negative direction. Thenwe define the residue at infinity of f as

Resz=∞(f ) =1

2πi

∫C−0

f (z) dz.

Resz=∞(f ) =1

2πi

∫C−0

f (z) dz =− 12πi

∫C+

0

f (z) dz =− 12πi

∫C+

0

∑n=−∞

cnzn dz

= −c−1 =− 12πi

∫C+

0

∑n=−∞

cn−2

zn dz =− 12πi

∫C+

0

1z2

∑n=−∞

cn−2

zn−2 dz

= − 12πi

∫C+

0

1z2 f(

1z

)dz =−Resz=0

(1z2 f(

1z

))Bernd Schroder Louisiana Tech University, College of Engineering and Science

The Residue Theorem

Page 153: The Residue Theorem · 2010-10-04 · 5.We will prove the requisite theorem (the Residue Theorem) in this presentation and we will also lay the abstract groundwork. 6.We will then

logo1

Singularities Residues Residue Theorem Residue at Infinity Types of Isolated Singularities Residues at Poles

Theorem.

If f is analytic on C, except for a finite number of singularpoints that lie in the interior of a positively oriented simple closedcontour C, then ∫

Cf (z) dz = 2πiResz=0

[1z2 f(

1z

)].

Proof. See previous panel.

Bernd Schroder Louisiana Tech University, College of Engineering and Science

The Residue Theorem

Page 154: The Residue Theorem · 2010-10-04 · 5.We will prove the requisite theorem (the Residue Theorem) in this presentation and we will also lay the abstract groundwork. 6.We will then

logo1

Singularities Residues Residue Theorem Residue at Infinity Types of Isolated Singularities Residues at Poles

Theorem. If f is analytic on C, except for a finite number of singularpoints that lie in the interior of a positively oriented simple closedcontour C, then ∫

Cf (z) dz = 2πiResz=0

[1z2 f(

1z

)].

Proof. See previous panel.

Bernd Schroder Louisiana Tech University, College of Engineering and Science

The Residue Theorem

Page 155: The Residue Theorem · 2010-10-04 · 5.We will prove the requisite theorem (the Residue Theorem) in this presentation and we will also lay the abstract groundwork. 6.We will then

logo1

Singularities Residues Residue Theorem Residue at Infinity Types of Isolated Singularities Residues at Poles

Theorem. If f is analytic on C, except for a finite number of singularpoints that lie in the interior of a positively oriented simple closedcontour C, then ∫

Cf (z) dz = 2πiResz=0

[1z2 f(

1z

)].

Proof.

See previous panel.

Bernd Schroder Louisiana Tech University, College of Engineering and Science

The Residue Theorem

Page 156: The Residue Theorem · 2010-10-04 · 5.We will prove the requisite theorem (the Residue Theorem) in this presentation and we will also lay the abstract groundwork. 6.We will then

logo1

Singularities Residues Residue Theorem Residue at Infinity Types of Isolated Singularities Residues at Poles

Theorem. If f is analytic on C, except for a finite number of singularpoints that lie in the interior of a positively oriented simple closedcontour C, then ∫

Cf (z) dz = 2πiResz=0

[1z2 f(

1z

)].

Proof. See previous panel.

Bernd Schroder Louisiana Tech University, College of Engineering and Science

The Residue Theorem

Page 157: The Residue Theorem · 2010-10-04 · 5.We will prove the requisite theorem (the Residue Theorem) in this presentation and we will also lay the abstract groundwork. 6.We will then

logo1

Singularities Residues Residue Theorem Residue at Infinity Types of Isolated Singularities Residues at Poles

Theorem. If f is analytic on C, except for a finite number of singularpoints that lie in the interior of a positively oriented simple closedcontour C, then ∫

Cf (z) dz = 2πiResz=0

[1z2 f(

1z

)].

Proof. See previous panel.

Bernd Schroder Louisiana Tech University, College of Engineering and Science

The Residue Theorem

Page 158: The Residue Theorem · 2010-10-04 · 5.We will prove the requisite theorem (the Residue Theorem) in this presentation and we will also lay the abstract groundwork. 6.We will then

logo1

Singularities Residues Residue Theorem Residue at Infinity Types of Isolated Singularities Residues at Poles

Example.

Integrate the function f (z) =z2

z2 +1over the positively

oriented boundary of the square of numbers z = x+ iy with (x,y) in[−3,3]× [−3,3].

-

6ℑ(z)

ℜ(z)6

?

-

3

ri

r−i

Bernd Schroder Louisiana Tech University, College of Engineering and Science

The Residue Theorem

Page 159: The Residue Theorem · 2010-10-04 · 5.We will prove the requisite theorem (the Residue Theorem) in this presentation and we will also lay the abstract groundwork. 6.We will then

logo1

Singularities Residues Residue Theorem Residue at Infinity Types of Isolated Singularities Residues at Poles

Example. Integrate the function f (z) =z2

z2 +1over the positively

oriented boundary of the square of numbers z = x+ iy with (x,y) in[−3,3]× [−3,3].

-

6ℑ(z)

ℜ(z)6

?

-

3

ri

r−i

Bernd Schroder Louisiana Tech University, College of Engineering and Science

The Residue Theorem

Page 160: The Residue Theorem · 2010-10-04 · 5.We will prove the requisite theorem (the Residue Theorem) in this presentation and we will also lay the abstract groundwork. 6.We will then

logo1

Singularities Residues Residue Theorem Residue at Infinity Types of Isolated Singularities Residues at Poles

Example. Integrate the function f (z) =z2

z2 +1over the positively

oriented boundary of the square of numbers z = x+ iy with (x,y) in[−3,3]× [−3,3].

-

6ℑ(z)

ℜ(z)

6

?

-

3

ri

r−i

Bernd Schroder Louisiana Tech University, College of Engineering and Science

The Residue Theorem

Page 161: The Residue Theorem · 2010-10-04 · 5.We will prove the requisite theorem (the Residue Theorem) in this presentation and we will also lay the abstract groundwork. 6.We will then

logo1

Singularities Residues Residue Theorem Residue at Infinity Types of Isolated Singularities Residues at Poles

Example. Integrate the function f (z) =z2

z2 +1over the positively

oriented boundary of the square of numbers z = x+ iy with (x,y) in[−3,3]× [−3,3].

-

6ℑ(z)

ℜ(z)6

?

-

3

ri

r−i

Bernd Schroder Louisiana Tech University, College of Engineering and Science

The Residue Theorem

Page 162: The Residue Theorem · 2010-10-04 · 5.We will prove the requisite theorem (the Residue Theorem) in this presentation and we will also lay the abstract groundwork. 6.We will then

logo1

Singularities Residues Residue Theorem Residue at Infinity Types of Isolated Singularities Residues at Poles

Example. Integrate the function f (z) =z2

z2 +1over the positively

oriented boundary of the square of numbers z = x+ iy with (x,y) in[−3,3]× [−3,3].

-

6ℑ(z)

ℜ(z)6

?

-

3

ri

r−i

Bernd Schroder Louisiana Tech University, College of Engineering and Science

The Residue Theorem

Page 163: The Residue Theorem · 2010-10-04 · 5.We will prove the requisite theorem (the Residue Theorem) in this presentation and we will also lay the abstract groundwork. 6.We will then

logo1

Singularities Residues Residue Theorem Residue at Infinity Types of Isolated Singularities Residues at Poles

Example. Integrate the function f (z) =z2

z2 +1over the positively

oriented boundary of the square of numbers z = x+ iy with (x,y) in[−3,3]× [−3,3].

-

6ℑ(z)

ℜ(z)6

?

-

3

ri

r−i

Bernd Schroder Louisiana Tech University, College of Engineering and Science

The Residue Theorem

Page 164: The Residue Theorem · 2010-10-04 · 5.We will prove the requisite theorem (the Residue Theorem) in this presentation and we will also lay the abstract groundwork. 6.We will then

logo1

Singularities Residues Residue Theorem Residue at Infinity Types of Isolated Singularities Residues at Poles

Example. Integrate the function f (z) =z2

z2 +1over the positively

oriented boundary of the square of numbers z = x+ iy with (x,y) in[−3,3]× [−3,3].

-

6ℑ(z)

ℜ(z)6

?

-

3

ri

r−i

Bernd Schroder Louisiana Tech University, College of Engineering and Science

The Residue Theorem

Page 165: The Residue Theorem · 2010-10-04 · 5.We will prove the requisite theorem (the Residue Theorem) in this presentation and we will also lay the abstract groundwork. 6.We will then

logo1

Singularities Residues Residue Theorem Residue at Infinity Types of Isolated Singularities Residues at Poles

Example. Integrate the function f (z) =z2

z2 +1over the positively

oriented boundary of the square of numbers z = x+ iy with (x,y) in[−3,3]× [−3,3].

-

6ℑ(z)

ℜ(z)6

?

-

3

ri

r−i

Bernd Schroder Louisiana Tech University, College of Engineering and Science

The Residue Theorem

Page 166: The Residue Theorem · 2010-10-04 · 5.We will prove the requisite theorem (the Residue Theorem) in this presentation and we will also lay the abstract groundwork. 6.We will then

logo1

Singularities Residues Residue Theorem Residue at Infinity Types of Isolated Singularities Residues at Poles

Example. Integrate the function f (z) =z2

z2 +1over the positively

oriented boundary of the square of numbers z = x+ iy with (x,y) in[−3,3]× [−3,3].

-

6ℑ(z)

ℜ(z)6

?

-

3

r

i

r−i

Bernd Schroder Louisiana Tech University, College of Engineering and Science

The Residue Theorem

Page 167: The Residue Theorem · 2010-10-04 · 5.We will prove the requisite theorem (the Residue Theorem) in this presentation and we will also lay the abstract groundwork. 6.We will then

logo1

Singularities Residues Residue Theorem Residue at Infinity Types of Isolated Singularities Residues at Poles

Example. Integrate the function f (z) =z2

z2 +1over the positively

oriented boundary of the square of numbers z = x+ iy with (x,y) in[−3,3]× [−3,3].

-

6ℑ(z)

ℜ(z)6

?

-

3

ri

r−i

Bernd Schroder Louisiana Tech University, College of Engineering and Science

The Residue Theorem

Page 168: The Residue Theorem · 2010-10-04 · 5.We will prove the requisite theorem (the Residue Theorem) in this presentation and we will also lay the abstract groundwork. 6.We will then

logo1

Singularities Residues Residue Theorem Residue at Infinity Types of Isolated Singularities Residues at Poles

Example. Integrate the function f (z) =z2

z2 +1over the positively

oriented boundary of the square of numbers z = x+ iy with (x,y) in[−3,3]× [−3,3].

-

6ℑ(z)

ℜ(z)6

?

-

3

ri

r

−i

Bernd Schroder Louisiana Tech University, College of Engineering and Science

The Residue Theorem

Page 169: The Residue Theorem · 2010-10-04 · 5.We will prove the requisite theorem (the Residue Theorem) in this presentation and we will also lay the abstract groundwork. 6.We will then

logo1

Singularities Residues Residue Theorem Residue at Infinity Types of Isolated Singularities Residues at Poles

Example. Integrate the function f (z) =z2

z2 +1over the positively

oriented boundary of the square of numbers z = x+ iy with (x,y) in[−3,3]× [−3,3].

-

6ℑ(z)

ℜ(z)6

?

-

3

ri

r−i

Bernd Schroder Louisiana Tech University, College of Engineering and Science

The Residue Theorem

Page 170: The Residue Theorem · 2010-10-04 · 5.We will prove the requisite theorem (the Residue Theorem) in this presentation and we will also lay the abstract groundwork. 6.We will then

logo1

Singularities Residues Residue Theorem Residue at Infinity Types of Isolated Singularities Residues at Poles

Example. Integrate the function f (z) =z2

z2 +1over the positively

oriented boundary of the square of numbers z = x+ iy with (x,y) in[−3,3]× [−3,3].

1z2 f(

1z

)=

1z2

(1z

)2(1z

)2+1

=1z2

11+ z2

=1z2

11− (−z2)

=1z2

∑n=0

(−1)nz2n

=1z2 +

∑n=1

(−1)nz2n−2

and hence∫

Cf (z) dz = 2πiResz=0

[1z2 f(

1z

)]= 0.

Bernd Schroder Louisiana Tech University, College of Engineering and Science

The Residue Theorem

Page 171: The Residue Theorem · 2010-10-04 · 5.We will prove the requisite theorem (the Residue Theorem) in this presentation and we will also lay the abstract groundwork. 6.We will then

logo1

Singularities Residues Residue Theorem Residue at Infinity Types of Isolated Singularities Residues at Poles

Example. Integrate the function f (z) =z2

z2 +1over the positively

oriented boundary of the square of numbers z = x+ iy with (x,y) in[−3,3]× [−3,3].

1z2 f(

1z

)

=1z2

(1z

)2(1z

)2+1

=1z2

11+ z2

=1z2

11− (−z2)

=1z2

∑n=0

(−1)nz2n

=1z2 +

∑n=1

(−1)nz2n−2

and hence∫

Cf (z) dz = 2πiResz=0

[1z2 f(

1z

)]= 0.

Bernd Schroder Louisiana Tech University, College of Engineering and Science

The Residue Theorem

Page 172: The Residue Theorem · 2010-10-04 · 5.We will prove the requisite theorem (the Residue Theorem) in this presentation and we will also lay the abstract groundwork. 6.We will then

logo1

Singularities Residues Residue Theorem Residue at Infinity Types of Isolated Singularities Residues at Poles

Example. Integrate the function f (z) =z2

z2 +1over the positively

oriented boundary of the square of numbers z = x+ iy with (x,y) in[−3,3]× [−3,3].

1z2 f(

1z

)=

1z2

(1z

)2(1z

)2+1

=1z2

11+ z2

=1z2

11− (−z2)

=1z2

∑n=0

(−1)nz2n

=1z2 +

∑n=1

(−1)nz2n−2

and hence∫

Cf (z) dz = 2πiResz=0

[1z2 f(

1z

)]= 0.

Bernd Schroder Louisiana Tech University, College of Engineering and Science

The Residue Theorem

Page 173: The Residue Theorem · 2010-10-04 · 5.We will prove the requisite theorem (the Residue Theorem) in this presentation and we will also lay the abstract groundwork. 6.We will then

logo1

Singularities Residues Residue Theorem Residue at Infinity Types of Isolated Singularities Residues at Poles

Example. Integrate the function f (z) =z2

z2 +1over the positively

oriented boundary of the square of numbers z = x+ iy with (x,y) in[−3,3]× [−3,3].

1z2 f(

1z

)=

1z2

(1z

)2(1z

)2+1

=1z2

11+ z2

=1z2

11− (−z2)

=1z2

∑n=0

(−1)nz2n

=1z2 +

∑n=1

(−1)nz2n−2

and hence∫

Cf (z) dz = 2πiResz=0

[1z2 f(

1z

)]= 0.

Bernd Schroder Louisiana Tech University, College of Engineering and Science

The Residue Theorem

Page 174: The Residue Theorem · 2010-10-04 · 5.We will prove the requisite theorem (the Residue Theorem) in this presentation and we will also lay the abstract groundwork. 6.We will then

logo1

Singularities Residues Residue Theorem Residue at Infinity Types of Isolated Singularities Residues at Poles

Example. Integrate the function f (z) =z2

z2 +1over the positively

oriented boundary of the square of numbers z = x+ iy with (x,y) in[−3,3]× [−3,3].

1z2 f(

1z

)=

1z2

(1z

)2(1z

)2+1

=1z2

11+ z2

=1z2

11− (−z2)

=1z2

∑n=0

(−1)nz2n

=1z2 +

∑n=1

(−1)nz2n−2

and hence∫

Cf (z) dz = 2πiResz=0

[1z2 f(

1z

)]= 0.

Bernd Schroder Louisiana Tech University, College of Engineering and Science

The Residue Theorem

Page 175: The Residue Theorem · 2010-10-04 · 5.We will prove the requisite theorem (the Residue Theorem) in this presentation and we will also lay the abstract groundwork. 6.We will then

logo1

Singularities Residues Residue Theorem Residue at Infinity Types of Isolated Singularities Residues at Poles

Example. Integrate the function f (z) =z2

z2 +1over the positively

oriented boundary of the square of numbers z = x+ iy with (x,y) in[−3,3]× [−3,3].

1z2 f(

1z

)=

1z2

(1z

)2(1z

)2+1

=1z2

11+ z2

=1z2

11− (−z2)

=1z2

∑n=0

(−1)nz2n

=1z2 +

∑n=1

(−1)nz2n−2

and hence∫

Cf (z) dz = 2πiResz=0

[1z2 f(

1z

)]= 0.

Bernd Schroder Louisiana Tech University, College of Engineering and Science

The Residue Theorem

Page 176: The Residue Theorem · 2010-10-04 · 5.We will prove the requisite theorem (the Residue Theorem) in this presentation and we will also lay the abstract groundwork. 6.We will then

logo1

Singularities Residues Residue Theorem Residue at Infinity Types of Isolated Singularities Residues at Poles

Example. Integrate the function f (z) =z2

z2 +1over the positively

oriented boundary of the square of numbers z = x+ iy with (x,y) in[−3,3]× [−3,3].

1z2 f(

1z

)=

1z2

(1z

)2(1z

)2+1

=1z2

11+ z2

=1z2

11− (−z2)

=1z2

∑n=0

(−1)nz2n

=1z2 +

∑n=1

(−1)nz2n−2

and hence∫

Cf (z) dz = 2πiResz=0

[1z2 f(

1z

)]= 0.

Bernd Schroder Louisiana Tech University, College of Engineering and Science

The Residue Theorem

Page 177: The Residue Theorem · 2010-10-04 · 5.We will prove the requisite theorem (the Residue Theorem) in this presentation and we will also lay the abstract groundwork. 6.We will then

logo1

Singularities Residues Residue Theorem Residue at Infinity Types of Isolated Singularities Residues at Poles

Example. Integrate the function f (z) =z2

z2 +1over the positively

oriented boundary of the square of numbers z = x+ iy with (x,y) in[−3,3]× [−3,3].

1z2 f(

1z

)=

1z2

(1z

)2(1z

)2+1

=1z2

11+ z2

=1z2

11− (−z2)

=1z2

∑n=0

(−1)nz2n

=1z2 +

∑n=1

(−1)nz2n−2

and hence∫

Cf (z) dz

= 2πiResz=0

[1z2 f(

1z

)]= 0.

Bernd Schroder Louisiana Tech University, College of Engineering and Science

The Residue Theorem

Page 178: The Residue Theorem · 2010-10-04 · 5.We will prove the requisite theorem (the Residue Theorem) in this presentation and we will also lay the abstract groundwork. 6.We will then

logo1

Singularities Residues Residue Theorem Residue at Infinity Types of Isolated Singularities Residues at Poles

Example. Integrate the function f (z) =z2

z2 +1over the positively

oriented boundary of the square of numbers z = x+ iy with (x,y) in[−3,3]× [−3,3].

1z2 f(

1z

)=

1z2

(1z

)2(1z

)2+1

=1z2

11+ z2

=1z2

11− (−z2)

=1z2

∑n=0

(−1)nz2n

=1z2 +

∑n=1

(−1)nz2n−2

and hence∫

Cf (z) dz = 2πiResz=0

[1z2 f(

1z

)]

= 0.

Bernd Schroder Louisiana Tech University, College of Engineering and Science

The Residue Theorem

Page 179: The Residue Theorem · 2010-10-04 · 5.We will prove the requisite theorem (the Residue Theorem) in this presentation and we will also lay the abstract groundwork. 6.We will then

logo1

Singularities Residues Residue Theorem Residue at Infinity Types of Isolated Singularities Residues at Poles

Example. Integrate the function f (z) =z2

z2 +1over the positively

oriented boundary of the square of numbers z = x+ iy with (x,y) in[−3,3]× [−3,3].

1z2 f(

1z

)=

1z2

(1z

)2(1z

)2+1

=1z2

11+ z2

=1z2

11− (−z2)

=1z2

∑n=0

(−1)nz2n

=1z2 +

∑n=1

(−1)nz2n−2

and hence∫

Cf (z) dz = 2πiResz=0

[1z2 f(

1z

)]= 0.

Bernd Schroder Louisiana Tech University, College of Engineering and Science

The Residue Theorem

Page 180: The Residue Theorem · 2010-10-04 · 5.We will prove the requisite theorem (the Residue Theorem) in this presentation and we will also lay the abstract groundwork. 6.We will then

logo1

Singularities Residues Residue Theorem Residue at Infinity Types of Isolated Singularities Residues at Poles

Definition.

Let z0 be a complex number, let r > 0 and let f be analyticin a deleted neighborhood of z0. In this case z0 is called an isolatedsingularity of f . Consider the Laurent expansion of f around z0

f (z) =∞

∑n=0

an(z− z0)n +b1

z− z0+

b2

(z− z0)2 +b3

(z− z0)3 + · · ·

The sum of the negative powers of z− z0 is also called the principalpart of f .

1. If bn = 0 for all n, then z0 is called a removable singularity.2. If there is a positive number m so that bm 6= 0 and bn = 0 for all

n > m, then z0 is called a pole of order m.3. If the number m in part 2 equals 1, then z0 is also called a

simple pole.4. If z0 is not removable and there is no number as in part 2, then z0

is called an essential singularity.

Bernd Schroder Louisiana Tech University, College of Engineering and Science

The Residue Theorem

Page 181: The Residue Theorem · 2010-10-04 · 5.We will prove the requisite theorem (the Residue Theorem) in this presentation and we will also lay the abstract groundwork. 6.We will then

logo1

Singularities Residues Residue Theorem Residue at Infinity Types of Isolated Singularities Residues at Poles

Definition. Let z0 be a complex number, let r > 0 and let f be analyticin a deleted neighborhood of z0.

In this case z0 is called an isolatedsingularity of f . Consider the Laurent expansion of f around z0

f (z) =∞

∑n=0

an(z− z0)n +b1

z− z0+

b2

(z− z0)2 +b3

(z− z0)3 + · · ·

The sum of the negative powers of z− z0 is also called the principalpart of f .

1. If bn = 0 for all n, then z0 is called a removable singularity.2. If there is a positive number m so that bm 6= 0 and bn = 0 for all

n > m, then z0 is called a pole of order m.3. If the number m in part 2 equals 1, then z0 is also called a

simple pole.4. If z0 is not removable and there is no number as in part 2, then z0

is called an essential singularity.

Bernd Schroder Louisiana Tech University, College of Engineering and Science

The Residue Theorem

Page 182: The Residue Theorem · 2010-10-04 · 5.We will prove the requisite theorem (the Residue Theorem) in this presentation and we will also lay the abstract groundwork. 6.We will then

logo1

Singularities Residues Residue Theorem Residue at Infinity Types of Isolated Singularities Residues at Poles

Definition. Let z0 be a complex number, let r > 0 and let f be analyticin a deleted neighborhood of z0. In this case z0 is called an isolatedsingularity of f .

Consider the Laurent expansion of f around z0

f (z) =∞

∑n=0

an(z− z0)n +b1

z− z0+

b2

(z− z0)2 +b3

(z− z0)3 + · · ·

The sum of the negative powers of z− z0 is also called the principalpart of f .

1. If bn = 0 for all n, then z0 is called a removable singularity.2. If there is a positive number m so that bm 6= 0 and bn = 0 for all

n > m, then z0 is called a pole of order m.3. If the number m in part 2 equals 1, then z0 is also called a

simple pole.4. If z0 is not removable and there is no number as in part 2, then z0

is called an essential singularity.

Bernd Schroder Louisiana Tech University, College of Engineering and Science

The Residue Theorem

Page 183: The Residue Theorem · 2010-10-04 · 5.We will prove the requisite theorem (the Residue Theorem) in this presentation and we will also lay the abstract groundwork. 6.We will then

logo1

Singularities Residues Residue Theorem Residue at Infinity Types of Isolated Singularities Residues at Poles

Definition. Let z0 be a complex number, let r > 0 and let f be analyticin a deleted neighborhood of z0. In this case z0 is called an isolatedsingularity of f . Consider the Laurent expansion of f around z0

f (z) =∞

∑n=0

an(z− z0)n +b1

z− z0+

b2

(z− z0)2 +b3

(z− z0)3 + · · ·

The sum of the negative powers of z− z0 is also called the principalpart of f .

1. If bn = 0 for all n, then z0 is called a removable singularity.2. If there is a positive number m so that bm 6= 0 and bn = 0 for all

n > m, then z0 is called a pole of order m.3. If the number m in part 2 equals 1, then z0 is also called a

simple pole.4. If z0 is not removable and there is no number as in part 2, then z0

is called an essential singularity.

Bernd Schroder Louisiana Tech University, College of Engineering and Science

The Residue Theorem

Page 184: The Residue Theorem · 2010-10-04 · 5.We will prove the requisite theorem (the Residue Theorem) in this presentation and we will also lay the abstract groundwork. 6.We will then

logo1

Singularities Residues Residue Theorem Residue at Infinity Types of Isolated Singularities Residues at Poles

Definition. Let z0 be a complex number, let r > 0 and let f be analyticin a deleted neighborhood of z0. In this case z0 is called an isolatedsingularity of f . Consider the Laurent expansion of f around z0

f (z) =∞

∑n=0

an(z− z0)n +b1

z− z0+

b2

(z− z0)2 +b3

(z− z0)3 + · · ·

The sum of the negative powers of z− z0 is also called the principalpart of f .

1. If bn = 0 for all n, then z0 is called a removable singularity.2. If there is a positive number m so that bm 6= 0 and bn = 0 for all

n > m, then z0 is called a pole of order m.3. If the number m in part 2 equals 1, then z0 is also called a

simple pole.4. If z0 is not removable and there is no number as in part 2, then z0

is called an essential singularity.

Bernd Schroder Louisiana Tech University, College of Engineering and Science

The Residue Theorem

Page 185: The Residue Theorem · 2010-10-04 · 5.We will prove the requisite theorem (the Residue Theorem) in this presentation and we will also lay the abstract groundwork. 6.We will then

logo1

Singularities Residues Residue Theorem Residue at Infinity Types of Isolated Singularities Residues at Poles

Definition. Let z0 be a complex number, let r > 0 and let f be analyticin a deleted neighborhood of z0. In this case z0 is called an isolatedsingularity of f . Consider the Laurent expansion of f around z0

f (z) =∞

∑n=0

an(z− z0)n +b1

z− z0+

b2

(z− z0)2 +b3

(z− z0)3 + · · ·

The sum of the negative powers of z− z0 is also called the principalpart of f .

1. If bn = 0 for all n, then z0 is called a removable singularity.

2. If there is a positive number m so that bm 6= 0 and bn = 0 for alln > m, then z0 is called a pole of order m.

3. If the number m in part 2 equals 1, then z0 is also called asimple pole.

4. If z0 is not removable and there is no number as in part 2, then z0is called an essential singularity.

Bernd Schroder Louisiana Tech University, College of Engineering and Science

The Residue Theorem

Page 186: The Residue Theorem · 2010-10-04 · 5.We will prove the requisite theorem (the Residue Theorem) in this presentation and we will also lay the abstract groundwork. 6.We will then

logo1

Singularities Residues Residue Theorem Residue at Infinity Types of Isolated Singularities Residues at Poles

Definition. Let z0 be a complex number, let r > 0 and let f be analyticin a deleted neighborhood of z0. In this case z0 is called an isolatedsingularity of f . Consider the Laurent expansion of f around z0

f (z) =∞

∑n=0

an(z− z0)n +b1

z− z0+

b2

(z− z0)2 +b3

(z− z0)3 + · · ·

The sum of the negative powers of z− z0 is also called the principalpart of f .

1. If bn = 0 for all n, then z0 is called a removable singularity.2. If there is a positive number m so that bm 6= 0 and bn = 0 for all

n > m, then z0 is called a pole of order m.

3. If the number m in part 2 equals 1, then z0 is also called asimple pole.

4. If z0 is not removable and there is no number as in part 2, then z0is called an essential singularity.

Bernd Schroder Louisiana Tech University, College of Engineering and Science

The Residue Theorem

Page 187: The Residue Theorem · 2010-10-04 · 5.We will prove the requisite theorem (the Residue Theorem) in this presentation and we will also lay the abstract groundwork. 6.We will then

logo1

Singularities Residues Residue Theorem Residue at Infinity Types of Isolated Singularities Residues at Poles

Definition. Let z0 be a complex number, let r > 0 and let f be analyticin a deleted neighborhood of z0. In this case z0 is called an isolatedsingularity of f . Consider the Laurent expansion of f around z0

f (z) =∞

∑n=0

an(z− z0)n +b1

z− z0+

b2

(z− z0)2 +b3

(z− z0)3 + · · ·

The sum of the negative powers of z− z0 is also called the principalpart of f .

1. If bn = 0 for all n, then z0 is called a removable singularity.2. If there is a positive number m so that bm 6= 0 and bn = 0 for all

n > m, then z0 is called a pole of order m.3. If the number m in part 2 equals 1, then z0 is also called a

simple pole.

4. If z0 is not removable and there is no number as in part 2, then z0is called an essential singularity.

Bernd Schroder Louisiana Tech University, College of Engineering and Science

The Residue Theorem

Page 188: The Residue Theorem · 2010-10-04 · 5.We will prove the requisite theorem (the Residue Theorem) in this presentation and we will also lay the abstract groundwork. 6.We will then

logo1

Singularities Residues Residue Theorem Residue at Infinity Types of Isolated Singularities Residues at Poles

Definition. Let z0 be a complex number, let r > 0 and let f be analyticin a deleted neighborhood of z0. In this case z0 is called an isolatedsingularity of f . Consider the Laurent expansion of f around z0

f (z) =∞

∑n=0

an(z− z0)n +b1

z− z0+

b2

(z− z0)2 +b3

(z− z0)3 + · · ·

The sum of the negative powers of z− z0 is also called the principalpart of f .

1. If bn = 0 for all n, then z0 is called a removable singularity.2. If there is a positive number m so that bm 6= 0 and bn = 0 for all

n > m, then z0 is called a pole of order m.3. If the number m in part 2 equals 1, then z0 is also called a

simple pole.4. If z0 is not removable and there is no number as in part 2, then z0

is called an essential singularity.

Bernd Schroder Louisiana Tech University, College of Engineering and Science

The Residue Theorem

Page 189: The Residue Theorem · 2010-10-04 · 5.We will prove the requisite theorem (the Residue Theorem) in this presentation and we will also lay the abstract groundwork. 6.We will then

logo1

Singularities Residues Residue Theorem Residue at Infinity Types of Isolated Singularities Residues at Poles

Example.

The function f (z) =sin(z)

zhas a removable singularity at

z = 0.sin(z)

z=

1z

sin(z)

=1z

∑n=0

(−1)n

(2n+1)!z2n+1

=∞

∑n=0

(−1)n

(2n+1)!z2n

Bernd Schroder Louisiana Tech University, College of Engineering and Science

The Residue Theorem

Page 190: The Residue Theorem · 2010-10-04 · 5.We will prove the requisite theorem (the Residue Theorem) in this presentation and we will also lay the abstract groundwork. 6.We will then

logo1

Singularities Residues Residue Theorem Residue at Infinity Types of Isolated Singularities Residues at Poles

Example. The function f (z) =sin(z)

zhas a removable singularity at

z = 0.

sin(z)z

=1z

sin(z)

=1z

∑n=0

(−1)n

(2n+1)!z2n+1

=∞

∑n=0

(−1)n

(2n+1)!z2n

Bernd Schroder Louisiana Tech University, College of Engineering and Science

The Residue Theorem

Page 191: The Residue Theorem · 2010-10-04 · 5.We will prove the requisite theorem (the Residue Theorem) in this presentation and we will also lay the abstract groundwork. 6.We will then

logo1

Singularities Residues Residue Theorem Residue at Infinity Types of Isolated Singularities Residues at Poles

Example. The function f (z) =sin(z)

zhas a removable singularity at

z = 0.sin(z)

z

=1z

sin(z)

=1z

∑n=0

(−1)n

(2n+1)!z2n+1

=∞

∑n=0

(−1)n

(2n+1)!z2n

Bernd Schroder Louisiana Tech University, College of Engineering and Science

The Residue Theorem

Page 192: The Residue Theorem · 2010-10-04 · 5.We will prove the requisite theorem (the Residue Theorem) in this presentation and we will also lay the abstract groundwork. 6.We will then

logo1

Singularities Residues Residue Theorem Residue at Infinity Types of Isolated Singularities Residues at Poles

Example. The function f (z) =sin(z)

zhas a removable singularity at

z = 0.sin(z)

z=

1z

sin(z)

=1z

∑n=0

(−1)n

(2n+1)!z2n+1

=∞

∑n=0

(−1)n

(2n+1)!z2n

Bernd Schroder Louisiana Tech University, College of Engineering and Science

The Residue Theorem

Page 193: The Residue Theorem · 2010-10-04 · 5.We will prove the requisite theorem (the Residue Theorem) in this presentation and we will also lay the abstract groundwork. 6.We will then

logo1

Singularities Residues Residue Theorem Residue at Infinity Types of Isolated Singularities Residues at Poles

Example. The function f (z) =sin(z)

zhas a removable singularity at

z = 0.sin(z)

z=

1z

sin(z)

=1z

∑n=0

(−1)n

(2n+1)!z2n+1

=∞

∑n=0

(−1)n

(2n+1)!z2n

Bernd Schroder Louisiana Tech University, College of Engineering and Science

The Residue Theorem

Page 194: The Residue Theorem · 2010-10-04 · 5.We will prove the requisite theorem (the Residue Theorem) in this presentation and we will also lay the abstract groundwork. 6.We will then

logo1

Singularities Residues Residue Theorem Residue at Infinity Types of Isolated Singularities Residues at Poles

Example. The function f (z) =sin(z)

zhas a removable singularity at

z = 0.sin(z)

z=

1z

sin(z)

=1z

∑n=0

(−1)n

(2n+1)!z2n+1

=∞

∑n=0

(−1)n

(2n+1)!z2n

Bernd Schroder Louisiana Tech University, College of Engineering and Science

The Residue Theorem

Page 195: The Residue Theorem · 2010-10-04 · 5.We will prove the requisite theorem (the Residue Theorem) in this presentation and we will also lay the abstract groundwork. 6.We will then

logo1

Singularities Residues Residue Theorem Residue at Infinity Types of Isolated Singularities Residues at Poles

Example.

The function f (z) =1

1− z2 has simple poles at z = 1 and at

z =−1.1

1− z2 =1

z+11

z−1

Bernd Schroder Louisiana Tech University, College of Engineering and Science

The Residue Theorem

Page 196: The Residue Theorem · 2010-10-04 · 5.We will prove the requisite theorem (the Residue Theorem) in this presentation and we will also lay the abstract groundwork. 6.We will then

logo1

Singularities Residues Residue Theorem Residue at Infinity Types of Isolated Singularities Residues at Poles

Example. The function f (z) =1

1− z2 has simple poles at z = 1 and at

z =−1.

11− z2 =

1z+1

1z−1

Bernd Schroder Louisiana Tech University, College of Engineering and Science

The Residue Theorem

Page 197: The Residue Theorem · 2010-10-04 · 5.We will prove the requisite theorem (the Residue Theorem) in this presentation and we will also lay the abstract groundwork. 6.We will then

logo1

Singularities Residues Residue Theorem Residue at Infinity Types of Isolated Singularities Residues at Poles

Example. The function f (z) =1

1− z2 has simple poles at z = 1 and at

z =−1.1

1− z2

=1

z+11

z−1

Bernd Schroder Louisiana Tech University, College of Engineering and Science

The Residue Theorem

Page 198: The Residue Theorem · 2010-10-04 · 5.We will prove the requisite theorem (the Residue Theorem) in this presentation and we will also lay the abstract groundwork. 6.We will then

logo1

Singularities Residues Residue Theorem Residue at Infinity Types of Isolated Singularities Residues at Poles

Example. The function f (z) =1

1− z2 has simple poles at z = 1 and at

z =−1.1

1− z2 =1

z+11

z−1

Bernd Schroder Louisiana Tech University, College of Engineering and Science

The Residue Theorem

Page 199: The Residue Theorem · 2010-10-04 · 5.We will prove the requisite theorem (the Residue Theorem) in this presentation and we will also lay the abstract groundwork. 6.We will then

logo1

Singularities Residues Residue Theorem Residue at Infinity Types of Isolated Singularities Residues at Poles

Example.

The function f (z) =1+ z2

z3 has a pole of order 3 at z = 0.

1+ z2

z3 =1z3 +

1z

Bernd Schroder Louisiana Tech University, College of Engineering and Science

The Residue Theorem

Page 200: The Residue Theorem · 2010-10-04 · 5.We will prove the requisite theorem (the Residue Theorem) in this presentation and we will also lay the abstract groundwork. 6.We will then

logo1

Singularities Residues Residue Theorem Residue at Infinity Types of Isolated Singularities Residues at Poles

Example. The function f (z) =1+ z2

z3 has a pole of order 3 at z = 0.

1+ z2

z3 =1z3 +

1z

Bernd Schroder Louisiana Tech University, College of Engineering and Science

The Residue Theorem

Page 201: The Residue Theorem · 2010-10-04 · 5.We will prove the requisite theorem (the Residue Theorem) in this presentation and we will also lay the abstract groundwork. 6.We will then

logo1

Singularities Residues Residue Theorem Residue at Infinity Types of Isolated Singularities Residues at Poles

Example. The function f (z) =1+ z2

z3 has a pole of order 3 at z = 0.

1+ z2

z3

=1z3 +

1z

Bernd Schroder Louisiana Tech University, College of Engineering and Science

The Residue Theorem

Page 202: The Residue Theorem · 2010-10-04 · 5.We will prove the requisite theorem (the Residue Theorem) in this presentation and we will also lay the abstract groundwork. 6.We will then

logo1

Singularities Residues Residue Theorem Residue at Infinity Types of Isolated Singularities Residues at Poles

Example. The function f (z) =1+ z2

z3 has a pole of order 3 at z = 0.

1+ z2

z3 =1z3 +

1z

Bernd Schroder Louisiana Tech University, College of Engineering and Science

The Residue Theorem

Page 203: The Residue Theorem · 2010-10-04 · 5.We will prove the requisite theorem (the Residue Theorem) in this presentation and we will also lay the abstract groundwork. 6.We will then

logo1

Singularities Residues Residue Theorem Residue at Infinity Types of Isolated Singularities Residues at Poles

Example.

The function f (z) = e1z has an essential singularity at

z = 0.

e1z =

∑n=0

1n!

1zn

Bernd Schroder Louisiana Tech University, College of Engineering and Science

The Residue Theorem

Page 204: The Residue Theorem · 2010-10-04 · 5.We will prove the requisite theorem (the Residue Theorem) in this presentation and we will also lay the abstract groundwork. 6.We will then

logo1

Singularities Residues Residue Theorem Residue at Infinity Types of Isolated Singularities Residues at Poles

Example. The function f (z) = e1z has an essential singularity at

z = 0.

e1z =

∑n=0

1n!

1zn

Bernd Schroder Louisiana Tech University, College of Engineering and Science

The Residue Theorem

Page 205: The Residue Theorem · 2010-10-04 · 5.We will prove the requisite theorem (the Residue Theorem) in this presentation and we will also lay the abstract groundwork. 6.We will then

logo1

Singularities Residues Residue Theorem Residue at Infinity Types of Isolated Singularities Residues at Poles

Example. The function f (z) = e1z has an essential singularity at

z = 0.

e1z

=∞

∑n=0

1n!

1zn

Bernd Schroder Louisiana Tech University, College of Engineering and Science

The Residue Theorem

Page 206: The Residue Theorem · 2010-10-04 · 5.We will prove the requisite theorem (the Residue Theorem) in this presentation and we will also lay the abstract groundwork. 6.We will then

logo1

Singularities Residues Residue Theorem Residue at Infinity Types of Isolated Singularities Residues at Poles

Example. The function f (z) = e1z has an essential singularity at

z = 0.

e1z =

∑n=0

1n!

1zn

Bernd Schroder Louisiana Tech University, College of Engineering and Science

The Residue Theorem

Page 207: The Residue Theorem · 2010-10-04 · 5.We will prove the requisite theorem (the Residue Theorem) in this presentation and we will also lay the abstract groundwork. 6.We will then

logo1

Singularities Residues Residue Theorem Residue at Infinity Types of Isolated Singularities Residues at Poles

Theorem.

Let the function f be analytic in a deleted neighborhood ofthe point z0. Then z0 is a pole of order m if and only if there is functionΦ that is analytic in a neighborhood of z0, Φ(z0) 6= 0 and so that

f (z) =Φ(z)

(z− z0)m

for all z in a deleted neighborhood of z0. Moreover,

Resz=z0(f ) =Φ(m−1)(z0)(m−1)!

Bernd Schroder Louisiana Tech University, College of Engineering and Science

The Residue Theorem

Page 208: The Residue Theorem · 2010-10-04 · 5.We will prove the requisite theorem (the Residue Theorem) in this presentation and we will also lay the abstract groundwork. 6.We will then

logo1

Singularities Residues Residue Theorem Residue at Infinity Types of Isolated Singularities Residues at Poles

Theorem. Let the function f be analytic in a deleted neighborhood ofthe point z0.

Then z0 is a pole of order m if and only if there is functionΦ that is analytic in a neighborhood of z0, Φ(z0) 6= 0 and so that

f (z) =Φ(z)

(z− z0)m

for all z in a deleted neighborhood of z0. Moreover,

Resz=z0(f ) =Φ(m−1)(z0)(m−1)!

Bernd Schroder Louisiana Tech University, College of Engineering and Science

The Residue Theorem

Page 209: The Residue Theorem · 2010-10-04 · 5.We will prove the requisite theorem (the Residue Theorem) in this presentation and we will also lay the abstract groundwork. 6.We will then

logo1

Singularities Residues Residue Theorem Residue at Infinity Types of Isolated Singularities Residues at Poles

Theorem. Let the function f be analytic in a deleted neighborhood ofthe point z0. Then z0 is a pole of order m if and only if there is functionΦ that is analytic in a neighborhood of z0

, Φ(z0) 6= 0 and so that

f (z) =Φ(z)

(z− z0)m

for all z in a deleted neighborhood of z0. Moreover,

Resz=z0(f ) =Φ(m−1)(z0)(m−1)!

Bernd Schroder Louisiana Tech University, College of Engineering and Science

The Residue Theorem

Page 210: The Residue Theorem · 2010-10-04 · 5.We will prove the requisite theorem (the Residue Theorem) in this presentation and we will also lay the abstract groundwork. 6.We will then

logo1

Singularities Residues Residue Theorem Residue at Infinity Types of Isolated Singularities Residues at Poles

Theorem. Let the function f be analytic in a deleted neighborhood ofthe point z0. Then z0 is a pole of order m if and only if there is functionΦ that is analytic in a neighborhood of z0, Φ(z0) 6= 0

and so that

f (z) =Φ(z)

(z− z0)m

for all z in a deleted neighborhood of z0. Moreover,

Resz=z0(f ) =Φ(m−1)(z0)(m−1)!

Bernd Schroder Louisiana Tech University, College of Engineering and Science

The Residue Theorem

Page 211: The Residue Theorem · 2010-10-04 · 5.We will prove the requisite theorem (the Residue Theorem) in this presentation and we will also lay the abstract groundwork. 6.We will then

logo1

Singularities Residues Residue Theorem Residue at Infinity Types of Isolated Singularities Residues at Poles

Theorem. Let the function f be analytic in a deleted neighborhood ofthe point z0. Then z0 is a pole of order m if and only if there is functionΦ that is analytic in a neighborhood of z0, Φ(z0) 6= 0 and so that

f (z) =Φ(z)

(z− z0)m

for all z in a deleted neighborhood of z0.

Moreover,

Resz=z0(f ) =Φ(m−1)(z0)(m−1)!

Bernd Schroder Louisiana Tech University, College of Engineering and Science

The Residue Theorem

Page 212: The Residue Theorem · 2010-10-04 · 5.We will prove the requisite theorem (the Residue Theorem) in this presentation and we will also lay the abstract groundwork. 6.We will then

logo1

Singularities Residues Residue Theorem Residue at Infinity Types of Isolated Singularities Residues at Poles

Theorem. Let the function f be analytic in a deleted neighborhood ofthe point z0. Then z0 is a pole of order m if and only if there is functionΦ that is analytic in a neighborhood of z0, Φ(z0) 6= 0 and so that

f (z) =Φ(z)

(z− z0)m

for all z in a deleted neighborhood of z0. Moreover,

Resz=z0(f ) =Φ(m−1)(z0)(m−1)!

Bernd Schroder Louisiana Tech University, College of Engineering and Science

The Residue Theorem

Page 213: The Residue Theorem · 2010-10-04 · 5.We will prove the requisite theorem (the Residue Theorem) in this presentation and we will also lay the abstract groundwork. 6.We will then

logo1

Singularities Residues Residue Theorem Residue at Infinity Types of Isolated Singularities Residues at Poles

Proof.

If f has a pole of order m around z0, then the Laurentexpansion of z for 0 < |z− z0|< R (for some R) is (with c−m 6= 0)

f (z) =∞

∑n=−m

cn(z− z0)n =1

(z− z0)m

∑n=−m

cn(z− z0)n+m

=1

(z− z0)m

∑k=0

ck−m(z− z0)k =:Φ(z)

(z− z0)m .

Conversely, if f (z) =Φ(z)

(z− z0)m with Φ(z0) 6= 0, then the above

computation in reverse shows that f has a pole of order m. Moreover,in this situation, the coefficient c−1 = Resz=z0(f ) of the Laurent

expansion of f is the coefficient am−1 =Φ(m−1)(z0)(m−1)!

of the power

series expansion of Φ.

Bernd Schroder Louisiana Tech University, College of Engineering and Science

The Residue Theorem

Page 214: The Residue Theorem · 2010-10-04 · 5.We will prove the requisite theorem (the Residue Theorem) in this presentation and we will also lay the abstract groundwork. 6.We will then

logo1

Singularities Residues Residue Theorem Residue at Infinity Types of Isolated Singularities Residues at Poles

Proof. If f has a pole of order m around z0, then the Laurentexpansion of z for 0 < |z− z0|< R (for some R) is (with c−m 6= 0)

f (z) =∞

∑n=−m

cn(z− z0)n

=1

(z− z0)m

∑n=−m

cn(z− z0)n+m

=1

(z− z0)m

∑k=0

ck−m(z− z0)k =:Φ(z)

(z− z0)m .

Conversely, if f (z) =Φ(z)

(z− z0)m with Φ(z0) 6= 0, then the above

computation in reverse shows that f has a pole of order m. Moreover,in this situation, the coefficient c−1 = Resz=z0(f ) of the Laurent

expansion of f is the coefficient am−1 =Φ(m−1)(z0)(m−1)!

of the power

series expansion of Φ.

Bernd Schroder Louisiana Tech University, College of Engineering and Science

The Residue Theorem

Page 215: The Residue Theorem · 2010-10-04 · 5.We will prove the requisite theorem (the Residue Theorem) in this presentation and we will also lay the abstract groundwork. 6.We will then

logo1

Singularities Residues Residue Theorem Residue at Infinity Types of Isolated Singularities Residues at Poles

Proof. If f has a pole of order m around z0, then the Laurentexpansion of z for 0 < |z− z0|< R (for some R) is (with c−m 6= 0)

f (z) =∞

∑n=−m

cn(z− z0)n =1

(z− z0)m

∑n=−m

cn(z− z0)n+m

=1

(z− z0)m

∑k=0

ck−m(z− z0)k =:Φ(z)

(z− z0)m .

Conversely, if f (z) =Φ(z)

(z− z0)m with Φ(z0) 6= 0, then the above

computation in reverse shows that f has a pole of order m. Moreover,in this situation, the coefficient c−1 = Resz=z0(f ) of the Laurent

expansion of f is the coefficient am−1 =Φ(m−1)(z0)(m−1)!

of the power

series expansion of Φ.

Bernd Schroder Louisiana Tech University, College of Engineering and Science

The Residue Theorem

Page 216: The Residue Theorem · 2010-10-04 · 5.We will prove the requisite theorem (the Residue Theorem) in this presentation and we will also lay the abstract groundwork. 6.We will then

logo1

Singularities Residues Residue Theorem Residue at Infinity Types of Isolated Singularities Residues at Poles

Proof. If f has a pole of order m around z0, then the Laurentexpansion of z for 0 < |z− z0|< R (for some R) is (with c−m 6= 0)

f (z) =∞

∑n=−m

cn(z− z0)n =1

(z− z0)m

∑n=−m

cn(z− z0)n+m

=1

(z− z0)m

∑k=0

ck−m(z− z0)k

=:Φ(z)

(z− z0)m .

Conversely, if f (z) =Φ(z)

(z− z0)m with Φ(z0) 6= 0, then the above

computation in reverse shows that f has a pole of order m. Moreover,in this situation, the coefficient c−1 = Resz=z0(f ) of the Laurent

expansion of f is the coefficient am−1 =Φ(m−1)(z0)(m−1)!

of the power

series expansion of Φ.

Bernd Schroder Louisiana Tech University, College of Engineering and Science

The Residue Theorem

Page 217: The Residue Theorem · 2010-10-04 · 5.We will prove the requisite theorem (the Residue Theorem) in this presentation and we will also lay the abstract groundwork. 6.We will then

logo1

Singularities Residues Residue Theorem Residue at Infinity Types of Isolated Singularities Residues at Poles

Proof. If f has a pole of order m around z0, then the Laurentexpansion of z for 0 < |z− z0|< R (for some R) is (with c−m 6= 0)

f (z) =∞

∑n=−m

cn(z− z0)n =1

(z− z0)m

∑n=−m

cn(z− z0)n+m

=1

(z− z0)m

∑k=0

ck−m(z− z0)k =:Φ(z)

(z− z0)m .

Conversely, if f (z) =Φ(z)

(z− z0)m with Φ(z0) 6= 0, then the above

computation in reverse shows that f has a pole of order m. Moreover,in this situation, the coefficient c−1 = Resz=z0(f ) of the Laurent

expansion of f is the coefficient am−1 =Φ(m−1)(z0)(m−1)!

of the power

series expansion of Φ.

Bernd Schroder Louisiana Tech University, College of Engineering and Science

The Residue Theorem

Page 218: The Residue Theorem · 2010-10-04 · 5.We will prove the requisite theorem (the Residue Theorem) in this presentation and we will also lay the abstract groundwork. 6.We will then

logo1

Singularities Residues Residue Theorem Residue at Infinity Types of Isolated Singularities Residues at Poles

Proof. If f has a pole of order m around z0, then the Laurentexpansion of z for 0 < |z− z0|< R (for some R) is (with c−m 6= 0)

f (z) =∞

∑n=−m

cn(z− z0)n =1

(z− z0)m

∑n=−m

cn(z− z0)n+m

=1

(z− z0)m

∑k=0

ck−m(z− z0)k =:Φ(z)

(z− z0)m .

Conversely, if f (z) =Φ(z)

(z− z0)m with Φ(z0) 6= 0, then the above

computation in reverse shows that f has a pole of order m.

Moreover,in this situation, the coefficient c−1 = Resz=z0(f ) of the Laurent

expansion of f is the coefficient am−1 =Φ(m−1)(z0)(m−1)!

of the power

series expansion of Φ.

Bernd Schroder Louisiana Tech University, College of Engineering and Science

The Residue Theorem

Page 219: The Residue Theorem · 2010-10-04 · 5.We will prove the requisite theorem (the Residue Theorem) in this presentation and we will also lay the abstract groundwork. 6.We will then

logo1

Singularities Residues Residue Theorem Residue at Infinity Types of Isolated Singularities Residues at Poles

Proof. If f has a pole of order m around z0, then the Laurentexpansion of z for 0 < |z− z0|< R (for some R) is (with c−m 6= 0)

f (z) =∞

∑n=−m

cn(z− z0)n =1

(z− z0)m

∑n=−m

cn(z− z0)n+m

=1

(z− z0)m

∑k=0

ck−m(z− z0)k =:Φ(z)

(z− z0)m .

Conversely, if f (z) =Φ(z)

(z− z0)m with Φ(z0) 6= 0, then the above

computation in reverse shows that f has a pole of order m. Moreover,in this situation, the coefficient c−1 = Resz=z0(f ) of the Laurent

expansion of f

is the coefficient am−1 =Φ(m−1)(z0)(m−1)!

of the power

series expansion of Φ.

Bernd Schroder Louisiana Tech University, College of Engineering and Science

The Residue Theorem

Page 220: The Residue Theorem · 2010-10-04 · 5.We will prove the requisite theorem (the Residue Theorem) in this presentation and we will also lay the abstract groundwork. 6.We will then

logo1

Singularities Residues Residue Theorem Residue at Infinity Types of Isolated Singularities Residues at Poles

Proof. If f has a pole of order m around z0, then the Laurentexpansion of z for 0 < |z− z0|< R (for some R) is (with c−m 6= 0)

f (z) =∞

∑n=−m

cn(z− z0)n =1

(z− z0)m

∑n=−m

cn(z− z0)n+m

=1

(z− z0)m

∑k=0

ck−m(z− z0)k =:Φ(z)

(z− z0)m .

Conversely, if f (z) =Φ(z)

(z− z0)m with Φ(z0) 6= 0, then the above

computation in reverse shows that f has a pole of order m. Moreover,in this situation, the coefficient c−1 = Resz=z0(f ) of the Laurent

expansion of f is the coefficient am−1 =Φ(m−1)(z0)(m−1)!

of the power

series expansion of Φ.

Bernd Schroder Louisiana Tech University, College of Engineering and Science

The Residue Theorem

Page 221: The Residue Theorem · 2010-10-04 · 5.We will prove the requisite theorem (the Residue Theorem) in this presentation and we will also lay the abstract groundwork. 6.We will then

logo1

Singularities Residues Residue Theorem Residue at Infinity Types of Isolated Singularities Residues at Poles

Proof. If f has a pole of order m around z0, then the Laurentexpansion of z for 0 < |z− z0|< R (for some R) is (with c−m 6= 0)

f (z) =∞

∑n=−m

cn(z− z0)n =1

(z− z0)m

∑n=−m

cn(z− z0)n+m

=1

(z− z0)m

∑k=0

ck−m(z− z0)k =:Φ(z)

(z− z0)m .

Conversely, if f (z) =Φ(z)

(z− z0)m with Φ(z0) 6= 0, then the above

computation in reverse shows that f has a pole of order m. Moreover,in this situation, the coefficient c−1 = Resz=z0(f ) of the Laurent

expansion of f is the coefficient am−1 =Φ(m−1)(z0)(m−1)!

of the power

series expansion of Φ.

Bernd Schroder Louisiana Tech University, College of Engineering and Science

The Residue Theorem

Page 222: The Residue Theorem · 2010-10-04 · 5.We will prove the requisite theorem (the Residue Theorem) in this presentation and we will also lay the abstract groundwork. 6.We will then

logo1

Singularities Residues Residue Theorem Residue at Infinity Types of Isolated Singularities Residues at Poles

Example.

Find all poles, their order and their residues for

f (z) =(

z+1z−1

)3

.

The only singularity of f is z = 1.

f (z) =(

z+1z−1

)3

=1

(z−1)3

(z3 +3z2 +3z+1

)Φ(z) = z3 +3z2 +3z+1, m = 3

Φ′(z) = 3z2 +6z+3

Φ′′(z) = 6z+6

Φ′′(1) = 12

Resz=1(f ) =122!

= 6

Bernd Schroder Louisiana Tech University, College of Engineering and Science

The Residue Theorem

Page 223: The Residue Theorem · 2010-10-04 · 5.We will prove the requisite theorem (the Residue Theorem) in this presentation and we will also lay the abstract groundwork. 6.We will then

logo1

Singularities Residues Residue Theorem Residue at Infinity Types of Isolated Singularities Residues at Poles

Example. Find all poles, their order and their residues for

f (z) =(

z+1z−1

)3

.

The only singularity of f is z = 1.

f (z) =(

z+1z−1

)3

=1

(z−1)3

(z3 +3z2 +3z+1

)Φ(z) = z3 +3z2 +3z+1, m = 3

Φ′(z) = 3z2 +6z+3

Φ′′(z) = 6z+6

Φ′′(1) = 12

Resz=1(f ) =122!

= 6

Bernd Schroder Louisiana Tech University, College of Engineering and Science

The Residue Theorem

Page 224: The Residue Theorem · 2010-10-04 · 5.We will prove the requisite theorem (the Residue Theorem) in this presentation and we will also lay the abstract groundwork. 6.We will then

logo1

Singularities Residues Residue Theorem Residue at Infinity Types of Isolated Singularities Residues at Poles

Example. Find all poles, their order and their residues for

f (z) =(

z+1z−1

)3

.

The only singularity of f is z = 1.

f (z) =(

z+1z−1

)3

=1

(z−1)3

(z3 +3z2 +3z+1

)Φ(z) = z3 +3z2 +3z+1, m = 3

Φ′(z) = 3z2 +6z+3

Φ′′(z) = 6z+6

Φ′′(1) = 12

Resz=1(f ) =122!

= 6

Bernd Schroder Louisiana Tech University, College of Engineering and Science

The Residue Theorem

Page 225: The Residue Theorem · 2010-10-04 · 5.We will prove the requisite theorem (the Residue Theorem) in this presentation and we will also lay the abstract groundwork. 6.We will then

logo1

Singularities Residues Residue Theorem Residue at Infinity Types of Isolated Singularities Residues at Poles

Example. Find all poles, their order and their residues for

f (z) =(

z+1z−1

)3

.

The only singularity of f is z = 1.

f (z) =(

z+1z−1

)3

=1

(z−1)3

(z3 +3z2 +3z+1

)Φ(z) = z3 +3z2 +3z+1, m = 3

Φ′(z) = 3z2 +6z+3

Φ′′(z) = 6z+6

Φ′′(1) = 12

Resz=1(f ) =122!

= 6

Bernd Schroder Louisiana Tech University, College of Engineering and Science

The Residue Theorem

Page 226: The Residue Theorem · 2010-10-04 · 5.We will prove the requisite theorem (the Residue Theorem) in this presentation and we will also lay the abstract groundwork. 6.We will then

logo1

Singularities Residues Residue Theorem Residue at Infinity Types of Isolated Singularities Residues at Poles

Example. Find all poles, their order and their residues for

f (z) =(

z+1z−1

)3

.

The only singularity of f is z = 1.

f (z) =(

z+1z−1

)3

=1

(z−1)3

(z3 +3z2 +3z+1

)

Φ(z) = z3 +3z2 +3z+1, m = 3

Φ′(z) = 3z2 +6z+3

Φ′′(z) = 6z+6

Φ′′(1) = 12

Resz=1(f ) =122!

= 6

Bernd Schroder Louisiana Tech University, College of Engineering and Science

The Residue Theorem

Page 227: The Residue Theorem · 2010-10-04 · 5.We will prove the requisite theorem (the Residue Theorem) in this presentation and we will also lay the abstract groundwork. 6.We will then

logo1

Singularities Residues Residue Theorem Residue at Infinity Types of Isolated Singularities Residues at Poles

Example. Find all poles, their order and their residues for

f (z) =(

z+1z−1

)3

.

The only singularity of f is z = 1.

f (z) =(

z+1z−1

)3

=1

(z−1)3

(z3 +3z2 +3z+1

)Φ(z) = z3 +3z2 +3z+1

, m = 3

Φ′(z) = 3z2 +6z+3

Φ′′(z) = 6z+6

Φ′′(1) = 12

Resz=1(f ) =122!

= 6

Bernd Schroder Louisiana Tech University, College of Engineering and Science

The Residue Theorem

Page 228: The Residue Theorem · 2010-10-04 · 5.We will prove the requisite theorem (the Residue Theorem) in this presentation and we will also lay the abstract groundwork. 6.We will then

logo1

Singularities Residues Residue Theorem Residue at Infinity Types of Isolated Singularities Residues at Poles

Example. Find all poles, their order and their residues for

f (z) =(

z+1z−1

)3

.

The only singularity of f is z = 1.

f (z) =(

z+1z−1

)3

=1

(z−1)3

(z3 +3z2 +3z+1

)Φ(z) = z3 +3z2 +3z+1, m = 3

Φ′(z) = 3z2 +6z+3

Φ′′(z) = 6z+6

Φ′′(1) = 12

Resz=1(f ) =122!

= 6

Bernd Schroder Louisiana Tech University, College of Engineering and Science

The Residue Theorem

Page 229: The Residue Theorem · 2010-10-04 · 5.We will prove the requisite theorem (the Residue Theorem) in this presentation and we will also lay the abstract groundwork. 6.We will then

logo1

Singularities Residues Residue Theorem Residue at Infinity Types of Isolated Singularities Residues at Poles

Example. Find all poles, their order and their residues for

f (z) =(

z+1z−1

)3

.

The only singularity of f is z = 1.

f (z) =(

z+1z−1

)3

=1

(z−1)3

(z3 +3z2 +3z+1

)Φ(z) = z3 +3z2 +3z+1, m = 3

Φ′(z) = 3z2 +6z+3

Φ′′(z) = 6z+6

Φ′′(1) = 12

Resz=1(f ) =122!

= 6

Bernd Schroder Louisiana Tech University, College of Engineering and Science

The Residue Theorem

Page 230: The Residue Theorem · 2010-10-04 · 5.We will prove the requisite theorem (the Residue Theorem) in this presentation and we will also lay the abstract groundwork. 6.We will then

logo1

Singularities Residues Residue Theorem Residue at Infinity Types of Isolated Singularities Residues at Poles

Example. Find all poles, their order and their residues for

f (z) =(

z+1z−1

)3

.

The only singularity of f is z = 1.

f (z) =(

z+1z−1

)3

=1

(z−1)3

(z3 +3z2 +3z+1

)Φ(z) = z3 +3z2 +3z+1, m = 3

Φ′(z) = 3z2 +6z+3

Φ′′(z) = 6z+6

Φ′′(1) = 12

Resz=1(f ) =122!

= 6

Bernd Schroder Louisiana Tech University, College of Engineering and Science

The Residue Theorem

Page 231: The Residue Theorem · 2010-10-04 · 5.We will prove the requisite theorem (the Residue Theorem) in this presentation and we will also lay the abstract groundwork. 6.We will then

logo1

Singularities Residues Residue Theorem Residue at Infinity Types of Isolated Singularities Residues at Poles

Example. Find all poles, their order and their residues for

f (z) =(

z+1z−1

)3

.

The only singularity of f is z = 1.

f (z) =(

z+1z−1

)3

=1

(z−1)3

(z3 +3z2 +3z+1

)Φ(z) = z3 +3z2 +3z+1, m = 3

Φ′(z) = 3z2 +6z+3

Φ′′(z) = 6z+6

Φ′′(1) = 12

Resz=1(f ) =122!

= 6

Bernd Schroder Louisiana Tech University, College of Engineering and Science

The Residue Theorem

Page 232: The Residue Theorem · 2010-10-04 · 5.We will prove the requisite theorem (the Residue Theorem) in this presentation and we will also lay the abstract groundwork. 6.We will then

logo1

Singularities Residues Residue Theorem Residue at Infinity Types of Isolated Singularities Residues at Poles

Example. Find all poles, their order and their residues for

f (z) =(

z+1z−1

)3

.

The only singularity of f is z = 1.

f (z) =(

z+1z−1

)3

=1

(z−1)3

(z3 +3z2 +3z+1

)Φ(z) = z3 +3z2 +3z+1, m = 3

Φ′(z) = 3z2 +6z+3

Φ′′(z) = 6z+6

Φ′′(1) = 12

Resz=1(f ) =122!

= 6

Bernd Schroder Louisiana Tech University, College of Engineering and Science

The Residue Theorem

Page 233: The Residue Theorem · 2010-10-04 · 5.We will prove the requisite theorem (the Residue Theorem) in this presentation and we will also lay the abstract groundwork. 6.We will then

logo1

Singularities Residues Residue Theorem Residue at Infinity Types of Isolated Singularities Residues at Poles

Example. Find all poles, their order and their residues for

f (z) =(

z+1z−1

)3

.

The only singularity of f is z = 1.

f (z) =(

z+1z−1

)3

=1

(z−1)3

(z3 +3z2 +3z+1

)Φ(z) = z3 +3z2 +3z+1, m = 3

Φ′(z) = 3z2 +6z+3

Φ′′(z) = 6z+6

Φ′′(1) = 12

Resz=1(f ) =122!

= 6

Bernd Schroder Louisiana Tech University, College of Engineering and Science

The Residue Theorem

Page 234: The Residue Theorem · 2010-10-04 · 5.We will prove the requisite theorem (the Residue Theorem) in this presentation and we will also lay the abstract groundwork. 6.We will then

logo1

Singularities Residues Residue Theorem Residue at Infinity Types of Isolated Singularities Residues at Poles

Example.

Show that Resz=iLog(z)

(z2 +1)2 =π +2i

8.

Log(z)

(z2 +1)2 =Log(z)(

(z+ i)(z− i))2 =

1(z− i)2

Log(z)(z+ i)2

Φ(z) =Log(z)(z+ i)2 , m = 2

Φ′(z) =

1z (z+ i)2−2(z+ i)Log(z)

(z+ i)4 =1z (z+ i)−2Log(z)

(z+ i)3

=z+ i−2Log(z)z

z(z+ i)3

Φ′(i) =

i+ i−2Log(i)ii(i+ i)3 =

2i−2i π

2 ii(2i)3 =

2i+π

8

Resz=iLog(z)

(z2 +1)2 =11!

π +2i8

=π +2i

8

Bernd Schroder Louisiana Tech University, College of Engineering and Science

The Residue Theorem

Page 235: The Residue Theorem · 2010-10-04 · 5.We will prove the requisite theorem (the Residue Theorem) in this presentation and we will also lay the abstract groundwork. 6.We will then

logo1

Singularities Residues Residue Theorem Residue at Infinity Types of Isolated Singularities Residues at Poles

Example. Show that Resz=iLog(z)

(z2 +1)2 =π +2i

8.

Log(z)

(z2 +1)2 =Log(z)(

(z+ i)(z− i))2 =

1(z− i)2

Log(z)(z+ i)2

Φ(z) =Log(z)(z+ i)2 , m = 2

Φ′(z) =

1z (z+ i)2−2(z+ i)Log(z)

(z+ i)4 =1z (z+ i)−2Log(z)

(z+ i)3

=z+ i−2Log(z)z

z(z+ i)3

Φ′(i) =

i+ i−2Log(i)ii(i+ i)3 =

2i−2i π

2 ii(2i)3 =

2i+π

8

Resz=iLog(z)

(z2 +1)2 =11!

π +2i8

=π +2i

8

Bernd Schroder Louisiana Tech University, College of Engineering and Science

The Residue Theorem

Page 236: The Residue Theorem · 2010-10-04 · 5.We will prove the requisite theorem (the Residue Theorem) in this presentation and we will also lay the abstract groundwork. 6.We will then

logo1

Singularities Residues Residue Theorem Residue at Infinity Types of Isolated Singularities Residues at Poles

Example. Show that Resz=iLog(z)

(z2 +1)2 =π +2i

8.

Log(z)

(z2 +1)2 =Log(z)(

(z+ i)(z− i))2

=1

(z− i)2Log(z)(z+ i)2

Φ(z) =Log(z)(z+ i)2 , m = 2

Φ′(z) =

1z (z+ i)2−2(z+ i)Log(z)

(z+ i)4 =1z (z+ i)−2Log(z)

(z+ i)3

=z+ i−2Log(z)z

z(z+ i)3

Φ′(i) =

i+ i−2Log(i)ii(i+ i)3 =

2i−2i π

2 ii(2i)3 =

2i+π

8

Resz=iLog(z)

(z2 +1)2 =11!

π +2i8

=π +2i

8

Bernd Schroder Louisiana Tech University, College of Engineering and Science

The Residue Theorem

Page 237: The Residue Theorem · 2010-10-04 · 5.We will prove the requisite theorem (the Residue Theorem) in this presentation and we will also lay the abstract groundwork. 6.We will then

logo1

Singularities Residues Residue Theorem Residue at Infinity Types of Isolated Singularities Residues at Poles

Example. Show that Resz=iLog(z)

(z2 +1)2 =π +2i

8.

Log(z)

(z2 +1)2 =Log(z)(

(z+ i)(z− i))2 =

1(z− i)2

Log(z)(z+ i)2

Φ(z) =Log(z)(z+ i)2 , m = 2

Φ′(z) =

1z (z+ i)2−2(z+ i)Log(z)

(z+ i)4 =1z (z+ i)−2Log(z)

(z+ i)3

=z+ i−2Log(z)z

z(z+ i)3

Φ′(i) =

i+ i−2Log(i)ii(i+ i)3 =

2i−2i π

2 ii(2i)3 =

2i+π

8

Resz=iLog(z)

(z2 +1)2 =11!

π +2i8

=π +2i

8

Bernd Schroder Louisiana Tech University, College of Engineering and Science

The Residue Theorem

Page 238: The Residue Theorem · 2010-10-04 · 5.We will prove the requisite theorem (the Residue Theorem) in this presentation and we will also lay the abstract groundwork. 6.We will then

logo1

Singularities Residues Residue Theorem Residue at Infinity Types of Isolated Singularities Residues at Poles

Example. Show that Resz=iLog(z)

(z2 +1)2 =π +2i

8.

Log(z)

(z2 +1)2 =Log(z)(

(z+ i)(z− i))2 =

1(z− i)2

Log(z)(z+ i)2

Φ(z) =Log(z)(z+ i)2

, m = 2

Φ′(z) =

1z (z+ i)2−2(z+ i)Log(z)

(z+ i)4 =1z (z+ i)−2Log(z)

(z+ i)3

=z+ i−2Log(z)z

z(z+ i)3

Φ′(i) =

i+ i−2Log(i)ii(i+ i)3 =

2i−2i π

2 ii(2i)3 =

2i+π

8

Resz=iLog(z)

(z2 +1)2 =11!

π +2i8

=π +2i

8

Bernd Schroder Louisiana Tech University, College of Engineering and Science

The Residue Theorem

Page 239: The Residue Theorem · 2010-10-04 · 5.We will prove the requisite theorem (the Residue Theorem) in this presentation and we will also lay the abstract groundwork. 6.We will then

logo1

Singularities Residues Residue Theorem Residue at Infinity Types of Isolated Singularities Residues at Poles

Example. Show that Resz=iLog(z)

(z2 +1)2 =π +2i

8.

Log(z)

(z2 +1)2 =Log(z)(

(z+ i)(z− i))2 =

1(z− i)2

Log(z)(z+ i)2

Φ(z) =Log(z)(z+ i)2 , m = 2

Φ′(z) =

1z (z+ i)2−2(z+ i)Log(z)

(z+ i)4 =1z (z+ i)−2Log(z)

(z+ i)3

=z+ i−2Log(z)z

z(z+ i)3

Φ′(i) =

i+ i−2Log(i)ii(i+ i)3 =

2i−2i π

2 ii(2i)3 =

2i+π

8

Resz=iLog(z)

(z2 +1)2 =11!

π +2i8

=π +2i

8

Bernd Schroder Louisiana Tech University, College of Engineering and Science

The Residue Theorem

Page 240: The Residue Theorem · 2010-10-04 · 5.We will prove the requisite theorem (the Residue Theorem) in this presentation and we will also lay the abstract groundwork. 6.We will then

logo1

Singularities Residues Residue Theorem Residue at Infinity Types of Isolated Singularities Residues at Poles

Example. Show that Resz=iLog(z)

(z2 +1)2 =π +2i

8.

Log(z)

(z2 +1)2 =Log(z)(

(z+ i)(z− i))2 =

1(z− i)2

Log(z)(z+ i)2

Φ(z) =Log(z)(z+ i)2 , m = 2

Φ′(z) =

1z (z+ i)2−2(z+ i)Log(z)

(z+ i)4

=1z (z+ i)−2Log(z)

(z+ i)3

=z+ i−2Log(z)z

z(z+ i)3

Φ′(i) =

i+ i−2Log(i)ii(i+ i)3 =

2i−2i π

2 ii(2i)3 =

2i+π

8

Resz=iLog(z)

(z2 +1)2 =11!

π +2i8

=π +2i

8

Bernd Schroder Louisiana Tech University, College of Engineering and Science

The Residue Theorem

Page 241: The Residue Theorem · 2010-10-04 · 5.We will prove the requisite theorem (the Residue Theorem) in this presentation and we will also lay the abstract groundwork. 6.We will then

logo1

Singularities Residues Residue Theorem Residue at Infinity Types of Isolated Singularities Residues at Poles

Example. Show that Resz=iLog(z)

(z2 +1)2 =π +2i

8.

Log(z)

(z2 +1)2 =Log(z)(

(z+ i)(z− i))2 =

1(z− i)2

Log(z)(z+ i)2

Φ(z) =Log(z)(z+ i)2 , m = 2

Φ′(z) =

1z (z+ i)2−2(z+ i)Log(z)

(z+ i)4 =1z (z+ i)−2Log(z)

(z+ i)3

=z+ i−2Log(z)z

z(z+ i)3

Φ′(i) =

i+ i−2Log(i)ii(i+ i)3 =

2i−2i π

2 ii(2i)3 =

2i+π

8

Resz=iLog(z)

(z2 +1)2 =11!

π +2i8

=π +2i

8

Bernd Schroder Louisiana Tech University, College of Engineering and Science

The Residue Theorem

Page 242: The Residue Theorem · 2010-10-04 · 5.We will prove the requisite theorem (the Residue Theorem) in this presentation and we will also lay the abstract groundwork. 6.We will then

logo1

Singularities Residues Residue Theorem Residue at Infinity Types of Isolated Singularities Residues at Poles

Example. Show that Resz=iLog(z)

(z2 +1)2 =π +2i

8.

Log(z)

(z2 +1)2 =Log(z)(

(z+ i)(z− i))2 =

1(z− i)2

Log(z)(z+ i)2

Φ(z) =Log(z)(z+ i)2 , m = 2

Φ′(z) =

1z (z+ i)2−2(z+ i)Log(z)

(z+ i)4 =1z (z+ i)−2Log(z)

(z+ i)3

=z+ i−2Log(z)z

z(z+ i)3

Φ′(i) =

i+ i−2Log(i)ii(i+ i)3 =

2i−2i π

2 ii(2i)3 =

2i+π

8

Resz=iLog(z)

(z2 +1)2 =11!

π +2i8

=π +2i

8

Bernd Schroder Louisiana Tech University, College of Engineering and Science

The Residue Theorem

Page 243: The Residue Theorem · 2010-10-04 · 5.We will prove the requisite theorem (the Residue Theorem) in this presentation and we will also lay the abstract groundwork. 6.We will then

logo1

Singularities Residues Residue Theorem Residue at Infinity Types of Isolated Singularities Residues at Poles

Example. Show that Resz=iLog(z)

(z2 +1)2 =π +2i

8.

Log(z)

(z2 +1)2 =Log(z)(

(z+ i)(z− i))2 =

1(z− i)2

Log(z)(z+ i)2

Φ(z) =Log(z)(z+ i)2 , m = 2

Φ′(z) =

1z (z+ i)2−2(z+ i)Log(z)

(z+ i)4 =1z (z+ i)−2Log(z)

(z+ i)3

=z+ i−2Log(z)z

z(z+ i)3

Φ′(i) =

i+ i−2Log(i)ii(i+ i)3

=2i−2i π

2 ii(2i)3 =

2i+π

8

Resz=iLog(z)

(z2 +1)2 =11!

π +2i8

=π +2i

8

Bernd Schroder Louisiana Tech University, College of Engineering and Science

The Residue Theorem

Page 244: The Residue Theorem · 2010-10-04 · 5.We will prove the requisite theorem (the Residue Theorem) in this presentation and we will also lay the abstract groundwork. 6.We will then

logo1

Singularities Residues Residue Theorem Residue at Infinity Types of Isolated Singularities Residues at Poles

Example. Show that Resz=iLog(z)

(z2 +1)2 =π +2i

8.

Log(z)

(z2 +1)2 =Log(z)(

(z+ i)(z− i))2 =

1(z− i)2

Log(z)(z+ i)2

Φ(z) =Log(z)(z+ i)2 , m = 2

Φ′(z) =

1z (z+ i)2−2(z+ i)Log(z)

(z+ i)4 =1z (z+ i)−2Log(z)

(z+ i)3

=z+ i−2Log(z)z

z(z+ i)3

Φ′(i) =

i+ i−2Log(i)ii(i+ i)3 =

2i−2i π

2 ii(2i)3

=2i+π

8

Resz=iLog(z)

(z2 +1)2 =11!

π +2i8

=π +2i

8

Bernd Schroder Louisiana Tech University, College of Engineering and Science

The Residue Theorem

Page 245: The Residue Theorem · 2010-10-04 · 5.We will prove the requisite theorem (the Residue Theorem) in this presentation and we will also lay the abstract groundwork. 6.We will then

logo1

Singularities Residues Residue Theorem Residue at Infinity Types of Isolated Singularities Residues at Poles

Example. Show that Resz=iLog(z)

(z2 +1)2 =π +2i

8.

Log(z)

(z2 +1)2 =Log(z)(

(z+ i)(z− i))2 =

1(z− i)2

Log(z)(z+ i)2

Φ(z) =Log(z)(z+ i)2 , m = 2

Φ′(z) =

1z (z+ i)2−2(z+ i)Log(z)

(z+ i)4 =1z (z+ i)−2Log(z)

(z+ i)3

=z+ i−2Log(z)z

z(z+ i)3

Φ′(i) =

i+ i−2Log(i)ii(i+ i)3 =

2i−2i π

2 ii(2i)3 =

2i+π

8

Resz=iLog(z)

(z2 +1)2 =11!

π +2i8

=π +2i

8

Bernd Schroder Louisiana Tech University, College of Engineering and Science

The Residue Theorem

Page 246: The Residue Theorem · 2010-10-04 · 5.We will prove the requisite theorem (the Residue Theorem) in this presentation and we will also lay the abstract groundwork. 6.We will then

logo1

Singularities Residues Residue Theorem Residue at Infinity Types of Isolated Singularities Residues at Poles

Example. Show that Resz=iLog(z)

(z2 +1)2 =π +2i

8.

Log(z)

(z2 +1)2 =Log(z)(

(z+ i)(z− i))2 =

1(z− i)2

Log(z)(z+ i)2

Φ(z) =Log(z)(z+ i)2 , m = 2

Φ′(z) =

1z (z+ i)2−2(z+ i)Log(z)

(z+ i)4 =1z (z+ i)−2Log(z)

(z+ i)3

=z+ i−2Log(z)z

z(z+ i)3

Φ′(i) =

i+ i−2Log(i)ii(i+ i)3 =

2i−2i π

2 ii(2i)3 =

2i+π

8

Resz=iLog(z)

(z2 +1)2 =11!

π +2i8

=π +2i

8

Bernd Schroder Louisiana Tech University, College of Engineering and Science

The Residue Theorem

Page 247: The Residue Theorem · 2010-10-04 · 5.We will prove the requisite theorem (the Residue Theorem) in this presentation and we will also lay the abstract groundwork. 6.We will then

logo1

Singularities Residues Residue Theorem Residue at Infinity Types of Isolated Singularities Residues at Poles

Example. Show that Resz=iLog(z)

(z2 +1)2 =π +2i

8.

Log(z)

(z2 +1)2 =Log(z)(

(z+ i)(z− i))2 =

1(z− i)2

Log(z)(z+ i)2

Φ(z) =Log(z)(z+ i)2 , m = 2

Φ′(z) =

1z (z+ i)2−2(z+ i)Log(z)

(z+ i)4 =1z (z+ i)−2Log(z)

(z+ i)3

=z+ i−2Log(z)z

z(z+ i)3

Φ′(i) =

i+ i−2Log(i)ii(i+ i)3 =

2i−2i π

2 ii(2i)3 =

2i+π

8

Resz=iLog(z)

(z2 +1)2 =11!

π +2i8

=π +2i

8

Bernd Schroder Louisiana Tech University, College of Engineering and Science

The Residue Theorem

Page 248: The Residue Theorem · 2010-10-04 · 5.We will prove the requisite theorem (the Residue Theorem) in this presentation and we will also lay the abstract groundwork. 6.We will then

logo1

Singularities Residues Residue Theorem Residue at Infinity Types of Isolated Singularities Residues at Poles

Example.

Integrate f (z) =1

z3(z+4)around the positively oriented

circle of radius 5 around the origin.

We will need the residues of f at 0 and at −4, or we need the residue

at 0 of1z2 f(

1z

).

1z2 f(

1z

)=

1z2

1(1z

)3 (1z +4

) =1

1z

1+4zz

=z2

1+4z

As1z2 f(

1z

)does not have a singularity at 0, the integral must be 0.

Bernd Schroder Louisiana Tech University, College of Engineering and Science

The Residue Theorem

Page 249: The Residue Theorem · 2010-10-04 · 5.We will prove the requisite theorem (the Residue Theorem) in this presentation and we will also lay the abstract groundwork. 6.We will then

logo1

Singularities Residues Residue Theorem Residue at Infinity Types of Isolated Singularities Residues at Poles

Example. Integrate f (z) =1

z3(z+4)around the positively oriented

circle of radius 5 around the origin.

We will need the residues of f at 0 and at −4, or we need the residue

at 0 of1z2 f(

1z

).

1z2 f(

1z

)=

1z2

1(1z

)3 (1z +4

) =1

1z

1+4zz

=z2

1+4z

As1z2 f(

1z

)does not have a singularity at 0, the integral must be 0.

Bernd Schroder Louisiana Tech University, College of Engineering and Science

The Residue Theorem

Page 250: The Residue Theorem · 2010-10-04 · 5.We will prove the requisite theorem (the Residue Theorem) in this presentation and we will also lay the abstract groundwork. 6.We will then

logo1

Singularities Residues Residue Theorem Residue at Infinity Types of Isolated Singularities Residues at Poles

Example. Integrate f (z) =1

z3(z+4)around the positively oriented

circle of radius 5 around the origin.

We will need the residues of f at 0 and at −4

, or we need the residue

at 0 of1z2 f(

1z

).

1z2 f(

1z

)=

1z2

1(1z

)3 (1z +4

) =1

1z

1+4zz

=z2

1+4z

As1z2 f(

1z

)does not have a singularity at 0, the integral must be 0.

Bernd Schroder Louisiana Tech University, College of Engineering and Science

The Residue Theorem

Page 251: The Residue Theorem · 2010-10-04 · 5.We will prove the requisite theorem (the Residue Theorem) in this presentation and we will also lay the abstract groundwork. 6.We will then

logo1

Singularities Residues Residue Theorem Residue at Infinity Types of Isolated Singularities Residues at Poles

Example. Integrate f (z) =1

z3(z+4)around the positively oriented

circle of radius 5 around the origin.

We will need the residues of f at 0 and at −4, or we need the residue

at 0 of1z2 f(

1z

).

1z2 f(

1z

)=

1z2

1(1z

)3 (1z +4

) =1

1z

1+4zz

=z2

1+4z

As1z2 f(

1z

)does not have a singularity at 0, the integral must be 0.

Bernd Schroder Louisiana Tech University, College of Engineering and Science

The Residue Theorem

Page 252: The Residue Theorem · 2010-10-04 · 5.We will prove the requisite theorem (the Residue Theorem) in this presentation and we will also lay the abstract groundwork. 6.We will then

logo1

Singularities Residues Residue Theorem Residue at Infinity Types of Isolated Singularities Residues at Poles

Example. Integrate f (z) =1

z3(z+4)around the positively oriented

circle of radius 5 around the origin.

We will need the residues of f at 0 and at −4, or we need the residue

at 0 of1z2 f(

1z

).

1z2 f(

1z

)

=1z2

1(1z

)3 (1z +4

) =1

1z

1+4zz

=z2

1+4z

As1z2 f(

1z

)does not have a singularity at 0, the integral must be 0.

Bernd Schroder Louisiana Tech University, College of Engineering and Science

The Residue Theorem

Page 253: The Residue Theorem · 2010-10-04 · 5.We will prove the requisite theorem (the Residue Theorem) in this presentation and we will also lay the abstract groundwork. 6.We will then

logo1

Singularities Residues Residue Theorem Residue at Infinity Types of Isolated Singularities Residues at Poles

Example. Integrate f (z) =1

z3(z+4)around the positively oriented

circle of radius 5 around the origin.

We will need the residues of f at 0 and at −4, or we need the residue

at 0 of1z2 f(

1z

).

1z2 f(

1z

)=

1z2

1(1z

)3 (1z +4

)

=1

1z

1+4zz

=z2

1+4z

As1z2 f(

1z

)does not have a singularity at 0, the integral must be 0.

Bernd Schroder Louisiana Tech University, College of Engineering and Science

The Residue Theorem

Page 254: The Residue Theorem · 2010-10-04 · 5.We will prove the requisite theorem (the Residue Theorem) in this presentation and we will also lay the abstract groundwork. 6.We will then

logo1

Singularities Residues Residue Theorem Residue at Infinity Types of Isolated Singularities Residues at Poles

Example. Integrate f (z) =1

z3(z+4)around the positively oriented

circle of radius 5 around the origin.

We will need the residues of f at 0 and at −4, or we need the residue

at 0 of1z2 f(

1z

).

1z2 f(

1z

)=

1z2

1(1z

)3 (1z +4

) =1

1z

1+4zz

=z2

1+4z

As1z2 f(

1z

)does not have a singularity at 0, the integral must be 0.

Bernd Schroder Louisiana Tech University, College of Engineering and Science

The Residue Theorem

Page 255: The Residue Theorem · 2010-10-04 · 5.We will prove the requisite theorem (the Residue Theorem) in this presentation and we will also lay the abstract groundwork. 6.We will then

logo1

Singularities Residues Residue Theorem Residue at Infinity Types of Isolated Singularities Residues at Poles

Example. Integrate f (z) =1

z3(z+4)around the positively oriented

circle of radius 5 around the origin.

We will need the residues of f at 0 and at −4, or we need the residue

at 0 of1z2 f(

1z

).

1z2 f(

1z

)=

1z2

1(1z

)3 (1z +4

) =1

1z

1+4zz

=z2

1+4z

As1z2 f(

1z

)does not have a singularity at 0, the integral must be 0.

Bernd Schroder Louisiana Tech University, College of Engineering and Science

The Residue Theorem

Page 256: The Residue Theorem · 2010-10-04 · 5.We will prove the requisite theorem (the Residue Theorem) in this presentation and we will also lay the abstract groundwork. 6.We will then

logo1

Singularities Residues Residue Theorem Residue at Infinity Types of Isolated Singularities Residues at Poles

Example. Integrate f (z) =1

z3(z+4)around the positively oriented

circle of radius 5 around the origin.

We will need the residues of f at 0 and at −4, or we need the residue

at 0 of1z2 f(

1z

).

1z2 f(

1z

)=

1z2

1(1z

)3 (1z +4

) =1

1z

1+4zz

=z2

1+4z

As1z2 f(

1z

)does not have a singularity at 0, the integral must be 0.

Bernd Schroder Louisiana Tech University, College of Engineering and Science

The Residue Theorem